Sie sind auf Seite 1von 476

GMATPrep - Default Pack Exam1_Repeat01 - Verbal - SENTENCE CORR ANALYSIS Page - 1

Question 09 03
Question 10 33
Question 14 51
Question 15 79
Question 16 111
Question 18 123
Question 19 145
Question 20 167
Question 23 177
Question 32 195
Question 34 255
Question 35 275
Question 37 305
Question 38 355
Question 40 409

Question 41 445
GMATPrep - Default Pack Exam1_Repeat01 - Verbal - SENTENCE CORR ANALYSIS Page - 2
Question 09
GMATPrep - Default Pack Exam1_Repeat01 - Verbal - SENTENCE CORR ANALYSIS Page - 4

Difficulty: 65% (MEDIUM)


Question Stats: 52% (01:37) correct 48% (00:54) wrong based on 1526 sessions
A:18% B:51% C:24% D:3% E:3%

https://gmatclub.com/forum/with-surface-temperatures-estimated-at-minus-230-degrees-142362.html

Marcab
VP
answer must be B
First of all we do have to consider the correct usage of "With".
ex- "With 4 childrens at home, Rachel barely has any spare time for her hobbies".
Notice that the two clauses of the above sentence are reinforcing into each other.
So the sentence must complete after the second clause. If it does'nt, then the third clause must be a modifier, modifying the
preceding clause.
B does it correctly.
", its 60 square miles of water thought to be frozen from top to bottom" is an absolute phrase that is modifying " Europa has long
been considered far too cold to support life".
Whats the OA and the source?
-s

EducationAisle
Director

anujag24 wrote:
I am still looking for reasons on use of clause starting with "its" in choice B. Is this correct usage?

With B, the sentence would be:

With surface temperatures estimated at minus 230 degrees Fahrenheit, Jupiter's moon Europa has long been considered far too cold to support life, its
60 square miles of water thought to be frozen from top to bottom.

Question 09
GMATPrep - Default Pack Exam1_Repeat01 - Verbal - SENTENCE CORR ANALYSIS Page - 5

The construct its 60 square miles of water thought to be frozen from top to bottom is not a clause.

This construct is basically an absolute modifier: Noun (its 60 square miles of water) + Noun modifier (thought to be frozen from top to bottom, in this
case, a past participial phrase).

p.s. Our book SC Nirvana discusses Absolute modifiers, their application and examples in significant detail. If you can PM you email, I can send you
the corresponding section.
________

EducationAisle
Director

With C, the sentence would be:

With surface temperatures estimated at minus 230 degrees Fahrenheit, Jupiter's moon Europa has long been considered as far too cold to support life
and has 60 square miles of water thought to be frozen from top to bottom.

Even if it is just considered, C has a meaning issue. The intended meaning is that Europa is too cold to support life; a proof given to this effect is that its
60 square miles of water is thought to be frozen from top to bottom.

However, C uses and and hence seems to incorrectly suggest two independent things about Europa:
i) Europa is too cold to support life
ii) Europa has 60 square miles of water thought to be frozen from top to bottom
_________________

Thanks,
Ashish (GMAT Faculty @ EducationAisle)

Question 09
GMATPrep - Default Pack Exam1_Repeat01 - Verbal - SENTENCE CORR ANALYSIS Page - 6

https://e-gmat.com/blogs/with-surface-temperatures-estimated-at%E2%80%A6gmatprep/

What does this question test?


This question tests your knowledge of how comma + and is used to connect elements in a sentence. It also tests your understanding of modifiers that
modify the preceding clause. In addition, it tests your knowledge of idiomatic usage of considered.

What does this sentence mean?


This sentence states that Jupiters moon Europa has been considered far too cold to support life. It also presents data corroborating this fact.

1. Surface temperatures are estimated at minus 230 degrees F.


2. 60 square miles of water exists on Europa. This water is thought to be frozen from top to bottom.

Note that the above is the logical meaning.

As such the sentence structure is such that we do not know how and with 60 square miles relates to the main clause Europa has long been
considered . This is because the sentence appears to be incomplete. Thus, this sentence does not communicate complete meaning effectively.

What are the errors in the original sentence?


With surface temperatures estimated at minus 230 degrees Fahrenheit, Jupiters moon Europa has long been considered far too cold to support life, and
with 60 square miles of water thought to be frozen from top to bottom.

Question 09
GMATPrep - Default Pack Exam1_Repeat01 - Verbal - SENTENCE CORR ANALYSIS Page - 7

The sentence structure is as follows:

Clause 1: With surface temperatures estimated at minus 230 degrees Fahrenheit, Jupiters moon Europa has long been considered far too cold to
support life,
Clause 2: and with 60 square miles of water thought to be frozen from top to bottom.

Notice carefully the usage of comma + and. This construction is used to express one of the following:

1. Connect more than 2 elements in a list


2. Connect two independent clauses

It does not make sense for with 60 square miles to be part of a list in this sentence. Thus, first usage is not applicable here. For second usage to be
applicable, what follows and should be an independent clause. But as you can see, there is no identifiable subject or verb in this supposed clause 2.

Thus, Choice A is incorrect because of incorrect use of comma + and. Essentially this usage has resulted in a fragment.

Answer Choice Analysis:


Choice A: Fragment Error.

Choice B: No Errors.

In this choice what follows the comma is a modifier its 60 square miles of water thought to be frozen from top to bottom, which modifies the
complete preceding clause. Such modifiers have following construction:

noun + noun modifier

In this case:

NOUN = its 60 square miles of water

Question 09
GMATPrep - Default Pack Exam1_Repeat01 - Verbal - SENTENCE CORR ANALYSIS Page - 8

Noun Modifier = verb-ed modifier thought to be frozen from top to bottom

In the context of this sentence, this modifier provides supports to the thought communicated in the previous clause Europa is considered far too cold to
support life. This modifier provides an evidence for the same such vast stretch of water is thought to be frozen.

Choice C: Idiom Error considered as is un-idiomatic usage.

Choice D: This choice has two errors:

1: Idiom Error as in Choice C.

2: Fragment Error: There is no verb in this sentence. Notice how has been considered has been converted into a verb-ed modifier.

Choice E: This choice has two errors

1: Idiom Error: Considered to be is un-idiomatic usage.

2: Fragment Error as in Choice D.

Thus, Choice B is the correct answer.

What are the key take-away messages?


1. When you see comma + and, ensure that it is used appropriately to either connect more than 2 elements in list or connect 2 independent clauses.
2. Considered as and considered to be are incorrect idioms

3. Modifiers of the construction noun + noun modifier can modify the preceding clause.

Question 09
GMATPrep - Default Pack Exam1_Repeat01 - Verbal - SENTENCE CORR ANALYSIS Page - 9

http://www.beatthegmat.com/sc-with-surface-temperatures-estimated-at-minus-230-degrees-t43734-15.html

brief explanations:

(a)
"and with ..." isn't parallel to anything.
AND sets up parallelism, so there must be something to which "with..." can be parallel (other prepositional phrase, or other
adverbial modifier). there is no such thing.

(b)
correct.
this is a type of modifier with which you should be familiar. (i have no idea what it's called - sorry)
here's another example:
john, his arms flailing in the wind, called out desperately for help.

note that the presence of frozen water SUPPORTS the claim that europa is "far too cold to support life", so it should
be a MODIFIER.
this is done here.
it's inappropriate to place these two things in parallel with AND.

(c)
AND is rhetorically inappropriate (see above).
"considered as" is unidiomatic here.

(d)
this is not a sentence. (the clause before "and" doesn't have a verb; "considered" is a participle, not a verb, here)
"considered as" is unidiomatic here.

Question 09
GMATPrep - Default Pack Exam1_Repeat01 - Verbal - SENTENCE CORR ANALYSIS Page - 10

(e)
i don't think "considered to be" is wrong, although it's wordier than just "considered..."
this sentence has no verb at all! the only verb forms present are participles and infinitives, none of which is eligible to be the main
verb of the sentence.

lunarpower GMAT Instructor

duongthang wrote:
"main clause, noun+which clause/adjective"---she is beautiful, a fact I know---

in this pattern, noun phrase is a summary of main clause.

yes.
this sort of modifier (COMMA + ABSTRACT NOUN) can be used to refer back to the WHOLE IDEA of the preceding clause.

let's say that scientists discover that X is 60 percent of Y, and that they are shocked by this finding.

then:
recent studies have shown that X is 60 percent of Y, which has shocked many in the scientific community.
incorrect.
this sentence implies that Y itself has shocked many in the scientific community. not the point.

recent studies have shown that X is 60 percent of Y, a finding that has shocked many in the scientific community.
or
recent studies have shown that X is 60 percent of Y, a statistic that has shocked many in the scientific community.
these are correct.

Question 09
GMATPrep - Default Pack Exam1_Repeat01 - Verbal - SENTENCE CORR ANALYSIS Page - 11

the noun "finding" or "statistic" refers to the whole idea of the preceding clause.

in fact, this is an essential reason for the very existence of these modifiers. they don't even exist in spoken language, but, in written english, they do
things that more "normal-sounding" modifiers (e.g., "which") aren't allowed to do.

for 2 problems that use this sort of modifier, see:


* #59 in the purple OG verbal supplement (in which this sort of modifier is present in the NON underlined section)
* #79 in the same source (in which it's present in the correct answer choice)

--

on the other hand, the same type of modifier can also describe the preceding noun (much like "which").

for instance:
james went for dinner and drinks with mr. easton, a consultant from the west end.
here, "a consultant" describes only "mr. easton", not the whole idea of the preceding clause.

james went for dinner and drinks with mr. easton, an outing that was much more enjoyable than working all day.
here, "an outing" describes the whole previous clause (going for dinner and drinks).

note that, if exactly one of these assignments is sensible and the other is absurd, then the sentence is fine--i.e., there's no "ambiguity".
for instance, in the last sentence above, mr. easton is clearly not "an outing", so the exact meaning of this sentence is uniquely specified.

lunarpower GMAT Instructor

brief explanations:

Question 09
GMATPrep - Default Pack Exam1_Repeat01 - Verbal - SENTENCE CORR ANALYSIS Page - 12

(a)
"and with ..." isn't parallel to anything.
AND sets up parallelism, so there must be something to which "with..." can be parallel (other prepositional phrase, or other
adverbial modifier). there is no such thing.

(b)
correct.
this is a type of modifier with which you should be familiar. (i have no idea what it's called - sorry)
here's another example:
john, his arms flailing in the wind, called out desperately for help.

note that the presence of frozen water SUPPORTS the claim that europa is "far too cold to support life", so it should
be a MODIFIER.
this is done here.
it's inappropriate to place these two things in parallel with AND.

(c)
AND is rhetorically inappropriate (see above).
"considered as" is unidiomatic here.

(d)
this is not a sentence. (the clause before "and" doesn't have a verb; "considered" is a participle, not a verb, here)
"considered as" is unidiomatic here.

(e)
i don't think "considered to be" is wrong, although it's wordier than just "considered..."
this sentence has no verb at all!
the only verb forms present are participles and infinitives, none of which is eligible to be the main verb of the sentence.

Question 09
GMATPrep - Default Pack Exam1_Repeat01 - Verbal - SENTENCE CORR ANALYSIS Page - 13

lunarpower GMAT Instructor

m&m wrote:
Lunar - explanation makes sense except the sentence as is doesn't seem right. If we remove the part between commas (in b) then I would think we
should have an "Are" after the word water. Don't know why gramatically but it just doesn't make sense to say "With XYZ its ABC of water thought" it
should be "are thought" - right???

no.
look, you're right - this construction is going to look really strange if you've never seen it. in fact, i acknowledged exactly this point above - it's a really
weird construction.
here's what i wrote:
Quote:
this is a type of modifier with which you should be familiar. (i have no idea what it's called - sorry)
here's another example:
john, his arms flailing in the wind, called out desperately for help.

remember the following principle, which will greatly simplify your test-taking life:
* if it appears in an officially correct answer, it's correct.
i don't know whether it says so above, but this is an official problem (it's from the gmatprep software). given that fact, any time you spend trying to
question the officially correct answer is 0% productive and 100% wasted.

also, remember the following principle:


it's much, much more important to RECOGNIZE EXAMPLES of grammatical constructions than to explain the actual rules behind those
examples.
really.
think about the way you evaluate your own native language. if you come across an example of sloppy writing in your native language, HOW do you
recognize it as sloppy? do you actually apply a bunch of formal rules to it?
no, you don't - you just realize, "hey, that doesn't look like what i've seen before." it's pure recognition.

Question 09
GMATPrep - Default Pack Exam1_Repeat01 - Verbal - SENTENCE CORR ANALYSIS Page - 14

you should try to get to this point with sentence correction.


when you get to this sort of obscure construction, just remember what it looks like and in what context it's found, so you can recognize similar
examples in the future.

the poster above has been kind enough to provide the label for this structure: it's called an "absolute phrase". (i didn't know this before.)
that is tremendously useful for looking up more examples online, but, if you actually have to use such labels while you're solving sentence correction
problems, you're going to be in some trouble as far as time management is concerned.

https://www.manhattanprep.com/gmat/forums/sc-with-surface-temperatures-estimated-at-minus-230-degrees-t7887.html

RonPurewal
ManhattanGMAT Staff

Re: SC: With surface temperatures estimated at minus 230 degrees

Sat Feb 27, 2010 6:45 am

purduesr wrote:In Choice A,

How come "With 60 square miles of..." isn't parallel to "To support life"

Aren't they prepositional phrase????

Question 09
GMATPrep - Default Pack Exam1_Repeat01 - Verbal - SENTENCE CORR ANALYSIS Page - 15

first of all, no, "to support life" is not a prepositional phrase. it's an infinitive. these are totally different constructions.

it's possible for "to" to be a preposition - e.g., i went to the grocery store - but that's not what this "to" is doing.
to help you tell the difference, note that ALL prepositions must be followed by NOUNS. since "support life" is a verb, not a noun, you
have proof that this "to" is not a preposition.

second, "with... is a MODIFIER, and "to support life" is PART OF THE MAIN CLAUSE. therefore, there are no grounds for parallelism
here.

RonPurewal
ManhattanGMAT Staff

Re: SC: With surface temperatures estimated at minus 230 degrees

Tue Mar 30, 2010 7:51 am

manjeet.singh wrote:Hi Ron..

How to identify such modifiers because while solving this question i ruled out the right answer choice considering it a
run on sentence

the best way to recognize them -- indeed, the only way -- is to look at examples of correct usage of each construction, until the

Question 09
GMATPrep - Default Pack Exam1_Repeat01 - Verbal - SENTENCE CORR ANALYSIS Page - 16

constructions are thoroughly burned into your head.

this particular sort of construction is known as an "absolute phrase".


google it if you want to see tons and tons and tons of examples.

here's one hit i got on google:


http://www.cc.ysu.edu/~tacopela/Writing/Absolutes1.htm

(by the way, this is the one and only reason to care what grammatical constructions are called -- so you can google them and look
at examples. if you're naming grammatical constructions during the actual problems, you're almost certainly not going to finish the
problems on time.)

RonPurewal
ManhattanGMAT Staff

Re: SC: With surface temperatures estimated at minus 230 degrees

Wed May 05, 2010 8:45 am

shatabdo.kal wrote:Ron don't you think an "And" is missing in B ?


life, AND its

no.

this is a type of modifier with which you should be familiar. (if you're the type who likes to name things, this is called an "absolute

Question 09
GMATPrep - Default Pack Exam1_Repeat01 - Verbal - SENTENCE CORR ANALYSIS Page - 17

phrase".)
here's another example:
john, his arms flailing in the wind, called out desperately for help.

--

far more importantly, remember that this is an OFFICIAL problem.


DO NOT QUESTION THE CORRECT ANSWERS TO OFFICIAL PROBLEMS.
it will always be a waste of your time to do this -- the official answers are never incorrect.

in summary:
* the WRONG question to ask is, "isn't that wrong?"
(the answer to this question will, 100% of the time, be "no, it's not wrong.")
* the RIGHT question to ask is, "what sort of construction is this, how is it used, and what can i do to recognize it in the future?"

RonPurewal
ManhattanGMAT Staff

Re: SC: With surface temperatures estimated at minus 230 degrees

Fri Dec 17, 2010 2:37 pm

punzo wrote:

RonPurewal wrote:

Question 09
GMATPrep - Default Pack Exam1_Repeat01 - Verbal - SENTENCE CORR ANALYSIS Page - 18

here's another example:


john, his arms flailing in the wind, called out desperately for help.

Hi Ron,

One question.
In the above example given by you, 'John' is the noun and 'his arm flailing in the wind' is the noun modifier, which is
touching the noun.
But the noun modifier 'its 60 square miles of ......' does not touch the noun 'Jupiter's moon Europa' that is supposed
to be modified.

Thanks.
Punzo

this is correct.

in fact, this sort of modifier ("absolute phrase") usually DOESN'T touch the noun that is actually describing.

check out this problem:


shark-s-tooth-t6220.html
in that correct answer, "each one ready..." modifies "spare teeth", which is the SUBJECT of the preceding clause.
In the sharks jaws, many spare teeth lie in seemingly limitless reserve, each one ready to slide into the appropriate position whenever an
active tooth is lost or worn down.

jnelson0612
ManhattanGMAT Staff

Question 09
GMATPrep - Default Pack Exam1_Repeat01 - Verbal - SENTENCE CORR ANALYSIS Page - 19

Re: SC: With surface temperatures estimated at minus 230 degrees

Sun Jun 05, 2011 9:17 am

jenn.kimbal wrote:Does the "and" in choices a, d and e necessarily create/mandate two independent clauses? Can
you say that these choices are wrong because they create fragments?

Hi Jenn,
Check out Ron's explanation of A:
(a)
"and with ..." isn't parallel to anything.
AND sets up parallelism, so there must be something to which "with..." can be parallel (other prepositional phrase, or other
adverbial modifier). there is no such thing.

The "and" does not mandate independent clauses; as Ron points out, a prepositional phrase or adverbial modifier would create the
necessary parallelism.

RonPurewal
ManhattanGMAT Staff

Re: SC: With surface temperatures estimated at minus 230 degrees

Question 09
GMATPrep - Default Pack Exam1_Repeat01 - Verbal - SENTENCE CORR ANALYSIS Page - 20

Mon Oct 31, 2011 4:55 am

martelena wrote:At first, I thought that the choice A had the parallelism

a parallel structure with "and" wouldn't make sense here, because these two descriptions are not separate and independent. (in the
structure "x and y", the "x" and "y" should be two separate, relatively independent things.)
when you first read the sentence, you should notice that the second description -- the description about a bunch of stuff frozen from
top to bottom -- is elaborating on, or illustrating, the first description (too cold to support life). therefore, for the sentence to make
sense, the second part should be written as a modifier of the first part.

But then I figured out that in that case we wouldn't need comma after "to support life", so I ruled out A mostly on that basis. Was I
at least partially correct?

the gmat doesn't test punctuation, so you shouldn't worry overly much about issues of punctuation.

For example, would be such sentence correct (as awkward as it is)?


"Jupiter's moon Europa has long been with surface temperatures estimated at minus 230 degrees Fahrenheit considered far too
cold to support life and with 60 square miles of water thought to be frozen from top to bottom."

thanks in advance

no, that's still very incorrect.


first, those structures shouldn't be parallel in the first place; see my explanation above.
second, you've turned "considered far too cold..." into a modifier that is modifying ... nothing that makes sense. (if that construction
is a modifier, then the only thing that it can logically modifier is europa.)
third, the sentence is also unidiomatic in a couple of ways, but i will refrain from discussing those because gmac has announced that

Question 09
GMATPrep - Default Pack Exam1_Repeat01 - Verbal - SENTENCE CORR ANALYSIS Page - 21

it doesn't test random idioms anymore.

--

a reminder:
don't try to fix the sentences!

even though this section of the test is called, ironically, "sentence correction", you do not need to be able to fix the sentences; you
only need to be able to select the correct answer choice from the choices given.
trying to fix the sentences is an irrelevant skill set; if you do too much of this, it will distract you from the skill set that you actually
need. moreover, most users' attempts to fix sentences create numerous other errors (like the three new errors mentioned above),
many of which are outside the scope of the gmat.

if you want extra practice using correct constructions, DO NOT rewrite the current sentences; instead, make your
own (much simpler) examples that use those constructions.

tim
ManhattanGMAT Staff

Re: SC: With surface temperatures estimated at minus 230 degrees

Mon Apr 23, 2012 5:28 am

the problem is the word "as", which you left out in your transcription but which the original poster acknowledged as a fatal mistake
almost immediately at the beginning of the thread. please read the posts carefully before you ask a question that has been
answered already..

Question 09
GMATPrep - Default Pack Exam1_Repeat01 - Verbal - SENTENCE CORR ANALYSIS Page - 22

RonPurewal
ManhattanGMAT Staff

Re: SC: With surface temperatures estimated at minus 230 degrees

Mon May 07, 2012 4:03 am

getmydream wrote:how to gauge whether verb form is eligible to be verb or participle?

if you can't tell verbs and participles apart, then that's symptomatic of a much deeper problem: you aren't thinking about what the
words actually mean when you read them.
past-tense verbs and participles have meanings that are essentially complete opposites -- the former suggests that the noun
actually does the action, while the latter suggests an action done to the noun -- so, if you can't reliably tell them apart, then you
must not be thinking much (if at all) about the meaning of the words.

for instance:
robots made (= verb) today's breakfast --> this is something that the robots did.
millions of robots made (= participle) in korea are shipped to the u.s. each year --> this is something done to the robots.

RonPurewal

Question 09
GMATPrep - Default Pack Exam1_Repeat01 - Verbal - SENTENCE CORR ANALYSIS Page - 23

ManhattanGMAT Staff

Re: SC: With surface temperatures estimated at minus 230 degrees

Thu May 17, 2012 10:19 am

getmydream wrote:so is that past participle are used in passive voice constructions only?

essentially, the meaning of the past participle is passive, yes.

tim
ManhattanGMAT Staff

Re: SC: With surface temperatures estimated at minus 230 degrees

Thu Mar 21, 2013 2:32 pm

that looks correct. i suspect you may have further questions on this one but am unable to tell what they may be. feel free to follow
up on this one if you like and we'll be glad to help you further..

RonPurewal
ManhattanGMAT Staff

Question 09
GMATPrep - Default Pack Exam1_Repeat01 - Verbal - SENTENCE CORR ANALYSIS Page - 24

Re: SC: With surface temperatures estimated at minus 230 degrees

Mon Jun 03, 2013 8:26 pm

reservoir11 wrote:I did not select option B thinking that the pronoun "its" (possessive) refers to "Jupiter's"
(possessive noun). Can you please explain if this is the right rule to apply or not.

Thanks
Raj

The sentence is about Europa, so, logically, "its" means Europa's.


What rule?

That's the correct answer, by the way -- so, if your "rule" tells you to eliminate it, then your "rule" is wrong.

noor_dalhousie
Students

Re: SC: With surface temperatures estimated at minus 230 degrees

Tue Nov 12, 2013 12:45 pm

Hi Ron

Question 09
GMATPrep - Default Pack Exam1_Repeat01 - Verbal - SENTENCE CORR ANALYSIS Page - 25

I did not select B because I thought "its 60 square miles of water thought to be frozen from top to bottom" is an independent clause
and should be accompanied by a conjuntion.

Please advise. GMAT next week :(

tim
ManhattanGMAT Staff

Re: SC: With surface temperatures estimated at minus 230 degrees

Sun Nov 24, 2013 3:28 am

Sorry, that's not an independent clause.

RonPurewal
ManhattanGMAT Staff

Re: SC: With surface temperatures estimated at minus 230 degrees

Tue Jun 17, 2014 6:57 pm

lindaliu9273 wrote:Hi instructors,

Question 09
GMATPrep - Default Pack Exam1_Repeat01 - Verbal - SENTENCE CORR ANALYSIS Page - 26

Thank you for explaining the parallel structure here. I learn new things here. But I just want to double check that the
parallel in C, "has long been ..." and "has..." correct if we don't consider whether the two parts are separate
and independent.

There is no such thing as "If we don't consider the meaning..."

Mechanics and intended meaning work together as a team; mechanical structure is determined by intended meaning. (Grammar is
just a vehicle for delivering an intended meaning!)

Without considering sense vs. nonsense, it's impossible to judge""or even to define""whether a construction is "correct".

RonPurewal
ManhattanGMAT Staff

Re: SC: With surface temperatures estimated at minus 230 degrees

Tue Jun 17, 2014 7:00 pm

(Some constructions are incorrect regardless of their meaning, because they just aren't valid English sentence constructions""in
the same way that some computer code doesn't actually run.
For a sentence to be correct, though, it has to express an intended meaning correctly. In the same way, it's impossible to say
whether software "works correctly" unless you actually know what it's supposed to do!)

RonPurewal
ManhattanGMAT Staff

Question 09
GMATPrep - Default Pack Exam1_Repeat01 - Verbal - SENTENCE CORR ANALYSIS Page - 27

Re: SC: With surface temperatures estimated at minus 230 degrees

Tue Jun 17, 2014 6:58 pm

I notice that Ron mentioned "it" in B refers to Europa, but should it usually refer to the closest non-human singler
noun? Here should be "life". I like to check the antecedent when I see a pronoun to see whether it's clear. But I find
this method sometimes makes me confused because pronouns sometimes don't strictly follow the rule.

Thanks a lot!

The distance between a pronoun and a noun is immaterial, as long as the context is clear.

In fact, pronouns are less likely to refer to immediately neighboring nouns.


That job is more often performed by modifiers, for which distance actually is a concern.

RonPurewal
ManhattanGMAT Staff

Re: SC: With surface temperatures estimated at minus 230 degrees

Wed Oct 15, 2014 9:27 am

I don't see anything wrong with "estimated at $10" in the first placeand that's before even taking into account the fact that it's
constructed almost exactly like the 230F sentence.

Question 09
GMATPrep - Default Pack Exam1_Repeat01 - Verbal - SENTENCE CORR ANALYSIS Page - 28

RichaChampion
Students

Ron,

I have one doubt/question in Option C-

Europa has long been considered as far too cold to support life and has

has long been a continuous tense, but after and the tense just has(This is not a continuous tense)-
If we ignore for some time that there are other errors like idiomatic and rhetoric etc.

It is a mandate that and should connect two equal tenses in parallel, or sequence of tenses should work!!

MohitS94
Forum Guests

Re: SC: With surface temperatures estimated at minus 230 degrees

Fri Dec 05, 2014 8:09 am

Richa,

Has been considered is present perfect tense. It works fine with has in the other parallel structure. The sequence of tenses
should be logical, and in this case it is. Option (C) has to be eliminated for other reasons. I had a bit of a problem with tenses too
when I first started with SC, but I found that a sentence works as long as the tenses make sense chronologically (for want of a better

Question 09
GMATPrep - Default Pack Exam1_Repeat01 - Verbal - SENTENCE CORR ANALYSIS Page - 29

word).

Oh, and has been considered is passive, so the other structure should be passive as well. Usually changing the voice mid-
sentence always produces a weird sounding sentence and I have generally been able to eliminate that choice on the basis of
"awkwardness". But the voice of different parallel structures is definitely something to keep in mind.

Ron would definitely do a better job of explaining all this. But for some reason I haven't seen a lot of activity on the forum recently.
Guess we just need to help each other out. :)

RichaChampion
Students

Absolute phrases have this construction-


[Noun] [Participial Phrase] [Main Clause]

I am unable to Identify the absolute phrase in Option B. Ron can you please help me.

RonPurewal
ManhattanGMAT Staff

Re: SC: With surface temperatures estimated at minus 230 degrees

Mon Dec 29, 2014 11:14 am

i don't know any of that terminology, so, unfortunately, i can't help you there. perhaps one of our other moderators could, though

Question 09
GMATPrep - Default Pack Exam1_Repeat01 - Verbal - SENTENCE CORR ANALYSIS Page - 30

i'll ask.

tim
ManhattanGMAT Staff

Re: SC: With surface temperatures estimated at minus 230 degrees

Fri Jan 16, 2015 11:54 pm

I think the fact that none of the moderators know this terminology is your surest indication that you're doing something wrong.
Focusing on obscure terminology is *not* what will help you on this test. Learn how to recognize issues and deal with them,
regardless of what technical terms can be attached to them.

.. https://www.manhattanprep.com/gmat/forums/sc-with-surface-temperatures-estimated-at-minus-230-degrees-t7887-45.html

Question 09
Question 10
Difficulty: 25% (medium/low)
Question Stats: 66% (01:36) correct 34% (00:42) wrong based on 950 sessions
A:7% B:5% C:66% D:9% E:12%

AdamKnewton
Knewton GMAT Instructor
I think what's cool about this question is you don't need to be 100% certain of has/have to be tempted by the right answer. The
modifying clause at the beginning is made clearer, and the main verb is simplified. Stylistically, it's a much better choice.

Here's a hint on how to be sure about whether "media" is singular/plural: Do you see another singular/plural world, in the non-
underlined portion of the sentence, that tells us the number of "media" as used here?
ajit257
Senior Manager
Hi Adam.....you nailed it ....its the word media that led me to choose a wrong ans. I do see expenditures as plural. How do you
suggest i tackle this..thanks
AdamKnewton
Knewton GMAT Instructor

Again, there's another word later on that tells us whether "media" is singular or plural. Once you see this, there's no longer any
question.

AdamKnewton
Knewton GMAT Instructor
Again, there's another word later on that tells us whether "media" is singular or plural. Once you see this, there's no longer any
question.
AdamKnewton
Knewton GMAT Instructor

The clue I'm actually talking about is the pronoun "they" later in the sentence. They is plural, and it's not underlined, and it without
question refers to the "media." Thus, MEDIA IS PLURAL, so if media is the subject, it must be "have."

Whenever you have a confusion over subject/verb, look for a pronoun clue; whenever you have confusion over pronoun/antecedent,
look for a verb clue. It doesn't always work, but sometimes there's extra info dropped for you there!

daagh
Retired Moderator

By focusing only on the number of the word media, a larger issue has been ignored.

C refers to media as plural while D maintains the plurality by referring to the subject as media analyses, Some may debate that
it is the media that can analyse and focus and not the analyses by themselves. This is an aberrant proposition, because in any
survey or study or report, it is the people behind that do the action, although we do mention that the survey reveals, study indicates
and report points out and so on.

But still, can we equate media with media analyses? Logically yes, but not contextually, I suppose. The original text keeps the media
as the subject. So C must be the answer for not distorting the subject.

daagh
Retired Moderato
True- the textual the media has had as a focus' is grammatically wrong because of number disagreement.

However the issue here is whether choice D is correct. If that choice were to be discussed in an independent context, I would say it
is a perfect sentence with a plural subject and a plural verb. But if the topic were a GMAT question, and if there were serious
grammatical errors in choices ABCE, leaving only D as the best of the bads, then we may have to reconcile to D in spite of the
distorted intent. In GMAT can you skip a question? You won't get the next question.

http://www.beatthegmat.com/media-has-or-media-have-t51167.html

EducationAisle GMAT Destroyer!

gmat1011 wrote:
Thought "Media" was singular on the same basis and eliminated C as it has a "have."

This sentence just reinforces why it is so important to read and understand the non-underlined portion of the sentence. The non-underlined portion says:
....they (media) have generally overlooked....

So, this should clearly tell you that media is plural. You didn't even need to know this to solve this question.

https://www.manhattanprep.com/gmat/forums/gmat-prep-t10268.html
RonPurewal
ManhattanGMAT Staff

Re: gmat prep

Fri Jul 02, 2010 3:06 am

FIRST QUESTION:
is this question actually from the official gmat prep software? i'd like to verify that this is the case before we do a ton of analysis on
it.

sudaif wrote:What is wrong with answer choice D?


Is it wrong because it is wordy?

it does seem that "have had as a focus" is a little bit more awkward than "have focused on", but i find it extremely unlikely that such
a subtle difference would be the sole basis for elimination of a choice.

Is "have had as a focus" incorrect?

i don't think that this is incorrect, no. however, occasionally the gmat makes some surprising and strange decisions regarding
idiomatic usage, so i can't say this with 100% certainty until we see some official confirmation, one way or the other.

Can "they" refer to media analyses?

make sure you know that pronoun ambiguity is not a criterion on which you can definitively eliminate. in other words, just because a
pronoun may be ambiguous does not mean that it's necessarily wrong.
for more details, see this post:
post40400.html#p40400

RonPurewal
ManhattanGMAT Staff

Re: gmat prep

Sat Aug 14, 2010 6:39 am

sudaif wrote:well yes, for the most part. but there was still one question outstanding - probably because I didn't
express it clearly enough.
let me give it another shot.
When i asked...can "they" refer to media analyses? I was trying to figure out if it would be okay for the sentence to
read
"media analyses (or they) have generally overlooked the cost of actually administering elections, which includes
facilities, transport, printing, staffing, and technology."
To paraphrase, the meaning of the sentence is not distorted if you say the "media analyses overlooked the cost of...."
or is it?

Thanks M!

yes, "they" could refer to media analyses -- i'm quite sure that could be correct, too. (in fact, in terms of making purely logical sense,
i think that's actually a little bit better.)
--

one thing i didn't mention above, though, because it didn't directly answer any of your questions -- but which is very important:
the best way to choose between (c) and (d) is to use PARALLELISM.

(c)
have focused on...
have overlooked...
perfectly parallel.

(d)
have had as a focus...
have overlooked...
not parallel (or, at least, badly inferior to the parallelism in (c).)

(c) wins.

ChrisB
ManhattanGMAT Staff

Re: gmat prep

Sun Dec 12, 2010 1:47 pm

Hi,
Good questions from the last two posters:

1. Media is a tough one because media historically is the plural form of "medium." In American English, media is understood to be a
collective noun and thus singular. If I had to choose between a split of the two I'd curse the GMAT to the depths of Hades, and then
pick media as a singular collective noun. That is because media in this sentence is used in the context of the news industry and by
convention "news media" is a singular collective noun. That said, I'd then get this problem wrong. In the future, know that media is
plural and fits into the same special category as data in that they are plural forms of singular nouns medium and datum
respectively.

2. Regarding pronoun agreement between "they" and "media analyses," one must understand that the pronoun takes the place of a
noun. Here analyses is the noun that is replaced by they. "Media" is an adjective that makes it clear what type of 'analyses' is
discussed in the sentence. Analyses and they agree in number but analyses does not make sense when substituted in for "they" in
the non-underlined portion of the sentence.

To see this insert analyses where you see they in this sentence: "but they have generally overlooked" becaomes "but media
analyses have generally overlooked"

The analyses can't overlook anything because they aren't something that can take action. "media", on the other hand, is a noun
that can take action.

Overall, I really dislike this question because I don't think the points in #1 are an open and shut case. Perhaps the GMAT is trying to
take a stand and end the use of media as a collective noun :(

Thanks!
Chris
RonPurewal
ManhattanGMAT Staff

Re: gmat prep

Fri Feb 11, 2011 5:48 am

raquel.antonious wrote:Isn't it rare to see Has/Have twice in one sentence?

not really; the repetition of helping verbs is actually quite common, especially in parallel structures.

How do we spot this error since it stands correct in an example like above?

it's not an error, so there isn't really an answer to this question.


what makes you think that it's an error? is there some source that has actually declared that this is an error?

jnelson0612
ManhattanGMAT Staff

Re: gmat prep

Sun Jun 05, 2011 9:33 am


There sure is. "Analyzing campaign expenditures" sounds/look like an adjective, modifying "the media," and "in analyzing campaign
expenditures" is an adverb, modifying WHERE "the media have focused on the costs and ethics of campaign finance".

Even though the -ING Modifier includes a comma, the fact that this warmup is unclear about what it refers to means that including
an "in" helps clarify that the warmup isn't referring to the media, but the clause as a whole.

xyq121573

Forum Guests

Posts: 15

Joined: Fri Nov 09, 2012 4:18 pm

Re: gmat prep

Mon Feb 18, 2013 11:25 am

RonPurewal wrote:yes, "they" could refer to media analyses -- i'm quite sure that could be correct, too. (in fact,
in terms of making purely logical sense, i think that's actually a little bit better.)
SC312

Forum Guests

Re: gmat prep

Sun Jul 20, 2014 4:02 pm

Hi Ron,

I have dismissed the option D) for a different reason.

Here is the option D) :

Media analyses of campaign expenditures have had [ as a focus ] the high costs and low ethics of campaign finance, ...

Therefore the sentence structure stands :

Subject + VERB + <ADVERBIAL MODIFIER modifying "have had > + Object

Now, if we drop the modifier, the sentence "Media analyses of campaign expenditures have had the high costs and low ethics of
campaign finance, .. " doesn't make sense. Therefore, the option looked to be incorrect.

Can you please confirm if the reasoning is correct ?

Thanks

RonPurewal
ManhattanGMAT Staff
Re: gmat prep

Wed Jul 23, 2014 6:17 am

No, not correct.

You can't indiscriminately "drop modifiers", and then object to the meaning of the resulting sentence.

"- If you "drop" a modifier that is enclosed by commas, the resulting sentence should still make sense.

"- If you "drop" a modifier that is NOT enclosed by commas, the resulting sentence will almost never make sense. It will still have
a legitimate grammatical structure (in terms of pure mechanics), but it will almost always become nonsense.

swadha.r.21
Students

Re: gmat prep

Sat Oct 03, 2015 1:53 am

sahilk47 wrote:

swadha.r.21 wrote:Hello all,


RonPurewal
ManhattanGMAT Staff

Re: gmat prep

Sun Oct 04, 2015 7:45 am

that doesn't really make sense, though.


if 'have overlooked' is something that only occurred in a smaller timeframe, within the timeframe of 'have been focusing', then...
what have they been doing for the rest of that time? it seems you're suggesting that, for some part of that time, they have not
overlooked 'Y'in which case the sentence would deceive the reader.

in any case, there's way, way too much thinking going on here.
in context, it should be absolutely clear that the forms of 'focus' and 'overlook' must be parallel, since both are general
observations about the campaign.
not even for a moment should you entertain any choice in which these items are non-parallel. why waste the time?
Question 14
https://gmatclub.com/forum/the-new-image-of-stone-age-people-as-systematic-hunters-of-75120.html

Difficulty: 45% (medium)


Question Stats: 61% (01:46) correct 39% (00:47) wrong based on 911 sessions
A:1% B:2% C:21% D:21% E:55%

http://www.beatthegmat.com/stone-age-t36393.html

lunarpower GMAT Instructor

parallel structure is one reason why (c) is worse than (d) or (e).

to wit, look at the blue parts below. note that "rather than" is a one-part signal - i.e., unlike two-part constructions such as "both ...
and" and "not only ... but also", it lacks a left-hand part indicating the beginning of the first parallel element. therefore, you can
choose to start the first parallel element wherever you want - meaning that you can choose to include or exclude "as" at your
convenience:

(c)
The new image of Stone Age people as systematic hunters of large animals, rather than as mere meat scavengers, has...
this parallelism is acceptable, but there are two undesirable things:
* "hunters OF large animals" isn't truly parallel to "meat scavengers"
* "meat scavengers" is awkward / unclear (you probably won't know this unless you're a native speaker of english and/or a writer)

(d)(e)
The new image of Stone Age people as systematic hunters of large animals, rather than mere scavengers of meat, has...
this is better parallelism (notice that "as" is excluded from the first part this time).
* note the EXACT parallelism between "hunters OF large animals" and "scavengers OF meat".

also, (c) implies, unambiguously (and absurdly), that germany itself "includes" 3 wooden spears.
(3 all the way!)
lunarpower GMAT Instructor

also

by far the easiest way to kill (c) is subject-verb agreement: "includes" (singular) doesn't make sense, because "tools" (the clearly
intended antecedent) is plural.

there is also a VERY subtle difference in meaning here, which is wholly idiomatic.
namely:
if you say "tools that include X", then X is A COMPONENT of the tools. so, for instance, "tools that include a bottle opener" means
that a bottle opener is one of many attachments.
on the other hand, "tools(,) including X" implies that X is ONE OF the tools. so, for instance, "tools(,) including a bottle opener"
means that the bottle opener itself is one of the tools in question.

clearly, S-V agreement is the easier way to go.

lunarpower GMAT Instructor

priyank12 wrote:
if we were given only 2 option to choose from...which one of the following will u choose

1) mere scavengers of meat, has emerged from the examination of tools that include

2) mere scavengers of meat, has emerged from the examination of tools, which include

both are wrong in this context (not grammatically), as both would imply that spears were components of the tools.
i'm not enamored of recommending an incorrect option over another incorrect option, and it's unwise practice for anyone else to do so (so i hope no one
else attempts to answer this dilemma).

--

in a different context, this boils down to the difference between "essential modifiers" and "nonessential modifiers". that's a
difference that is entirely contextual, not grammatical: i.e., it depends entirely on the meaning of what you're saying.

this is a topic that is covered at length on numerous websites. if you want to know more, google the aforementioned terms for more
(and more detailed) information than could possibly be posted here.

lunarpower GMAT Instructor

pnk wrote:
I have a number of your post, wherein you have mentioned that 'comma+verbing' modifies the subject of the clause. If that is correct will it be
appropriate to say 'including' modifies tools. While attempting this question I eliminated E just on that logic.

Pls help[/b]
ah, yes.

important:
"INCLUDING" is an EXCEPTION to the otherwise robust rules for comma+ing modifiers.

when you see "comma + including", you should NOT think of "including" as "__ing".
you should think of it as a random word that's not an "__ing" word; it just happens to be dressed that way. it qualifies the previous NOUN (or
NOUN+modifiers), by giving a PARTIAL list (not a complete list!) of examples.
thanks for pointing this out / calling it to our attention -- we'll be sure to include it in our revised unit on modifiers in the course.

e-GMAT GMAT Instructor

Great catch!!!

In fact Question#120 in OG12 also displays this usage. In non-underlined portion of the sentence, including is used with comma and
it modifies the preceding noun - personality traits.

Regards,

Payal

https://gmatclub.com/forum/the-new-image-of-stone-age-people-as-systematic-hunters-of-75120-20.html

sayantanc2k
Verbal Expert
EXPERT'S POST

warriorguy wrote:
I have a query -

In Option E, including should ideally be used to represent the result of previous clause and/or refer back to the main subject - the new image. But in
this case it is used to modify tools. Can someone help to understand that part?

Also, as per knowledge 'which' in option D cannot jump over a verb and modify tools. Is that correct?
A present participle modifier refers to an entire clause not just to depict the result of the clause. The modifier may modify in some other way as well,
such as depicting how the action in the previous clause is done.

Steffi won Wimbledon, defeating Sabatini.

In option E the present participle (including...) does not depict the result of the previous clause. You may consider it a means to have the new image that
stone age people are systematic hunters.

In option D, there is no verb between "which" and "tools" - "found" is a past participle modifier of "tools". The reason that D is wrong is as follows: in
GMAT it is often awkward (except some exceptions) to have two modifiers one after the other (without a conjunction in between) to refer to the same
noun.

https://www.manhattanprep.com/gmat/forums/the-new-image-of-stone-age-people-as-systematic-hunters-of-t2421.html

Hei

A and B are wrong because "has" should be used instead.


C is wrong because Germany doesn't include the three wooden spears.
Between D and E, I ended up with E since "which" *usually* refers to the preceding noun, which is Germany here. So it has the same
problem in C.
However, I don't think that "including" modifies "emerge", "new image" or "Germany". What is the function of "including" here? It
seems like modifying "tools". Can we really do that??

Thanks in advance
RonPurewal
ManhattanGMAT Staff

Re: The new image of Stone Age people as systematic hunters

Mon Mar 03, 2008 7:04 am

Hei wrote:What is the function of "including" here? It seems like modifying "tools".

i would agree. given that the other answer choices are wrong - for exactly the reasons you've specified (well done!) - i'm in fact
forced to agree.

Hei wrote: Can we really do that??

it would appear so.

the gmat makes its own rules, so it appears we've just learned another one: participial modifiers with -ing, even when they serve as
adjectives, have more freedom than do relative pronouns such as 'which'.

good eye.
RonPurewal
ManhattanGMAT Staff

Re: The new image of Stone Age people as systematic hunters

Fri Aug 08, 2008 6:21 am

Anonymous wrote:1) The comparison rule: The new image of Stone Age people as ______, rather than _______
Why are you not supposed to keep parallel structure here?

you ARE supposed to keep parallel structure. ironically, that parallel structure is another reason why (c) is worse than (d) or (e).

to wit, look at the blue parts below. note that "rather than" is a one-part signal - i.e., unlike two-part constructions such as "both ...
and" and "not only ... but also", it lacks a left-hand part indicating the beginning of the first parallel element. therefore, you can
choose to start the first parallel element wherever you want - meaning that you can choose to include or exclude "as" at your
convenience:

(c)
The new image of Stone Age people as systematic hunters of large animals, rather than as mere meat scavengers, has...
this parallelism is acceptable, but there are two undesirable things:
* "hunters OF large animals" isn't truly parallel to "meat scavengers"
* "meat scavengers" is awkward / unclear (you probably won't know this unless you're a native speaker of english and/or a writer)

(d)(e)
The new image of Stone Age people as systematic hunters of large animals, rather than mere scavengers of meat, has...
this is better parallelism (notice that "as" is excluded from the first part this time).
* note the EXACT parallelism between "hunters OF large animals" and "scavengers OF meat".
2) The modifier rule:
Is the "....tools found in Germany, which includes..." wrong because it makes it is used as a relative pronoun?

yes.
this construction implies, unambiguously (and absurdly), that germany itself "includes" 3 wooden spears.
3 all the way!

RonPurewal
ManhattanGMAT Staff

Re: The new image of Stone Age people as systematic hunters of

Thu Jun 11, 2009 7:07 am

kramacha1979 wrote:I had a hard time choosing between C and E.


Outside the parallelism error, why is the usage of including better than that.. ?

by far the easiest way to kill (c) is subject-verb agreement: "includes" (singular) doesn't make sense, because "tools" (the clearly
intended antecedent) is plural.

there is also a VERY subtle difference in meaning here, which is wholly idiomatic.
namely:
if you say "tools that include X", then X is A COMPONENT of the tools. so, for instance, "tools that include a bottle opener" means
that a bottle opener is one of many attachments.
on the other hand, "tools(,) including X" implies that X is ONE OF the tools. so, for instance, "tools(,) including a bottle opener"
means that the bottle opener itself is one of the tools in question.

clearly, S-V agreement is the easier way to go.

I thought 'that' refers to tools instead of germany...

thanks

it's supposed to.


you know that (c) is the wrong answer, right?
the correct answer to this problem is (e).

RonPurewal
ManhattanGMAT Staff

Re: The new image of Stone Age people as systematic hunters of

Tue Mar 30, 2010 8:06 am

anoo_anand wrote:though not underlined :

I have one query :

is not RATHER THAN --> redundant ?


hmm?

"rather than" is one of the most common constructions in written english.

or is it ...

More ..... Rather Than ... that is redundant ?

thanks

yes, that would be redundant, if those were being paired in the construction.

if "more" were part of another construction, though, that would be fine:


i opted to use my coupon to buy more food rather than more drinks
... this sentence is perfectly fine, since the construction is just "rather than". (the word "more" appears twice, but it's part of the
parallel constructions -- it's not paired with "rather than".)

RonPurewal
ManhattanGMAT Staff

Re: The new image of Stone Age people as systematic hunters of


Tue Oct 05, 2010 5:54 am

allandu wrote:Hi,

I remember that some time if the thing before the ",which" is appositive phrase, which can modify the NOUN before
the phrase. So if the ",which includes" in D change to ",which include" will it be correct?

it would still be inferior, though for reasons that are probably too subtle to be tested on the gmat.
specifically, if you had "tools, which include X", then the implication would be that X is actually a component of each tool. (for
instance, "...tools, which include carbon-fiber handles" --> each of the tools includes a carbon-fiber handle.)
by contrast, "tools, including X" implies that X is one of the tools.

is "including" in E somehow may modify the subject of the sentence?

Thanks

nope -- "including" is an exception to the usual comma -ing rules.


the best way to think about "including" is to consider it a preposition, i.e., don't think about it as a -ing construction at all.

usually, comma + "including" refers to the noun or noun phrase that is located before the comma, as it does in the
correct answer here.

punzo
Students
Re: The new image of Stone Age people as systematic hunters of

Sat Dec 11, 2010 9:33 am

I have a query on the use of 'That' and 'Which'

As I understand, 'which' refers to the noun immediately preceding the comma(,), I am confused on the use of "that" in such a
context.
For example,

as mere meat scavengers, has emerged from examining tools found in Germany, "which" includes

Here, 'which' indicates towards 'Germany'.

as mere meat scavengers, has emerged from examining tools found in Germany "that" includes

Here, what does 'that' refer to. Does, 'That' refer to the preceding noun or to the subject of the clause.

Thanks.
Punzo.

ChrisB
ManhattanGMAT Staff

Re: The new image of Stone Age people as systematic hunters of

Sun Dec 12, 2010 1:57 pm


Hi Punzo,

Good question. In this case, "that" is a relative pronoun that always modifies the noun that immediately precedes it. In this case
then, that is modifying Germany. Thus the use of "that" is incorrect for the same reasons you cited regarding the use of the ",
which."

It is helpful to remember that ", which" , "that" , "who" and "whose" typically signal noun modifiers and when you see these words
you should check to see whether they correctly modify the noun that they touch.

Thanks,
Chris

RonPurewal
ManhattanGMAT Staff

Re: The new image of Stone Age people as systematic hunters of

Thu Oct 06, 2011 7:06 am

parveenjain wrote:Had the "that" being refereed to "tools", shouldn't it be in plural form i.e. "those"?
So in that case, use of "that" is anyways wrong.
Please correct if I am interpreting it wrong?

for this kind of "that", you're wrong -- the plural and singular forms are both "that".
e.g.
Here is a box that once contained sacred relics.
Here are two boxes that once contained sacred relics.
both correct

tim
ManhattanGMAT Staff

Re: The new image of Stone Age people as systematic hunters of

Fri Aug 17, 2012 9:08 am

okay, i think i may see what you're getting at. if i'm interpreting your question correctly, here is the answer:

"X including Y" is equivalent to "X that includes Y" (essential modifier)

"X, including Y" is equivalent to "X, which includes Y" (nonessential modifier)

does that help at all? i'm still not sure what makes you think the OG contradicts with anything though; can you be more specific
about what you feel the contradiction is?

RonPurewal
ManhattanGMAT Staff

Re: The new image of Stone Age people as systematic hunters of


Wed Sep 17, 2014 5:39 am

eggpain24 wrote:and here is what I think

hunters of large animals, rather than mere scavengers of meat

could have the possibility of being construed as hunters of large animals, rather than of mere scavengers of meat

but, sure, it is illogical to a large extent

Nonsense interpretations can always be rejected.

If a nonsense interpretation exists, it does not compromise the correct reading of the sentence. Just ignore it.

More importantly, remember that the first step of SC should always be "Read the original sentence and figure out what it
means".
If you're even inventing interpretations like this one, I'd suggest that you aren't doing a good enough job of step #1. With a strong
grasp of the intended meaning, you shouldn't even think of nonsense interpretations.

RonPurewal
ManhattanGMAT Staff

Re: The new image of Stone Age people as systematic hunters of


Wed Sep 17, 2014 5:40 am

sanketm5 wrote:In the above example, is "rather than as "for the second part necessary, or not needed?

It's absent from the correct answer. The correct answer is not wrong.

RonPurewal
ManhattanGMAT Staff

Re: The new image of Stone Age people as systematic hunters of

Wed Sep 17, 2014 5:41 am

sanketm5 wrote:Would the below sentence be correct?


I just changed the rather than for the second part to rather than as..

The new image of Stone Age people as systematic hunters of large animals, rather than as mere scavengers of meat,
has emerged from the examination from the examination of tools found in Germany, including three wooden spears
that archaeologists believe to be above 400,000 years old.

It's still correct with the extra "as". Both versions are fine.
In general, the extra "as" won't be included unless the sentence is exceptionally difficult to read without it (e.g., if the two
descriptions are both very long).

RonPurewal
ManhattanGMAT Staff

Re: The new image of Stone Age people as systematic hunters of

Wed Jul 01, 2015 5:08 am

GMAC has thus far used 'which' only in 2 ways. i enumerated them here:
usage-of-which-t746-15.html#p104933

"tools found in germany" has neither of those two forms, so i wouldn't take that option. ('tools in germany', on the other hand, does
fit the mold, although that phrase becomes nonsense without 'found'.)

RonPurewal
ManhattanGMAT Staff

Re: The new image of Stone Age people as systematic hunters of

Wed Jul 01, 2015 5:08 am

GMAC has thus far used 'which' only in 2 ways. i enumerated them here:
usage-of-which-t746-15.html#p104933

"tools found in germany" has neither of those two forms, so i wouldn't take that option. ('tools in germany', on the other hand, does
fit the mold, although that phrase becomes nonsense without 'found'.)

RonPurewal
ManhattanGMAT Staff
usage-of-which-t746-15.html#p104933

Re: Usage of "which"

Wed Jul 09, 2014 10:46 am

To date, GMAC has only used "which" to modify two kinds of things:
"- Noun
"- Noun + prep + other noun

That's it.

So, for instance, both of these (beginnings of) sentences work:

New treatments for cancer, which is the leading killer of American adults over 65, ...

New treatments for cancer, which are often too expensive for patients to afford, ...

In this problem, the dollar figure isn't either of these.


"- It's not the first one, since "1992" is in that position.
"- It's not the second one either, since "$xxx in 1992" is not a noun+modifier construction. ("In 1992" describes the entire previous
action, with "rose".)

So, not workable.

RonPurewal
ManhattanGMAT Staff

Re: The new image of Stone Age people as systematic hunters of

Wed Jul 01, 2015 5:19 am

also

THIS DIFFERENCE IS TOO SUBTLE TO BE NECESSARY ON THE EXAM

the word 'including' conveys a VERY specific message:


'okay, i just mentioned a group of things. here are some, but not all, of those things.'

'which include' does NOT convey the same message.


the idea is hard to put into words, but here's my best attempt: when 'which' follows a plural noun, it describes something
about those nouns themselves, NOT about the group that they form.

'...tools, including screwdrivers' means that the GROUP contains screwdrivers.


'...tools, which include screwdrivers' means that THE TOOLS THEMSELVES (NOT the group) contain screwdrivers.

examples:
I went to Home Depot and bought several tools, including screwdrivers.
--> i bought a bunch of tools. some of those tools were screwdrivers.

I went to Home Depot and bought several multi-tools, which include screwdrivers.
--> each multi-tool actually includes screwdrivers. i.e., screwdrivers are components of these multi-tools. (click here if you're
not clear on what a 'multi-tool' is; that meaning is essential to the sentence.)

hopefully, you see what i mean.

again, THIS DIFFERENCE IS TOO SUBTLE TO BE NECESSARY ON THE EXAM. i just thought 'hey, that's interesting' when i
looked at your question, so i decided to write about it. (:

RichaChampion
Students

Re: The new image of Stone Age people as systematic hunters of

Mon Mar 07, 2016 5:41 am

RonPurewal wrote:

allandu wrote:Hi,

I remember that some time if the thing before the ",which" is appositive phrase, which can modify the NOUN
before the phrase. So if the ",which includes" in D change to ",which include" will it be correct?
Re: The new image of Stone Age people as systematic hunters of

Fri Mar 11, 2016 1:26 pm

it modifies the tools -- that's quite clear from context.

RichaC581 wrote:But how to decide that what will it modify noun(germany) or noun phrase( tools found in Germany)

you make this decision the same way you'd make any other such decision (for any modifier that's allowed to do more than one job)
-- with a combination of context + common sense.

RonPurewal
ManhattanGMAT Staff

Re: The new image of Stone Age people as systematic hunters of

Mon Jan 02, 2017 2:29 am

comma + "including" doesn't fall under the category of "comma + __ing" modifiers. you need to think of "including" as though it
didn't end in "__ing".

see here:
https://www.manhattanprep.com/gmat/foru ... ml#p102407

__
MUCH MORE IMPORTANTLY

...you should be able to eliminate choice D IMMEDIATELY!


...tools found in Germany, which includes... (singular verb)

clearly the subject of that verb SHOULD be "tools"... but "tools" is plural.
therefore, immediate elimination.

subject-verb disagreement is an absolutely fundamental, completely black-and-white error. if a choice contains a subject and
verb that don't agree, you shouldn't even look at anything else in that choice.

https://www.manhattanprep.com/gmat/forums/post102407.html#p102407

RonPurewal

ManhattanGMAT Staff

Re: United States Senator Daniel Inouye..

Sun May 04, 2014 12:05 pm

important:
If used to introduce a partial list of something, "INCLUDING" should NOT be considered an "__ing modifier".
In fact, you should think of the word "including", in these contexts, as though it didn't end with "-ing" at all.

comma + "including"...
...elaborates on a NOUN that comes before it (possibly a noun + modifiers);
AND
...is followed by only SOME, NOT ALL, of what that noun encompasses.

E.g.,

In gym class we performed ten different body-weight exercises, including push-ups, pull-ups, and planks.

In gym class we performed ten different exercises from the Presidential fitness test, including push-ups, pull-ups, and
planks.

In gym class we performed ten different body-weight exercises, three of which were push-ups, pull-ups, and planks.

...these three sentences work in the same way.


Question 15
https://gmatclub.com/forum/the-success-of-the-program-to-eradicate-smallpox-has-63049.html

Difficulty: 65% (hard)


Question Stats: 53% (01:58) correct 47% (01:36) wrong based on 2080 sessions
A:34% B:4% C:53% D:7% E:3%

ChrisLele
Magoosh GMAT Instructor
The success of the program to eradicate smallpox has stimulated experts to pursue what they had not previously considered
possible - better control, if not eradication, of the other infections such as measles and yaws.

a) what they had not previously considered possible - better control, if not eradication, of the other infections such
as

'the other' is redundant. 'such infections' implies other infections.

b) what they had not previously considered a possibility - a better control, if not eradication of such infections like

the indefinite article 'a' is not necessary. 'such...like' is not idiomatic. Should be 'such as.'

c) something they had not previously considered possible - better control, if not eradication, of such infections as

Maintains original meaning of sentence, and uses correct construction 'such...as.'

d) something they had previously thought impossible - better prevention, if not eradication, of

By omitting the 'such...as' the meaning of the sentence changes. No the sentence focuses only on measles and yaws. The 'such as'
broadens the scope of infections.

e) what was previously not considered possible, eradication, if not a better control of infections such as

Requires either a colon or a dash, instead of a comma, before eradication. The 'a' is unnecessary.
dianamao
Intern

gmattokyo wrote:
I got stumped by this Q in a mock test and see that the OA hasn't been posted here. It is C. bsd_lover has it right

here is an explanation from mgmat staff in their website (though I don't think I'll get this kind of SC right if given again )

the 'what' construction is problematic in this sentence, because, in general, 'what' conveys a sense of exclusivity / uniqueness. 'what they had not
considered possible' implies that there is only one thing satisfying that description, in contrast to 'something...' which admits the possibility of many
other such things.
....AND
the biggest bugbear in choice a is the word 'the', which creates an image of a definite set of other infections.

Wow, never knew about these rules. Changes the meaning of the sentence quite a bit. Thanks

sayantanc2k
Verbal Expert

apjoshua92 wrote:
C looks like the correct choice as it correctly uses "such as" instead of "like". The use of "the" in A looks awkward. although i don't know the
grammatical reason.
The definite article "the" here seems to refer a group of diseases that has already been mentioned previously - however it has not been. It is often
awkward to use a definite article when an item(s) is introduced for the first instance.

sayantanc2k
Verbal Expert

oishik2910 wrote:
i was confused between A and C
can anyone please clarify that according to A its "the other infections such as " the word such as indicates other infections that may not be related to
small pox
and C removes the word other and infection now seems to compare with only small pox

please clarify

Whether the "other" infections are related to small pox or not is not mentioned in either option A or C - that is not the point here. Moreover there is no
grammatical comparison. The point is that some infections ( measles, yaws) can be controlled by programs similar to that used to eradicate small pox.

The problem with A is mentioned in the posts below:


http://www.beatthegmat.com/smallpox-t25593.html

GMATGuruNY GMAT Instructor

Onell wrote:
Why is past perfect had previously considered correct?

Answer choice C:
The success of the program to eradicate smallpox has stimulated experts to pursue something they had not previously considered possible.

Action in the past:


The success...has stimulated experts to pursue.
The present perfect implies a past action that affects the present. Thus, sometime in the past, the success with smallpox stimulated experts to pursue
better control of other diseases.

Action even further in the past:


...something they had not previously considered possible.
Previously, the experts had not considered better control possible, but then this action stopped: clearly, the experts DID consider better control possible
when they were stimulated to pursue it.

Putting it all together:

The success has stimulated experts to pursue...something they had not previously considered possible.

GMATGuruNY GMAT Instructor

gmat062011 wrote:
- Correct: I never read this book, but I read the other books on the shelf.
- Correct: I never read this book, but I read other books on the shelf, such as "Right Hand, Left Hand" and "The Rise and Fall of the Third Reich."
- Incorrect: I never read this book, but I read the other books on the shelf, such as "Right Hand, Left Hand" and "The Rise and Fall of the Third Reich."
- Minor problem is "what they had not..." vs. "something they had not..." The "what" construction is awfully strong, suggesting that this was THE ONE
THING they hadn't thought possible.
* As an analogy, compare the meanings of "I want to do what I love for a living" and "I want to do something I love for a living." The first suggests that
the speaker has one particular field in mind; the second doesn't.

StaceyKoprince
ManhattanGMAT Staff

I'm still unable to grasp the grammar rule that makes option C better than option A
Is it wrong to have the definitive "the other" while giving examples using "such as"

Basically, yes. When you say "THE other" you imply one of two things: either certain other things (eg books) that you've previously specified (which
we haven't in this case) or ALL other things of that type, which is also not what's going on here.

The implication here is that there are SOME other infections, and here are a few examples.
RonPurewal

ManhattanGMAT Staff

Re: I have a question


Mon Feb 18, 2008 5:13 am

rschunti wrote:In option "C", both words "considered" and "possible" are adjective? Here is the link from marriam
online dictionary
a) Considered>>http://www.merriam-webster.com/dictionary/considered
b) possible>>http://www.merriam-webster.com/dictionary/possible

So can we use two adjectives this way without comma? So I have following questions:-
1) What these both adjectives are modifying. Pls can you give some other example where similar construction is
used?
2) Are they really functionning as adjectives?
2) What are the errors in other options?

ok, this image posting madness has gone on long enough.

let's make a deal: you post the TEXT of the question on this thread, and then we'll post a response. the whole image-posting thing is
supposed to be reserved for problems that need image posting (like math problems with diagrams, which can't be rendered on the
forum).

brian
ManhattanGMAT Staff

Thu Mar 06, 2008 11:35 pm

Thanks for posting the text -- definitely makes it easier and quicker for us to use.
As you asked, in Answer C, considered and possible are not servings as adjectives. Considered is part of the verb phrase "had not considered" where as
possible can be considered as a verb modifier (think adverb.)

Hope that helps.

Thanks.

Brian Lange

RonPurewal
ManhattanGMAT Staff

Re: LIKE vs. such as

Fri Mar 14, 2008 3:37 am

enginpasa1 wrote:I can see what is wrong with answer A. But I am stuck on the differences between such as.
SHouldn't it have a huge play in what is considered right and wrong?

not really sure what you're asking.

if you're referring to the difference between 'such as' and 'like', then of course we should mind that difference. it's very black-and-
white: 'such as' introduces examples, while 'like' refers to actual similarity. (in almost all examples in which the two are conflated,
'such as' is correct, so, if you're in doubt, go with 'such as'.)

if you're referring to the distinction between 'X such as Y' and 'such X as Y', that distinction is largely illusory; both forms are
considered correct, and there is little, if any, difference in usage between them.

RonPurewal
ManhattanGMAT Staff

Mon Apr 28, 2008 4:23 am

Anonymous wrote:hey dear tutors, I finally can't help posting a reply here, though I previous only read your responses and those well
covered all my concerns. My only concern with this one is that, in answer C, 'that' is missed between 'something' and 'they'. I believe
the clause beginning with 'they' is an attributive one, which modifies 'something'. Until GMAT came to my life, I had always believed
that it didn't matter to omit 'that ' or not in this case. But GMAT has seemed to particularly emphasize the correctiveness of having it.
Can you help clarify this point?

actually, no, i can't clarify the point any further; you've done an excellent job of dissecting the issue. the only response i can really give is this:
you have just learned that this construction is ok on the gmat.

think of this like one of those times when you were a kid, and you thought your parents had an ironclad rule X, and then one day you discovered that
they really didn't enforce X all the time. (we all had moments like this, no matter how much of disciplinarians our parents were) from then on, you
(probably) conducted yourself differently because of your newfound knowledge that X wasn't really a rule after all.

same deal here: this is an official problem, so now you know that this construction is ok. remember that. but - you should still look carefully for
ambiguity when you eliminate 'that'. if the gmat writes a problem on which little words such as 'that' are required, it's often because those words get rid
of some sort of ambiguity that would exist if they weren't there.

since the gmat is apparently ok both ways, you can also safely infer that they will NOT give you a problem whose sole determinant is the
presence/absence of 'that' (unless, as just mentioned, there's an ambiguity problem).
RonPurewal
ManhattanGMAT Staff

Wed Jul 30, 2008 5:08 am

thisisvb wrote:
thisisvb wrote:Sorry to bump this post up again, but I had one last Q:

D is incorrect because of 'previous possibility' which is unidiomatic and should rather be 'possibility previously'?
however, in the latter, 'previously' is an adverb and should not modify 'possibility' which is used as a noun. In the former, adj
'previous' correctly modifies the noun 'possibilty'

am I correct? I have my exam in less than a month and am horrible at SC..

Help, anyone??

'previous possibility' isn't unidiomatic; it just means something other than what you're actually trying to say.

namely, previous possibility would mean that it WAS a possibility in the past, but it isn't a possibility anymore. ("all the previous possibilities have
vanished.")

not previously considered a possibility means that people once didn't know that it was a possibility, but now they do realize that it is (currently) a
possibility.
shobujgmat

Re: The success of the program to eradicate smallpox

Tue May 05, 2009 2:19 am

Well:
1st: if we correct the sescond part of the setence of choice A:

what they had not previously considered possible - better control, if not eradication, of other infections such as

is A will be correct

And isn't chioce C- change the meaning of the sentence.


because from your explanation:"what they had not..." vs. "something they had not..."The "what" construction is awfully
strong, suggesting that this was THE ONE THING they hadn't thought possible.

and why the secrifice the original meaning of the sentence?

pls shed some light on this topic


RonPurewal
ManhattanGMAT Staff

Re: The success of the program to eradicate smallpox

Wed May 06, 2009 3:16 pm

what they had not previously considered possible - better control, if not eradication, of other infections such as

is A will be correct

still inferior to "something they had..."

also, "other" is unnecessary and therefore wordy, since the other infections are mentioned by name.
if no specific infections were mentioned, then we'd need the word "other" to make it clear that we're talking about infections other
than smallpox.

and why the secrifice the original meaning of the sentence?

the original meaning is absurd. it is clearly impossible that there is only ONE thing that the researchers didn't consider possible.

in cases in which the original is absurd, outlandish, or illogical, you have the license - in fact, the duty - to change its meaning.
RonPurewal
ManhattanGMAT Staff

Re: The success of the program to eradicate smallpox

Wed Jul 01, 2009 5:01 am

hmgmat wrote:Also, I read somewhere that "what" can also means "things that". Is it a GMAT rule that "what" has to
be interpreted as "the thing that"?

Thanks in advance.

ah, no, i wasn't meaning to differentiate between singular and plural. "what..." can certainly refer to more than one thing, or to a
plural noun.

What was most noticeable about the cathedral was the gargoyles on top of the roof.
this is legitimate.

the distinction i was trying to emphasize was that between an exhaustive treatment (implied by "what") and the giving of examples
(implied by constructions such as "something").
"what" can refer to either a singular or a plural, although there may be other clues elsewhere in the sentence that require one or the
other.

nehajadoo
Students

Re: The success of the program to eradicate smallpox

Tue Oct 26, 2010 3:12 pm

I was hesitant to choose C and chose C in the end Only because i saw some other correct answer which had " such X as.." instead of
"such as x"

i honestly don't know why "such infections as" in C is correct


shouldn't it be "infections such as"?

thanks!

tim
ManhattanGMAT Staff

Re: The success of the program to eradicate smallpox

Mon Nov 08, 2010 5:11 pm

either construction is acceptable. A, of course, is wrong for other reasons..

RonPurewal
ManhattanGMAT Staff

Re: The success of the program to eradicate smallpox


Fri Aug 12, 2011 4:59 am

style.rohit007 wrote:Hi Ron ,

Can you please clarify , why is "past perfect" correct here?


Also , is this rule also not a "absolute rule" , as is the case with passive/active rule?
There is a similar problem in OG 12(q.50) but can't post here.
Can u plz clarify , in which cases this type of construction (containing past perfect tense eventhough simple past is
not used) is correct?
Response awaited.

Thanks.

you don't need the simple past; you just need an explicit past time reference.
that reference *can* be a simple past action, but it can also be a specified occurrence or past time. in the context of this problem,
the "past time reference" is the time at which the smallpox eradication program succeeded. i.e., before that point people had not
thought xxxxxxxx was possible, but only up to that point.

RonPurewal
ManhattanGMAT Staff

Re: The success of the program to eradicate smallpox


Tue Jul 03, 2012 6:01 am

morymory_1983 wrote:Hi Ron

If THE other + NOUN + SUCH AS... is not okey...


will

THE otherS + SUCH AS ...be okey?

in general, you shouldn't have both of these -- i.e., you shouldn't have "the" + noun + "such as", regardless of what else is around
it.
"the" implies a definite, specified item or set of items, while "such as" implies items that are unspecified but similar to whatever "x"
is. so, basically, the ideas of "the" and "such as" are contradictory.

ex:
you should do the exercises in the book.
--> it wouldn't make sense to have "such as" here, because the book will actually contain a specific group of exercises.

you should do exercises such as those in the book.


--> this means that you can basically do [i]any exercises of the same kind, regardless of whether they actually appear in the book.
since this isn't a definite set of exercises, it wouldn't make sense to use "the" here.

jlucero
ManhattanGMAT Staff
Re: The success of the program to eradicate smallpox

Thu Sep 06, 2012 5:28 pm

lxj2009 wrote:Hi, Manhattan staff, regarding D I still have 2 questions:

(1) possible and possiblity


Your previous reply is pasted below. What I think is that the scientists don't consider it a previous possiblity, as
written in D, means the scientists didn't think it possible in the past, but think it possible now. therefore for this part,
A and D have equal meanings. What's wrong with my opinion here?

previous possibility would mean that it WAS a possibility in the past, but it isn't a possibility anymore. ("all the
previous possibilities have vanished.")

Focus less on the possible(adj)/possibility(noun) split:


Anything is possible.
Anything is a possibility.

And more on the previous(adj)/previously(adv) split:


This was a previous possibility. (previous modifies possibility)
This was previously possible. (previously modifies was)

Now look at options C and D:


(C) (It has stimulated experts to pursue something) that they had not previously considered possible.
(D) (It has stimulated experts to pursue something) that had not been considered a previous possibility.

(C) Before, we had not considered this possible. CORRECT


(D) We had not considered this a (previous possibility). You didn't consider this as a previous possibility? Did you consider this a
current possibility? Or a future possibility? Or something else?

lxj2009 wrote:(2) is there any thing wrong with "perhaps eradication" in D? I think both "if not eradication" and "perhaps
eradication" are right in meaning according to the sentence's context. It makes sense that scientists want to eradicate other
similar insects after they have eradicated one kind. What do you think of it?

Yes. Perhaps is an adverb which can't modify "eradication" in this sentence. I can't say "I want perhaps eradication". "If not" works
as a conjunction to connect two ideas: better control, if not eradication.

jlucero
ManhattanGMAT Staff

Re: Re:

Thu Sep 06, 2012 5:31 pm

gmatalongthewatchtower wrote:

brian wrote:Thanks for posting the text -- definitely makes it easier and quicker for us to use.

As you asked, in Answer C, considered and possible are not servings as adjectives. Considered is part of the
verb phrase "had not considered" where as possible can be considered as a verb modifier (think adverb.)

Hope that helps.

Thanks.
no, that's not what i meant. what i meant is that "what xxxx" is exclusive -- i.e., it doesn't leave out any of the xxxxx.
e.g.
on the table is what your friend left at my house yesterday.
--> this means that everything your friend left behind is on the table. there could be only 1 thing, or there could be 1,000,000
things.
but...
on the table is something your friend left at my house yesterday.
--> your friend may have left other things besides the one that's on the table.

jlucero
ManhattanGMAT Staff

Re: The success of the program to eradicate smallpox

Sat Sep 14, 2013 5:32 pm

harishmullapudi wrote:option C reads "something they had not previously considered possible -- better
control, if not eradication, of such infections as"

Isn't the usage "had not previously considered" wrong here? Because it uses Past Perfect that is unnecessary here?

Please help...
(1) If C is the correct answer, don't waste your time wondering if anything in C is ok. It is. GMAC is judge and jury when it comes to
making these rules. Our goal is to make rules/generalizations that help us to learn how to correctly decide on GMAC's questions.

(2) I had not considered is a valid expression, because you are implying that you now do consider this. "I had not considered muffins
a breakfast food" implies that I now have considered them a breakfast food. So I had not considered something, before I decided to
consider it.

RonPurewal

ManhattanGMAT Staff

Re: The success of the program to eradicate smallpox

Tue Jun 17, 2014 7:12 pm

rustom.hakimiyan wrote:I'm still a little confused about the the previous/previously split. How can you tell that
previously modifies was but previous modifies possibility?

Thanks!

"Previous" is an adjective; "previously" is an adverb.


subhojyoti.it

Course Students

Re:

Thu Feb 19, 2015 6:01 am

RonPurewal wrote:There are 2 problems with A.


- Major problem is "THE other infections." THE is too definitive here, carrying the connotation of "every single one of the other
infections."
* THE is also incompatible with "such as":
- Correct: I never read this book, but I read the other books on the shelf.
- Correct: I never read this book, but I read other books on the shelf, such as "Right Hand, Left Hand" and "The Rise and Fall of the
Third Reich."
- Incorrect: I never read this book, but I read the other books on the shelf, such as "Right Hand, Left Hand" and "The Rise and Fall
of the Third Reich."
- Minor problem is "what they had not..." vs. "something they had not..." The "what" construction is awfully strong, suggesting
that this was THE ONE THING they hadn't thought possible.
* As an analogy, compare the meanings of "I want to do what I love for a living" and "I want to do something I love for a living."
The first suggests that the speaker has one particular field in mind; the second doesn't.

Ron,

In many official questions, I have seen the usage of what. Besides framing a question using What, what are the other ways we can
use?
As you have said in this question WHAT raises a very strong implication of the possibilities.
Thanks a lot.

RonPurewal
ManhattanGMAT Staff

Re: Re:

Sat Feb 21, 2015 7:25 pm

subhojyoti.it wrote:In many official questions, I have seen the usage of what. Besides framing a question using
What, what are the other ways we can use?

while this question is reasonable, it's the type of question that's impossible for a human being to answer; our brains don't store
information in this sort of way. ("quick! name all the nouns you know that start with 'd' -- same problem.)

so, here are some examples, which may not encompass the full diversity of ways in which "what" can be used:

The witness told the officer what he saw at the crime scene.
--> the witness told the officer everything he knew.

The witness told the officer things he saw at the crime scene.
--> the witness told the officer only some of the things he knew.

You should take what is in the box.


--> you should take all of it.

You should take something that's in the box.


--> you should take one item.
these examples should be enough to give you the basic idea.

RonPurewal
ManhattanGMAT Staff

Re: Re:

Sun Apr 19, 2015 3:28 am

yes--as long as you read all of the other books on the shelf, including those two.

in this sentence, "including x and y" would serve only to emphasize those two books (for whatever reason), since it contributes
nothing to the meaning.

inc.manni

Forum Guests

Re: The success of the program to eradicate smallpox

Sat Jun 27, 2015 3:41 am

The Use of Past Perfect "had not previously considered" should require another past action - probably stimulated - rather than has
stimulated (present perfect)
I have seen certain answer choices explained as wrong just on the basis of using past perfect along with present perfect rather than
simple past.

Yes i do agree that correct choice is correct - May be the correct choice is best among the other choices and may not be ideal.

Would appreciate your reply.

RonPurewal

ManhattanGMAT Staff

Re: The success of the program to eradicate smallpox

Wed Jul 01, 2015 5:00 am

there's no need to try to make 'rules' for this sort of thing; a basic conceptual understanding of these tenses (= a really, really,
REALLY basic understanding) is enough to let you see why that combination is unlikely (NOT impossible).

* 'had __ed' describes the situation at some specific past point. i.e., it occurs within a past timeframe.
(the timeframe of the sentence is NOT the timeframe in which 'had __ed' actually occurred. the point is that 'had __ed' is something
already completed or manifested by that point.)

* "has/have __ed" describes the situation at present.


analogously, the action of "has/have __ed" should be something that has already been completed, or has already
occurred/manifested, and that thus affects the present situation in some way.

RonPurewal
ManhattanGMAT Staff
Re: The success of the program to eradicate smallpox

Wed Jul 01, 2015 5:01 am

...so, if a sentence contains both of these, it must contain 2 distinct timeframesone past, to accommodate 'had __ed',
and one present, to accommodate 'has/have __ed'.

this is certainly not impossible. in fact, it can happen in perfectly normal sentencessometimes even fairly short ones:
I have been thinking about what Cindy told me she had seen.

note how this works:


I have been thinking = recently (not right now)... but the point is that this 'thinking' affects my current situation and/or
thought processes. this is the present timeframe.
Cindy told me... = there's your past timeframe...
...she had seen = this observation, which cindy had made previously, had some impact upon cindy when she told me
whatever she told me.

obviously you will not have to think about that much stuff on this exam. in fact, it's much more likely that you won't have to think
about ANY of it.
(verb-tense splits are VERY often just distractions; they are quite frequently paired with issues that are MUCH easier to
resolve.
if a verb-tense split is small or subtle, it will ALWAYS be paired with some easier means of elimination.)

on the other hand, if you actually do understand how each individual piece of that sentence works, then your understanding of
these tenses should be more than adequate for the exam.

RonPurewal
ManhattanGMAT Staff
Re: The success of the program to eradicate smallpox

Wed Jul 01, 2015 5:03 am

incidentally, if you want to improve your intuition about verb tenses in a way that's enjoyable, try reading some fiction.

in fiction, tenses tend to be blended and mixed, MUCH more than in nonfiction. (fiction contains lots and lots of sentences like the
one i made up above. nonfiction, while interesting, is much more likely to use the same couple of tenses over and over and over.)
Question 16
https://gmatclub.com/forum/currently-26-billion-barrels-a-year-world-consumption-of-99973.html

Difficulty: 5% (low)
Question Stats: 66% (01:14) correct 34% (00:16) wrong based on 901 sessions
A:66% B:7% C:5% D:4% E:18%

RonPurewal
ManhattanGMAT Staff

Re: SC: Currently 26 billion barrels a year...

Thu Apr 16, 2009 3:15 am

sinhavis wrote:
Source is GMATPrep practice test.

OA is A

I chose E thinking "Currently 26 billion barrels a year" modifies oil.

nope.

oil would be measured in barrels, not barrels per year. remember how literally accurate you have to be with these things.

analogy:
"50 degrees" would refer to a temperature.
"2 degrees per century" would refer to a rise/drop/change in temperature (possibly due to an encroaching ice age, etc.)
(a) is the only choice beginning with a noun that's suitable to this opening modifier, so, (a) it is.

RonPurewal

ManhattanGMAT Staff

Re: SC: Currently 26 billion barrels a year...

Sun Sep 20, 2009 3:12 am

goelmohit2002 wrote:Can someone please tell what is the problem with D....

why can't opening modifier modify the rise....

IMO the rise can be 26 barrels a year

it's unidiomatic to say "the rise in X is SPECIFIC NUMBER". you just can't do that.

(you can say that the rise is ADJ. for instance, you can say the rise has been extremely rapid.)

salman30
Course Students

Re: SC: Currently 26 billion barrels a year...

Wed Oct 20, 2010 3:20 pm

kernnel7799 wrote:Hi,
Can anyone help to explain why (B) is wrong?
Thanks!

B) the world consumption is consuming oil at an increasing rate of

There is an issue of meaning here. How world consumption can consume oil ? World consumption of oil should be correct.

mschwrtz

ManhattanGMAT Staff

Re: SC: Currently 26 billion barrels a year...

Wed Oct 27, 2010 6:41 pm

Yes, B is redundant.

thx
danielpatinkin

Re: SC: Currently 26 billion barrels a year...

Thu Dec 23, 2010 1:51 pm

Answer choice E is below:

E) "oil is consumed by the world at an increasing rate of..."

Regardless of the preceding modifier, this is not a strong answer choice for 2 reasons:

1. It is in passive voice. The subject "oil" is being acted upon by the object "the world". The GMAT prefers active voice, so "the
world is consuming oil..." would be a better structure.

2. "Increasing" is a misplaced modifier and creates an inappropriate meaning. What is increasing is the consumption of oil,
not the rate itself. If it were an "increasing rate", then, for example, one year the rate would be 2%, the next year the rate
might be 2.3%, then the next year the rate might be 2.7%, and so on.

I hope that helps!

- Dan P

RonPurewal

ManhattanGMAT Staff
Re: SC: Currently 26 billion barrels a year...

Sun Jan 16, 2011 2:18 am

manassingh wrote:Initially i picked E as the answer choice. But looks like i was wrong. I would like to confirm my
understanding.

Why is E wrong ?
Currently 26 billion barrels a year, Oil is

Currently 26 billion barrels a year This is a modifier and is supposed to modify the first word/non appearing after
comma. In this case, it is oil

Here is modifier does not tell much how are Currently 26 billion barrels a year and oil] related, hence this is NOT A
CORRECT choice.

Why is A Correct ?
Currently 26 billion barrels a year, word consumption of oil...

Here there is a clear relationship between the modifier(Currently 26 billion barrels a year) and word consumption of
oil

Hence A is the correct choice.

Is this a fair assesement ?

it's a fair assessment, yes.


these points were also discussed earlier in this thread:
post24909.html#p24909

jlucero

ManhattanGMAT Staff

Re: SC: Currently 26 billion barrels a year...

Fri Nov 02, 2012 4:04 pm

aditi.bhargava1 wrote:A question on the phrase 'increasing rate'. Is this used correctly in the answer choices? The
correct answer choice (A) does not use the word 'increasing' when talking about the rate. Thanks!

A red herring issue. The problem is that we need to say oil consumption is rising/increasing. Not the world is consuming oil at a
rising/increasing rate. There are times when the words can't be used interchangeably, but not here

RonPurewal

ManhattanGMAT Staff

Re: SC: Currently 26 billion barrels a year...


Wed Sep 11, 2013 2:06 am

The GMAT doesn't actually "prefer" the active voice. That's a misconception that we've since removed from our materials. (Note that
the post you're quoting is three years old.)

It's true that far more sentences appear in the active voice than in the passive.
However, when there is a split between active and passive constructions in an official problem -- which is the only time you
should ever care about this issue at all -- it's exactly 50/50. Exactly.

In OG12, there are 34 splits between active and passive. In 17 of them, the correct answer contains the active construction. In the
other 17, it has the passive.

thanghnvn

Forum Guests

Posts: 705

Joined: Wed Jan 14, 2009 9:09 pm

Re: SC: Currently 26 billion barrels a year...


Mon Jan 05, 2015 4:14 am

OA is A

in choice A, "currently" is an adverb. this adverb should modify the verb, adjective or another adverb. where is the head word of
"currently"? "currently " refers to what

pls, explain

RonPurewal
ManhattanGMAT Staff

Re: SC: Currently 26 billion barrels a year...

Thu Jan 08, 2015 10:13 am

you can understand this sort of thing as a transformed version of a sentence with "is"/"are".

e.g.,

John is currently a high-school teacher.

Currently a high-school teacher, John is much less stressed than he was in his previous job as an investment banker.

RichaChampion
Students
Re: SC: Currently 26 billion barrels a year...

Mon Nov 28, 2016 5:23 am

"increasing rate of" Isn't this redundant or perhaps Unidiomatic and can be treated as split for elimination here in this question

RonPurewal
ManhattanGMAT Staff

Re: SC: Currently 26 billion barrels a year...

Sun Dec 18, 2016 3:33 pm

no. that's the WRONG meaning for THIS sentence, but, it's definitely possible for a rate to be "increasing".
for example, a stock investment can have an "increasing rate of return" if it yields greater % returns from one year to the next.

THIS sentence is talking about a rate of increase -- NOT a rate that's actually increasing -- so, that phrasing is wrong here because
it's inaccurate, not because it's "redundant".

__

__by the way, your signature says you already have a 740 on the GMAT and you're still studying for a re-take. you almost certainly
shouldn't do that; even if your were to get a higher score on a subsequent administration, the top schools would still most likely
judge you negatively, for having misaligned priorities.
you may want to post a question in our "ask an admissions consultant" thread for more on that.
Question 18

Question 18
https://gmatclub.com/forum/as-the-former-chair-of-the-planning-board-for-18-consecutive-69993.html

Difficulty: 45% (medium)


Question Stats: 50% (01:29) correct 50% (00:31) wrong based on 1485 sessions
A:26% B:12% C:10% D:1% E:50%

icandy
VP

bigfernhead wrote:
I actually don't think former is redundant. Former is an adjective here.

Former means that the person is no longer an active chair member.

Without the former, (As the chair) - it gives the meaning that the person is still currently part of the board.

Where did this question come from? argh.

leonidas wrote:
IMO (E),
The logical meaning of the sentence talks about the past event. "chair of the planning board for 18 consecutive years and a board member for 28
years". "Former" here sounds redundant. What is OA?

Very diff Q.

@BFH,

Why would some one attend meetings as former chair ?


E is my answer
bigfernhead
Retired Moderator

I don't think it's saying that a former chair is currently attending meetings.

This is my interpretation:
When the former is used, the meaning of the sentence is exact:
- The action has taken in the past, and the member is no longer part of the board.

When the former is not used, it gives the 2 meanings, either that:
1. The action has taken in the past, and the member is no longer part of the board nor attends the meetings.
OR
2. the action has taken in the past, BUT can continue into the future, and member is still part of the board.

I don't agree that the intent of the sentence was to infer #2 above, therefore, that's why I thought former was necessary and not
redundant.

redbeanaddict
Manager

E. Because former and for 28 years is redundant.

TommyWallach
Manhattan GMAT Instructor
Hey All,

I got asked to take this one on by PM, so here I am! Yay!

As the former chair of the planning board for 18 consecutive years and a board member for 28 years, Joan Philkill attended more
than 400 meetings and reviewed more than 700 rezoning applications.

A. As the former
PROBLEM: The "as" part is correct (See answer choice E), but you can't be a former chair for 18 years. Also, she didn't do these
things AS the FORMER chair. She did them AS the chair, and NOW she is the former chair.

B. The former
PROBLEM: Technically, this could be called okay, because we have a really long noun modifier ("the former chair...") modifying
another noun, "Joan Philkill." However, answer choice A simply makes more sense, considering the definition of "As" given above.

C. Former
PROBLEM: We need the article "the" here. We use it to refer to a person or thing that is unique. Obviously we're talking about a
unique chair of the board here (Joan).

D. She was
PROBLEM: Now, we end up with two independent clauses separated by a comma. This isn't allowed.

E. As the
ANSWER: The preposition "as" can be used to to mean "during the time of being (the thing specified)." For example, I could say "As
a child, I was often sick." In this sentence, we're told that "During the time of being the former chair...Joan attended more than 400
meetings..."

Hope that helps!


-t

ChrisLele
Magoosh GMAT Instructor

While Joan P. was the former head, the way the sentence reads makes it sound like she was the former head for 18 years. During this time she served as
the head of the committee not the former head. Therefore, you can get rid of A, B, and C. D represents a comma splice so (E) it is.
_________________
Christopher Lele

https://www.manhattanprep.com/gmat/blog/2012/04/20/meaning-is-mean-a-gmatprep-sentence-correction-problem/

Meaning is Mean! A GMATPrep Sentence Correction Problem


by Stacey Koprince Apr 20, 2012

Weve talked a lot about meaning in sentence correction recently and Ive got another problem along that same theme for you. The problem I chose
comes from the new GMATPrep 2.0 (warning: you may not want to read the explanation until after youve used the new software yourself, just in case
you see the same problem!). This one actually did also show up in the old version of GMATPrep, but I saw it years ago and forgot about it. When I saw
it during my 2.0 test last week, I had the same reaction that I did when I first saw the problem about 5 years ago: I cant believe they actually did that!

Heres the problem. Set your timer for 1 minute 15 seconds and go for it!

* As the former chair of the planning board for 18 consecutive years and a board member for 28 years, Joan Philkill attended more than 400 meetings
and reviewed more than 700 rezoning applications.
(A) As the former
(B) The former
(C) Former
(D) She was
(E) As the

Short underline should be easy right? I received this question as #14 on my test and I got the first 13 questions right. That should give you an idea of
the difficulty level. : )

My first reaction to the original sentence was: hmm, that sounds fine and I dont see any glaring grammar issues either. My next step, whenever I cant
find anything wrong with the original, is to compare the remaining answers. In this case, the comparison appears easy, because each answer only
contains a few words but such short answers often mean that the issue tested is going to be an obscure one. The shorter the underline, the tougher it is
because you either know the one or two issues being tested, or you dont. While longer underlines take more time to read, they also give us more
opportunities to narrow down the answers.

I noticed that the original sentence started with as, so I looked to see whether the other answers changed this in any way (one thing I thought was that
they might try to use like). I quickly realized that they werent focused so much on what is the right opening word for the subordinate clause but rather is
this the main clause or a subordinate clause? (Main clause = main sentence; subordinate clause = extra information or modifier, not the main sentence.)

From my read-through of the original, I was already aware that the main clause is the part beginning Joan Philkill attended As such, the stuff before the
comma needs to be a subordinate clause. But I also noticed something else while examining these answers, something that hadnt occurred to me when I
read the original sentence: the last two answer choices dropped the word former.

Thats interesting. They dropped the word and didnt replace it with any synonym or anything else that would have the same meaning. Does that mean
that answers D and E must both be wrong? The alternative would be that something was wrong with the original meaning it was either illogical or
ambiguous. And this line of thinking is what led me to realize that some kind of meaning issue is being tested and Id better figure out whats going on!

What are the two main options? Answer E is basically the same as A, except it has dropped the word former, so lets compare just those two:

As the former chair of the board, Joan attended lots of meetings.

As the chair of the board, Joan attended lots of meetings.


Whats the difference in meaning between those two sentences? Lets try a simpler example.

As the former president, I ran the country.

As the president, I ran the country.

When I ran the country, was I the president or the former president? Ah, I see. The above two sentences are the equivalent of:

While I was the former president, I ran the country.

While I was the president, I ran the country.

While actually doing the action, my title was president. It was only afterwards that my title changed to former president the person who used to be
president. I am the former president now; I was the president while I was running the country. It wouldnt make sense to say I was the former president
while I was running the country.

The same is true for Joan. She is the former chair now she used to be the chair in the past. But it wouldnt make sense to say that she was the former
chair of the planning board for 18 years, as answer A indicates. That would mean that she once was the chair, but she quit 18 years ago, and for the past
18 years, she has not been the chair of the planning board. Further, it would mean that she attended all those meetings in the 18 years after she was no
longer the chair she attended the meetings while her title was the former chair.

Heres the timeline of Joans titles:

board member: 28 years

chair of board: 18 years (presumably included in the 28 years above, though we dont know exactly how they overlap)

former chair of board: dont know how long, but occurred after; she could still have been a regular board member during this time or she could have left
the board entirely

Wow. So answers A, B, and C are all wrong because its illogical to say that she attended all those meetings while she was the former chair for 18 years.
Alternatively, B and C drop the word as at the front, so we can also uncover another meaning issue here.
Former heavy metal rocker, Joan Jett was elected mayor of her hometown.

The modifier at the beginning is now just extra info about this person, but it doesnt have anything to do with the action in the main clause. The original
sentence, however, indicated that she attended these meetings while she was in office, so weve lost that meaning.

That leaves us with D and E. Whats the difference between these two? Ah, now were getting back to the subordinate clause / main clause issue.

(D) She was chair and a board member, Joan attended more than

(E) As the chair and a board member, Joan attended more than

Answer D gives us two main clauses connected by a comma, otherwise known as a run-on sentence. I walked to the store, I twisted my ankle. Thats not
an allowed structure.

Answer E gives us a subordinate clause followed by the main clause. As I walked to the store, I twisted my ankle. Thats an allowed structure and its
the only answer left.

The correct answer is E.

Key Takeaways for Meaning and Short Underlines

(1)Short underlines dont give us many clues, and any grammar rules tested will often be on the obscure side. If youre not sure what to do
grammatically, make sure youre examining meaning as well.

(2) Meaning issues come down to two main categories: logic and ambiguity. If something is illogical, dont pick that choice. Outright grammar errors
and illogical meaning are definitely wrong. If something is ambiguous, on the other hand, make a note but leave it until youve checked the remaining
answers. Ambiguity is more like wordiness in this way its probably wrong, but check the other answers first to make sure.

(3) When in doubt on short underlines, guess even more aggressively than usual. The lack of many differences means that, if we dont know, we just
dont know. Dont keep going back and forth between answers pick and move on.

* GMATPrep questions courtesy of the Graduate Management Admissions Council. Usage of this question does not imply endorsement by GMAC.
https://www.manhattanprep.com/gmat/forums/as-the-former-chair-of-the-planning-board-for-18-consecutive-t842.html

Well correct answer is E, as says the GMATprep, but what are the diferences between A) and E). Moreover A) includes former and
seems to be correct, so if E) does not include it, why is E) better than A).

Am I missing somethig for being a non native?

I also would like to ask the experts: In this verval part I thought I did it really good, but at the end I finished with a 34. I got wrong
questions number 7,8,12,14,17,18,22,24,25,31,34,37,38,39,41

(At the end I was obviously runnig out of time so I had to more or less made a guess. I read through the questions really fast).

I got the first 6 right, which I think it should have boosted me to a high level, and actually the questions have been quite hard during
the whole exam (that is why I runned out of time). And then I havent fail many questions in a row, actually my errors are spread
trough all the exam, getting 3 correct answer and 2 wrong, or similar that I thought it will keep improving my performance smoothly.
So when I saw the 34 I was very dissapointed, I thought I was going to get arround a 38 or so.

What do you think? Since the GMATprep is the most reliable exam, this result has lower my confidence, because Ive been
preparing verbal quite hard and since I felt I did this one pretty good now I dont really know what to expect.

I just one to add one more thing. In my opinion, MGMAT tests for the RC part are considerably easier than the real GMAT ones, which
by the way are sometimes quite short but very complicated (at least for non native). Actually in MGMAT is the part I do better within
verbal, and in the GMATprep is the part I do worst. I have noticed also that MGMAT test are easier but the questions might be trickier
(the 700-800), while the GMATprep tests are by far more complicated but the questions are easier.

The examples for SC and CR are really well done in MGMAT tests. And for the quant part, I thing the problems are harder in MGMAT
but very similiar, so they are a great training for the real test.
Thanks

dbernst
ManhattanGMAT Staff

Tue Aug 07, 2007 9:49 am

Luci, you are missing a nuance in the meaning of the sentence that is difficult even for a native speaker of English to identify. An individual would not
be the former chair of a planning board for 18 consecutive years; instead, the sentence is attempting to say that Joan was actully the chair of the planning
board for 18 consecutive years and that she attended these meetings during those 18 years. Thus, choices A, B, and C and be eliminated. As D is a run on
sentence, E is the best answer.

As for your scoring questions, I have two thoughts. First, there is basically no benefit to answering the first 6 questions correctly if you are unable to
work relatively accurately through the entire section. I am not sure who created or perpetuated this myth about the early questions, but answering them
correctly is only beneficial if you are then able to continue to correctly answer those questions above your "ability level" throughout the remainder of the
test. If not, your score will regress, especially when you miss 5 of the final 6 questions (as you did). Second, the number of questions you answer
correctly, though important, is not vital to your score. What truly matters is which questions you answer correctly. To achieve your goal score you must
answer most of the questions below this difficulty level correctly. If you are inconsistent with questions of a difficulty below your "ability level," you
will never rise to a point that places you firmly within your desired range.

Hope that helps. Please feel to reply with further questions.


-dan

RonPurewal
ManhattanGMAT Staff
Tue Jun 10, 2008 6:41 am

guest wrote:former must be removed not for grammatical reasons but to correct the meaning of the statement. With "former" left in
there it would imply that the women had the role of "former chair" for the 18 consecutive years. Instead she was the actual chair for
those 18 years. She is now the former chair but was not such during those 18 cons years. hope this helps.

indeed.

RonPurewal

ManhattanGMAT Staff

Re: As the former chair of the planning board for 18 consecutive

Tue Apr 28, 2009 6:33 am

soundok wrote:Could I use " The chair of..." rather than" AS the former chair" ?

you could probably do that, but it wouldn't be as clear as the correct option here.

if you did that, it wouldn't be 100% clear that she undertook all these actions while she was chair.

it wouldn't be grammatically correct, but "as" makes it crystal clear that the cited actions took place during her tenure as chair.
tankobe
Prospective Students
excellent explanation...

RonPurewal
ManhattanGMAT Staff

Re: As the former chair of the planning board for 18 consecutive

Fri Feb 17, 2012 7:23 am

teal777 wrote:I still don't understand what is wrong with (B)? I though (B) has special idiom used correctly -

'article + designations of the person + , + main clause beginning with the name of the person........

(b) has a nonsense meaning.

for an excellent explanation, read the fourth post in this thread, the post written by "guest" (in all lowercase -- i.e., not the post by
"Guest" that appears directly before it).

RonPurewal
ManhattanGMAT Staff
Re: As the former chair of the planning board for 18 consecutive

Mon May 07, 2012 4:16 am

dave, you're neglecting the timeframe in the modifier. it's not possible to be a "former X for 18 years".

you can be an X for 18 years, whatever X may be, but "former X" is something that lasts forever.

RonPurewal
ManhattanGMAT Staff

Re: As the former chair of the planning board for 18 consecutive

Thu May 17, 2012 9:52 am

davetzulin wrote:thanks Ron, that makes perfect sense.

you're welcome.

RonPurewal
ManhattanGMAT Staff

Re: As the former chair of the planning board for 18 consecutive

Mon Apr 28, 2014 1:41 pm


cshen02 wrote:Hey guys!

Just made up three forms of openings:


As the chair, Joan Philkill

If followed by an account of something Ms. Philkill actually did as chair of whatever committee, sure, this could work.

"Chair" must also be followed by "of xxxxx committee/organization", of course, unless the identity of that organization is already
clear in context. (You can't just say that someone is "the chair" without answering the question Chair of what?)

A chair Joan Philkill

This could not work. I don't think anything like it could ever work, either.

RonPurewal
ManhattanGMAT Staff

Re: As the former chair of the planning board for 18 consecutive

Mon Apr 28, 2014 1:42 pm

The chair Joan Philkill

This kind of construction exists. For example, you can write After he first announced his retirement from the NBA, (the) basketball
star Michael Jordan had a short career in professional baseball.
"The" isn't generally necessary; it is most often used when this type of construction starts the sentence.

With "the chair", though, this construction doesn't work, unless we already know (from some previous sentence) the answer to Chair
of what?
(You can't put "chair of xxxxxxx" into this construction; it's too long. The construction becomes unreadable.)

RonPurewal

ManhattanGMAT Staff

Re: As the former chair of the planning board for 18 consecutive

Mon Apr 28, 2014 1:50 pm

cshen02 wrote:I remember Ron wrote a good one on this....just couldn't recall where I read it first....Would anyone
help?

Cheers!

Maybe one of these:

post31581.html#p31581
post35618.html#p35618

RonPurewal

ManhattanGMAT Staff

Re: S.C : Thelonious Monk, who was a jazz pianist

Wed Sep 30, 2009 5:45 am

jonathancreange wrote:I dont understand in sentence d why we dont write :

"Jazz pianist and composer, Thelonious Monk" or " The Jazz pianist and composer Thelonious Monk".

this is a special idiom. if you preface someone's name with a noun describing their occupation (or other word describing what that
person does), WITHOUT 'A'/'AN', you DO NOT use a comma.

if there's an article, you DO use a comma.

if it's an adjective, you DO use a comma.

example:
Jazz pianist and composer Thelonious Monk ... --> correct
A jazz pianist and composer, Thelonious Monk ... --> correct
Creative and original, Thelonious Monk ... --> correct
because without the "," it seems strange like saying : "Writer and British Shakespeare wrote good books."

well, "writer" is a noun, and "british" is an adjective. that's nonparallel, and so is wrong no matter what country you come from.

--

also"and most importantly"don't forget that the gmat does not test punctuation. so, none of this merits your time or
consideration in the first place.

RonPurewal

ManhattanGMAT Staff

Re: Restrictive vs non-restrictive modifier

Fri Jan 08, 2010 6:21 am

guys, the situation is more complicated with "THE".

here's the deal:

if you are using "THE (description) (name)" to introduce someone for the first time, then you DON'T use a comma.

if you have already introduced two or more people, but you are using "THE (description) to single out one of them,
then you DO use a comma.

here are some examples:


The author Ernest Hemingway was known for his drunken and violent escapades.
--> here we are introducing hemingway, so we don't use a comma.

Among her friends were an author and a painter; the author, Ernest Hemingway, went on to become an icon of American literature.
--> we've already introduced "the author"; this time we use a comma to single him out.

hope that helps

RonPurewal
ManhattanGMAT Staff

Re: As the former chair of the planning board for 18 consecutive

Mon May 26, 2014 11:52 am

asnkarlygash wrote:1) A novelist who turned away from literary realism to write romantic stories about the peasant
life and landscape of northern Sweden, Selma Lagerlof became in 1909 the first woman and also the first Swedish
writer to win the Nobel Prize for Literature.

vs. As a novelist

This sentence could be written with "As a novelist", but it would be somewhat weird (and unnecessary) to circumscribe Lagerlof's
accomplishments in that way.

If she'd had two entirely different careers, then this sort of modifier would make more sense:
As a novelist, Lagerlof won a Nobel Prize; as an artist, on the other hand, she remained relatively unknown for her entire life.

This last sentence cannot be written without the two instances of "as", since it must convey that essentially opposite things were
true in different aspects of her life.

RonPurewal

ManhattanGMAT Staff

Re: As the former chair of the planning board for 18 consecutive

Mon May 26, 2014 11:53 am

2) As the chair of the planning board for 18 consecutive years and a board member for 28 years, Joan Philkill
attended more than 400 meetings and reviewed more than 700 rezoning applications.
vs. The former. (Why can't a former chair for 18 consecutive years? It is ok with me. Besides the meaning stuff, is it
grammatically correct? I think it is grammatically correct because the 1) sentence is the correct answer without AS)

Trying to separate grammar from meaning is unwise (and, in most cases, impossible). They work together, not separately.

Yes, this sentence is "grammatically correct", in the sense that OTHER sentences can work correctly with the same structure.
With this particular sentence, though, this construction is nonsense, for the reasons explained earlier in this thread.

Sreeharshak368
Students

Re: As the former chair of the planning board for 18 consecutive

Sun Dec 06, 2015 4:28 pm

How is option D a run on sentence? Can someone explain in some detail?

Is it because "She was" creates an independent clause that should end with a semi colon or a full stop. The moment we use a
comma between two independent clauses they become a run on sentecne

RonPurewal

ManhattanGMAT Staff

Re: As the former chair of the planning board for 18 consecutive

Wed Dec 09, 2015 10:19 am

i don't know the terminology you're using. so, if the terms are important, then i might not understand what you are asking.

in any case:
She was xxxx for 28 years. --> this is a complete sentence
Joan Pilkin did this and did that. --> this is another complete sentence

you can't just glue 2 complete sentences together with a comma.


Question 19

Question 19
https://gmatclub.com/forum/most-of-the-purported-health-benefits-of-tea-comes-from-105341.html

Difficulty: 25% (medium/low)


Question Stats: 64% (01:39) correct 36% (00:51) wrong based on 1880 sessions
A:6% B:4% C:11% D:65% E:15%

kt00381n
Manager

I would pick D for parallelism here. "...that are ..found..and that inhibit.
In C ..."compounds also found ...and inhibit' I think it would be correct to say : "compounds also found, inhibit" without "and".

daagh
Retired Moderator

What is wrong with C?

When fully expanded C will read:

Most of the purported health benefits of tea come from antioxidantscompounds also found in beta carotene, vitamin E, and
vitamin C, and inhibit the formation of plaque along the bodys blood vessels.

The subject of the first part is - most (of the health benefits)- and the verb is come -. By default, the subject of the second part
is -( most ) of the health benefits -and its verb Inhibit.

You can see the distortion of the meaning that it is the purported health benefits that inhibit plaque rather then the anti-oxidants.
This is why C is not the right choice.

In E, the pronoun they may refer to the health benefits since subject rightly takes priority over any other noun that is placed nearer
than the subject, per se.

daagh
Retired Moderator

The dash (-) is a parenthetical sign (akin to a colon ) that describes the antioxidants, .
- compounds also found in beta carotene, vitamin E, and vitamin C,. This is an essential feature of the sentence and the text will
maintain the original meaning , even if the parenthetical contents removed. All the same, the subject remains - most. -

My intent is to point out the distortion of the meaning that it is the purported health benefits that inhibit plaque rather then the anti-
oxidants.

This is why C is not the right choice.

But I never said anything about S-V agreement. Have I been misunderstood?

EducationAisle
Director

Hmmas per your suggestion, substituting "-" for a ":" in C would read:

Most of the purported health benefits of tea come from antioxidants: compounds also found in beta carotene, vitamin E, and
vitamin C, and inhibit the formation of plaque along the bodys blood vessels.
Are you saying that compounds modifies antioxidants, while inhibit modifies Most of the purported health benefits?

I would strongly agree with metallicafan that phrases and clauses cannot be parallel and hence, C is not correct.

EducationAisle
Director

Hi!! The endeavor here is to figure out why C is wrong. C would read:

Most of the purported health benefits of tea come from antioxidants- compounds also found in beta carotene, vitamin E, and
vitamin C, and inhibit the formation of plaque along the bodys blood vessels

If instead of -, we had a comma, then the sentence would read:

Most of the purported health benefits of tea come from antioxidants, compounds also found in beta carotene, vitamin E, and
vitamin C, and inhibit the formation of plaque along the bodys blood vessels

In this case, 'compounds.vitamin C' would be a non-essential modifier and so, 'inhibit the formation' would describe Most of the
purported health benefits (and is hence, logically incorrect).

However, since in the actual sentence, we have - and not comma, we are wondering if 'inhibit the formation' still describes Most
of the purported health benefits.

p.s. Modifiers are not a part of speech but an umbrella category, if you will. Perhaps you would be aware that adjectives, adverbs
and all adjective and adverb phrases fall into this category of modifiers
sayantanc2k
Verbal Expert

abhishek03050 wrote:
Isn't compounds Plural?
I thought for Plural, we sould use THOSE and not THAT

In this answer, we have.. compounds that

My undersatnding made me eliminate option D .

mvictor wrote:
raghavs wrote:
Most of the purported health benefits of tea comes from antioxidantscompounds also found in beta carotene, vitamin E, and vitamin C that
inhibit the formation of plaque along the bodys blood vessels.

A. comes from antioxidantscompounds also found in beta carotene, vitamin E, and vitamin C that
B. comes from antioxidantscompounds that are also found in beta carotene, vitamin E, and vitamin C, and they
C. come from antioxidantscompounds also found in beta carotene, vitamin E, and vitamin C, and
D. come from antioxidantscompounds that are also found in beta carotene, vitamin E, and vitamin C and that
E. come from antioxidantscompounds also found in beta carotene, vitamin E, and vitamin C, and they

A and B are out right away because of the SV agreement error.


now..meaning is:
benefits of tea come from A
A - compounds that are found in X, y, z and that inhibit the formation.
C - benefits come and inhibit. so no.
D - looks good
E - they who? antioxidants? benefits? compounds? vessels? since D looks better, E is out.
"That" as relative pronoun can refer to singular or plural antecedent - OG has plenty of such examples.

https://www.manhattanprep.com/gmat/forums/most-of-the-purported-health-benefits-of-tea-t2558.html

RonPurewal
ManhattanGMAT Staff

Re: Most of the purported health benefits of tea

Wed Mar 19, 2008 5:14 am

mclaren7 wrote:E --- compounds ... and they (anything wrong with this structure?)

yes, something is indeed wrong.

at best, 'they' is an ambiguous pronoun that potentially refers either to 'antioxidants'/'compounds' or to 'health benefits'. at worst it
refers to 'health benefits', the subject of the preceding sentence, by default. either way, you've got problems.

the other problem is that a single dash of the sort that sets off the descriptor in this problem is akin to a single comma: both set off
an appositive phrase, which is NOT allowed to contain independent clauses (such as the one beginning with 'they' in choice e).
choice d follows the rules here, as, after the dash, it contains only subordinate clauses.
...all kinds of trouble :(

StaceyKoprince
ManhattanGMAT Staff

Thu Jul 10, 2008 1:19 pm

You can have either a single long dash or two long dashes - though they're relatively rare, I have seen them on the GMAT. D uses the single dash
correctly; E does not.

RonPurewal
ManhattanGMAT Staff

Fri Nov 21, 2008 8:48 am

aaa wrote:Is the parallelism in E wrong? "also found" vs. "they inhibit". Thanks

yes, that's bad parallelism.


there's also a pronoun issue with "they". at best it's ambiguous, and at worst it appears to refer to "health benefits", the noun with which it's parallel.

esledge

ManhattanGMAT Staff

Re: Most of the purported health benefits of tea

Mon Mar 09, 2009 3:41 pm

The long dash is used to indicate an example or an aside - a little bit of extra info that is related to the sentence but is presented
outside the core of the sentence.

Only the example or aside goes inside the dash, so that example either has to go all the way to the end of the sentence or we need
to "close" the dash at some point by putting a second one in.

Example:
The top winners of the school spelling bee - Jonah, Kari, and Lou - will compete in the regional spelling bee next month.

A good analogy for these two-dash sentences is that dashes = parentheses. As such, typically you will only find examples or other
subordinate clauses within them. I don't know for sure whether independent clauses are forbidden within the dashes, but it seems
unlikely that a GMAT answer would hinge on that. Note that our complaint with (E) was not just that the phrase after the dash was
an independent clause, but that the phrase meandered away from a limited and clear aside about antioxidants. In (D), the phrase
after the dash clearly described the antioxidants as compounds (that X and that Y).
RonPurewal
ManhattanGMAT Staff

Re: Most of the purported health benefits of tea

Sun Sep 20, 2009 6:15 am

1. I suppose that "benefit" is a countable noun. (1 benefit, 2 benefits, 3 benefits...). Correct?

yes.

in general, if something appears in a plural form, it's overwhelmingly likely that that thing is countable.

2. Most ( one among the 5 special Indefinite pronoun - SANAM ) is a plural subject because the object of the "of" construction is
"benefits" - plural.
Is the knowledge of count or non-count noun necessary to determine whether we should have a singular or plural noun ?

absolutely.

most of the chairs are gone. (countable)


most of the furniture is gone. (mass noun, not countable)

3. Is there any rule for "quantifier + of + noun"? What exactly determines the verb that should follow ?

there's no 1 magic rule, unfortunately.

it does follow the countable / not countable distinction pretty closely, although you should think about CONTEXT as well.
RonPurewal
ManhattanGMAT Staff

Re: Most of the purported health benefits of tea

Thu Aug 26, 2010 9:33 am

manish1sinha wrote:Ron,
Could please explain the following:
1)the role of single dash--can a single dash have a independent clause or only sub clause is permitted?if there is a
dash followed by a comma can the rule of 'both' be applied to it?

if you use a single dash, then the stuff that is set off by the dash must actually continue all the way to the end of the sentence. if
you are going to block off a modifier with dashes, then you must use two dashes to block off the modifier.

i.e., there is no such thing as a modifier that is blocked off by a dash on the left, but by a comma on the right. if you have a modifier
that does not extend to the end of the sentence, then you must use either two dashes or two commas to set it off.
therefore, your question here ("if there is a dash followed by a comma") actually doesn't make sense, because in any such situation
the dash and the comma would be entirely unrelated to each other.

2)by "both"--> you meant 2 dashes= 2 commas?

in that context, i meant


COMMA + (modifier that continues until the end of the sentence)
is the same as
DASH + (modifier that continues until the end of the sentence)

3)In choice 'E' there is a 'comma and' so I thought an independent clause is what is required there.

incorrect analysis.
i posted the correct analysis of this sort of situation here:
post43518.html#p43518

RonPurewal
ManhattanGMAT Staff

Re: Most of the purported health benefits of tea

Thu Aug 26, 2010 9:33 am

manish1sinha wrote:Ron,
Could please explain the following:
1)the role of single dash--can a single dash have a independent clause or only sub clause is permitted?if there is a
dash followed by a comma can the rule of 'both' be applied to it?

if you use a single dash, then the stuff that is set off by the dash must actually continue all the way to the end of the sentence. if
you are going to block off a modifier with dashes, then you must use two dashes to block off the modifier.
i.e., there is no such thing as a modifier that is blocked off by a dash on the left, but by a comma on the right. if you have a modifier
that does not extend to the end of the sentence, then you must use either two dashes or two commas to set it off.
therefore, your question here ("if there is a dash followed by a comma") actually doesn't make sense, because in any such situation
the dash and the comma would be entirely unrelated to each other.

2)by "both"--> you meant 2 dashes= 2 commas?

in that context, i meant


COMMA + (modifier that continues until the end of the sentence)
is the same as
DASH + (modifier that continues until the end of the sentence)

3)In choice 'E' there is a 'comma and' so I thought an independent clause is what is required there

incorrect analysis.
i posted the correct analysis of this sort of situation here:
post43518.html#p43518

mschwrtz
ManhattanGMAT Staff
Re: Most of the purported health benefits of tea

Tue Oct 19, 2010 9:28 pm

There is a tricky distinction here between two uses of that. When that is used as a relative pronoun (to introduce a noun-modifying
clause) it can modify a singular or a plural verb. When that is used as what is called a 'demonstrative pronoun,' it must be singular.

Now, I don't want you to memorize the term 'demonstrative pronoun,' so here's an easy way to keep track: If that touches the noun
or noun phrase it modifies, it can be singular or plural. If that refers to something it doesn't touch, it must be singular.

And don't forget that that is used in other ways to. In that sentence I just wrote it introduces a subordinate clause. And in the second
sentence of this paragraph that is an adjective.

jnelson0612
ManhattanGMAT Staff

Re: Most of the purported health benefits of tea

Sat Apr 30, 2011 12:54 pm

pitchaivijay wrote:Why C is wrong ?


Let's compare C to the correct answer, D.

C. come from antioxidants"compounds also found in beta carotene, vitamin E, and vitamin C, and
D. come from antioxidants"compounds that are also found in beta carotene, vitamin E, and vitamin C and that

Since the word "and" is a parallel marker, we must check parallelism. Notice how D is parallel after the dash:
compounds that are . . . . and that inhibit
We have nice verb parallelism here.

C breaks this parallelism by joining a noun and a verb:


compounds . . . and inhibit

RonPurewal
ManhattanGMAT Staff

Re: Most of the purported health benefits of tea

Sat Oct 15, 2011 5:00 am

dianapaolasanchez wrote:Most of the purported health benefits of tea come.... (plural) CORRECT

What happened if the sentence starts by...

The most of the purported health benefits of tea.... (singular)


there is no such construction in english.

perhaps you are thinking of "the most ADJ" -- as in this song is the most beautiful thing i've heard in months.
or perhaps you are thinking of the spanish noun la mayora, which can't be translated in this way.

RonPurewal
ManhattanGMAT Staff

Re: Most of the purported health benefits of tea

Tue Jul 03, 2012 6:16 am

mcmebk wrote:3. An additional question about E, if the dash is changed to comma, will it be a run-on sentence?
meaning it should be "; and they inhibit..."?

no.
your version (with the semicolon) would be incorrect, since "and they..." isn't a sentence by itself.

jlucero
ManhattanGMAT Staff
Re: Most of the purported health benefits of tea

Thu Oct 04, 2012 5:29 pm

vikram4689 wrote:1) do above 2 quotes mean that NONE of single dash or double dashes can separate an
Independent Clause from main clause.

2) Also following must be incorrect. are they ?


a) Time spent in prison can change your outlook - especially if you have a life sentence.
b) Jack was gentle, but persistent, with Carolyn - he would make her his wife.

1) Main clauses are independent clauses, so your question doesn't entirely make sense. However, you need to include the whole
main clause of a sentence in the front half of clause with one dash, or outside of the double dashes.

2) I'd say both of these are incorrect because they are too related to one another to be separated by the dash. HOWEVER, the GMAT
does not test your ability to know whether something should be broken up by a dash, a comma, or a parenthesis, so don't stress
yourself out here.

RonPurewal
ManhattanGMAT Staff
Re: Most of the purported health benefits of tea

Sun May 04, 2014 11:47 am

When you analyze parallelism, you should start by identifying the items that actually SHOULD appear in parallel.

This is a function of (i) context and (ii) common sense. It has nothing whatsoever to do with the grammar of the original sentence.
(Remember, the original sentence is incorrect in 80% of all cases!)

Once you have identified the structures that should be parallel, you should not look at any other structures! Other structures
are irrelevant.

In this problem, the sentence presents two interesting facts about antioxidants:
1/ They are also found in xxxxx other things.
2/ They inhibit the formation of yyyy stuff.
It should be 100% clear that these are "#1 and #2", or "bullet points", or however else you intuitively think about parallel
structures. If this is not clear, you are not yet ready to look at the answer choices.

So, when you think about parallelism, your eyes shouldn't be going anywhere near "come(s)" in the first place.

AceTheGM@

Course Students

Posts: 20
Joined: Mon Sep 09, 2013 1:25 pm

Re: Most of the purported health benefits of tea

Wed Jul 16, 2014 5:42 pm

pitchaivijay wrote:Why C is wrong ?

Let's compare C to the correct answer, D.

C. come from antioxidants"compounds also found in beta carotene, vitamin E, and vitamin C, and
D. come from antioxidants"compounds that are also found in beta carotene, vitamin E, and vitamin C and that

Since the word "and" is a parallel marker, we must check parallelism. Notice how D is parallel after the dash:
compounds that are . . . . and that inhibit
We have nice verb parallelism here.

C breaks this parallelism by joining a noun and a verb:


compounds . . . and inhibit

I understand why D is parallel (compounds that are...and that inhibit...), but why isn't C parallel? (compounds also... and inhibit)

RonPurewal
ManhattanGMAT Staff

Re: Most of the purported health benefits of tea

Thu Jul 17, 2014 4:50 am

There's no verb to be parallel to "inhibit".

"Found" is not a verb (obviously, since compounds can't find things). It's a description of the compounds (they are found in x and y
places).

RonPurewal
ManhattanGMAT Staff

Re: Most of the purported health benefits of tea

Sat Jan 31, 2015 3:29 pm

anshultomar wrote:Hi Ron,

If answer choice C had been "Compounds that are found < blah blah > and inhibit <blah blah> " then would have
been parallel and okay?

We can assume <that> in the second part of the parallel item.

Thanks
AT

technically that would be fine, but it's bad writing. (if you were presented with that sentenceand you weren't already familiar
with itthere's no way you would understand it in a single reading; you'd have to go over it several times to understand it.)

RonPurewal
ManhattanGMAT Staff

Re: Most of the purported health benefits of tea

Sat Jan 31, 2015 3:31 pm

^^ and, more generally, this is a good illustration of why you shouldn't "edit" official SC problems, and you shouldn't invent
your own answers to them.

in almost all such casesincluding the case at handthe answer is "no, that's wrong, but you don't need to worry about why."

this exam tests a VERY narrow band of the things that can go wrong in written english.
Question 20

Question 20
https://gmatclub.com/forum/in-archaeology-there-must-be-a-balance-between-explanation-78777.html
A:5% B:77% C:6% D:1% E:11%

Difficulty: 5% (low)
Question Stats: 78% (01:34) correct 22% (00:47) wrong based on 951 sessions

tenaman10
Senior Manager
In archaeology, there must be a balance between explanation of the value and workings of archaeology, revealing the mysteries of
past and present cultures, and to promote respect for archaeological sites.

A. between explanation of the value and workings of archaeology, revealing the mysteries of past and present cultures, and to
promote

B. among explaining the value and workings of archaeology, revealing the mysteries of past and present cultures, and
promoting..parallel and among X,Y and Z format is required ..CORRECT

C. between explaining the value and workings of archaeology, revealing the mysteries of past and present cultures, and when
promoting

D. among explaining the value and workings of archaeology, the revelation of the mysteries of past and present cultures, and to
promote

E. between explaining archaeologys value and workings, in the revealing the mysteries of past and present cultures, and in
promoting...explaining and in revealing is not parallel ..
MichaelS
Manhattan GMAT Instructor
As other commenters have noted, between is standard for two items, and among for three or more. The split between among and
between looks like a great place to start, then, unless you're not quite sure how many items the sentence is talking about. I wasn't
sure, so I started with parallelism.

The first and seems merely to link value and workings, and that compound is just the object of a preposition in one item in the
longer list of the things that must be balanced, explanation..., revealing..., and to promote..... You could now eliminate answers that
use between. (We should be careful that in some answers the list isn't somehow rejiggered to be just two items, each perhaps its
own compound, but that doesn't turn out to be an issue here.)

So eliminate A, C, and E.

In B, the three parallel items are all gerunds, explaining..., revealing..., and promoting.... That's perfectly fine.

In D, the three parallel items are the simple gerund explaining, the noun the revelation, and the infinitive to promote. No two of
those things are parallel, so eliminate D.

B it is.

By the way, if you'd like to see an earlier post of mine on parallelism issues with gerunds and other nouns, check out mohica-
civilization-88664.html

sanaexam
Intern
It is not east to realize that thr are 3 items.Its a 600-700 level question.:-/

http://www.beatthegmat.com/good-question-archaeology-values-t38667.html

Rich.C@EMPOWERgmat.com Beat The GMAT Legend

Hi All,

There SC is based on a specific style issue and parallelism:

1) "Between" vs. "Among": the word "between" applies to 2 ideas; "among" applies to 3 or more. Here, we have a list of 3 items, so
the word "among" is correct. Eliminate A, C and E

2) A 3-item List requires parallel items (in the same format). B uses 3 "-ing" verbs; D uses 1 "-ing" verb, 1 noun and 1 infinitive verb.
Eliminate D.
Final Answer: B

GMAT assassins aren't born, they're made,


Rich

ceilidh.erickson GMAT Instructor

neptune28 wrote:
I'm not sure I'd say that "balance among" is wrong, but "balance between" just sounds better to me. In addition, a Google search reveals that use of the
latter is about 30 times as common!

Be careful! This is an excellent example of how the GMAT tricks your ear with uncommon expressions that are, in fact, correct! You can't simply rely
on your ear if you want to get a good score. Other commenters are correct: "between" is used to compare two things, and "among" to compare 3 or
more. "Balance between" sounds better and is more common because in real life, we talk more often about balancing 2 things than balancing 3 or more
(just picture a circus performer doing a balancing act - balancing 2 things is easier than balancing 3 things!).

Whenever you notice a comma followed by AND, you're looking at a parallel structure. So in this original, as soon as you see "... , and to promote..."
you should ask yourself what the parallel list is. In the original sentence, the list was:
... a balance between:
1. explanation of the value and workings of archaeology
2. revealing the mysteries of past and present cultures
3. to promote respect for archaeological sites

Whenever we see parallelism, we want to check 2 things: MEANING and STRUCTURE. Should these 3 things be parallel from a meaning perspective?
Sure, these are all objectives of the field of archaeology. Are they structurally parallel? No. So, look through the answer choices and simply find the one
with matching structures:

A. between explanation of the value and workings of archaeology,


revealing the mysteries of past and present cultures,
and to promote
B. among explaining the value and workings of archaeology,
revealing the mysteries of past and present cultures,
and promoting
C. between explaining the value and workings of archaeology,
revealing the mysteries of past and present cultures,
and when promoting
D. among explaining the value and workings of archaeology, the
revelation of the mysteries of past and present cultures,
and to promote
E. between explaining archaeology's value and workings, in the
revealing the mysteries of past and present cultures,
and in promoting

How to Tackle Every Single GMAT Problem (Seriously!) Part 3


by Stacey Koprince Aug 17, 2015

Welcome to part 3 of our series on how to answer every single GMAT problem youll ever see. If you havent already read the earlier installments,
start with part 1 and work your way back to me.

This time, were going to test out the process with a GMATPrep Sentence Correction question from the free exams. Here you go:

In archaeology, there must be a balance between explanation of the value and workings of archaeology, revealing the mysteries of past and present
cultures, and to promote respect for archaeological sites.

(A) between explanation of the value and workings of archaeology, revealing the mysteries of past and present cultures, and to promote

(B) among explaining the value and workings of archaeology, revealing the mysteries of past and present cultures, and promoting

(C) between explaining the value and workings of archaeology, the revealing of the mysteries of past and present cultures, and when promoting
(D) among explaining the value and workings of archaeology, the revelation of the mysteries of past and present cultures, and to promote

(E) between explaining archaeologys value and workings, in the revealing of the mysteries of past and present cultures, and in promoting

Got your answer? Lets do this!

First, Glance at the problem. What does that even mean for SC? Theres so much text!

Dont read the sentence yet. Heres where to glance:

Start with the word before the underline and the first underlined word
Then glance down the first word or two of each answer choice

Thats it! Now what do you do with that information?

Some words are markers all by themselves: if you see the word and, you know something is going on with parallelism.

Other clues make themselves known when you compare differences in the answers, and theres always one difference at the beginning of each answer
choice. You may not always know what the difference signifies, but if you train well, you can learn to spot clues on a First Glance something like 70%
to 80% of the time.
Why do this before you even read the sentence? On SC, its often hard to find a starting point, since each sentence could be testing anything and just
what its testing is not always obvious. Sometimes you read a sentence and just shrug, not sure whats going on. If your first glance can provide you with
a valuable clue as to what is being tested (or what might be tested), then you can read that sentence with a point of view: youre actively looking for a
particular issue. That will more quickly point you towards at least one of the issues going on in the sentence!

So go back up and glance at that problem right now. What do you notice?

balance between

The word between is part of the idiom between X and Y. Less commonly, you may see a split in the answers between between and among. (See what I
just did there? ) Glance down the beginning of the answers yep, this one is testing between vs. among. Id jot down between/among on my scrap
paper. (Actually, Id use shorthand and write bet/am. I know what this means because Ive used this abbreviation before.)

Excellent! Im jumping to Reflect because my Glance actually gave me something to think about. Do you know how those two words are used?
Between is used when talking about exactly two things; among is used for three or more things. Now I know what to think about when I go back up for
my next step: Read.

Okay, the sentence has a list of three: explanation, revealing, and to promote. I know immediately thatbetween is wrong; I need to choose an answer that
uses among. Also, this construction requires parallelism and that list does not look very parallel. Ill come back to that in a minute.
Work! Cross off answers (A), (C), and (E). That was niceonly two to go. Now lets go back to check out that parallelism issue.

(B) among explaining the value and workings of archaeology, revealing the mysteries of past and present cultures, and promoting

(D) among explaining the value and workings of archaeology, the revelation of the mysteries of past and present cultures, and to promote

Answer (B)s list items are explaining, revealing, and promoting. Looking nicely parallel!

Answer (D)s list items are explaining, the revelation, and to promote. A participle, a noun, and an infinitive verb: not parallel! (An ing word can be a
noun or a verb, but a noun and an infinitive verb cannot be parallel to one another.)

The correct answer is (B). And I barely had to read anything among those five long answer choices. (See what I did again there? )

You may have noticed that, on SC, we have to back and forth between the steps of the process a lot. This is one of the annoying parts of SC. Every time
you get a clue, you figure out what it means (Reflect & Organize!) and cross off some answers (Work!), but then you typically have to go back and start
all over again. One clue typically wont allow you to cross off all four wrong answers (though this does happen sometimes).

In fact, we incorporate these ideas into our 4-step SC Process. (Yes, I know that the article I just linked technically lists 5 steps but the fifth step is
really just repeat.)

Try the process out again on the GMATPrep problem below and Ill give you the answer in our next installment.

The new image of Stone Age people as systematic hunters of large animals, rather than merely scavenging for meat, have emerged from the
examination of tools found in Germany, including three wooden spears that archaeologists believe to be about 400,000 years old.

(A) merely scavenging for meat, have emerged from the examination of tools found in Germany, including

(B) as merely scavenging for meat, have emerged from examining tools found in Germany, which include

(C) as mere meat scavengers, has emerged from examining tools found in Germany that includes

(D) mere scavengers of meat, has emerged from the examination of tools found in Germany, which includes
(E) mere scavengers of meat, has emerged from the examination of tools found in Germany, including

Key Takeaways for Every Problem You Will Ever Do:


(1) First, you want to see whether a quick look can give you an early idea of one topic the sentence may be testing. Glance at the word right before the
underline and at the first word of the underline: any clue markers there? Next, compare the first word or two of each answer choice: do the differences
signal any particular issues?

(2) If so, Reflect briefly on what is or might be happening, then Read the original sentence, Jotting down any important markers or reminders as you go.
When youve found a starting point, Reflect again to decide what to do with this piece of information.

(3) When you know youve got an error, cross off that answer choice and any others that repeat the same error. As you do this, keep an eye out for clues
that can help you find the next starting point, as in this problem when the first clue (between / among) led me to the second issue (parallelism). After
several rounds, youll either be down to one answer or youll realize that this one is too hard and youll guess from among the remaining choices.
Question 23

Question 23
https://gmatclub.com/forum/the-striking-differences-between-the-semantic-organization-133932.html

Difficulty: 5% (low)
Question Stats: 70% (01:25) correct 30% (00:22) wrong based on 773 sessions
A:70% B:6% C:17% D:3% E:4%

daagh
Retired Moderator
The subject is plural differences; hence, we need have as the verb. Ditch B, D and E, Between A and C, the relative pronoun which
modifies European languages a) distorting the meaning and b) flouting touch rule. Therefore, A is the correct choice.

gmatsaga
Manager
Subject: striking differences (plural) = should have a plural verb - "have"

Eliminate B D and E

Parallelism: the semantic organization (singular) = should be parallel to the other element

Eliminate (C) because of "those"

We have (A)

http://www.beatthegmat.com/striking-differences-t57020.html

grockit_andrea GMAT Instructor

To agree with the plural subject "differences," you need the plural verb "have," which allows you to narrow the options down to A and C. "Semantic
organization" is singular, so the correct pronoun for parallelism is the singular "that." Therefore, A is the correct answer. In addition, the modifying
phrase that begins with "which" in C seems to be referring to "european languages," which is incorrect.

Question 23
_________________
Andrea A.
Grockit Tutor

https://www.manhattanprep.com/gmat/forums/striking-differences-t7729.html

RonPurewal
ManhattanGMAT Staff

Re: Striking differences

Thu Aug 27, 2009 4:18 am

prengasn wrote:We need to make a comparison to "the semantic organization" which is singular. Thus we must
choose a singular referent: That instead of those. It's as saying:
"The striking differences between the semantic organization of ....and that(the semantic organization) of......."

Hope that helps

right on.

well explained.

Question 23
jlucero
ManhattanGMAT Staff

Re: Striking differences

Sat Nov 10, 2012 7:45 pm

byuwadd wrote:Okay, what if the author had left off "that of?"

The striking differences between the semantic organization of Native American languages and European languages,
in both grammar and vocabulary, have led...

Do we need the 'that of' there?

No, because of there being 2 parallel markers: The differences between X and Y. X and Y must be parallel unless we change it to:
The differences in X between Y and Z. The differences in organizations between NA languages and E languages.

RonPurewal
ManhattanGMAT Staff

Re: Striking differences

Sat Dec 22, 2012 7:56 pm

ajithalexjacob wrote:What are your thoughts on the usage of "in both grammar and vocabulary" vs "which include

Question 23
grammar and vocabulary" in choices A and C?

Is it alright to use either of them?

the latter ("...include...") creates a sentence with a nonsense meaning.

that construction yields a sentence that says "the striking differences between xxxxx and yyyyy languages, which include grammar
and vocabulary..."
the problem is that grammar and vocabulary themselves are not actually "included" in this group.

tim
ManhattanGMAT Staff

Re: Striking differences

Thu Jan 03, 2013 9:10 pm

normally an -ing word that follows a comma is an adverbial modifier, but this is not an absolute rule, and i certainly would not
eliminate B on that issue alone..

mcmebk
Forum Guests

Question 23
Re: Striking differences

Sun Sep 01, 2013 5:08 pm

tim wrote:normally an -ing word that follows a comma is an adverbial modifier, but this is not an absolute rule, and i
certainly would not eliminate B on that issue alone..

Hi Tim/Ron

I think including is a special case of verb+ing - Including is indeed a new word and is used as prep rather than a present participle.

One thing I am a little uncertain about is, when we use including, should it always immediately follow the noun?

I enjoy all different kinds of movies including actions and horrors.

can we say, various types of movie are produced nowadays, including actions and horrors?

Thanks.

RonPurewal
ManhattanGMAT Staff

See, this is the kind of question you don't have to think about. Don't forget that the problems are multiple-choice!

If you face this issue, just take the choice that places "including" as close as possible to the thing it's describing.

Question 23
I.e., if you faced a choice between your version (given here) and "various types of movies, including xxxxx, are produced
nowadays", then you'd take the latter.

RonPurewal
ManhattanGMAT Staff

Re: Striking differences

Thu Sep 26, 2013 6:45 am

zhanghan.neu wrote:I feel like the 'senmantic organzation' here is a non-countable noun. I feel it's talking about
some kind of strucutre/arrangement of the languages. This is how I decided to use 'that' of.

No. The choice between "that of" and "those of" is 100% singular/plural, and 0% anything else.

Don't take simple things and make them complicated.

For example,

The difference between his bag and those of the students is that...

This is wrong, because "those" (plural) can't stand for "bag" (singular).

If that's how you want the sentence to work, then, unfortunately, you need to use another noun.
The difference between his bag and the other students' bags is...

Question 23
Oh well.

Go look at #60 in OG12, or #61 in OG13. Specifically, compare choices (B) and (E)

tim
ManhattanGMAT Staff

Re: Striking differences

Sat Oct 26, 2013 10:08 am

zhanghan.neu wrote: Has the rule changed every since? Just want to make sure. So 'that of the other three countries'
is ok?

Thanks in advance!

No, the rule has been the same all along. What you need to do is find the antecedent in the sentence and make sure it matches up
in number with the pronoun. I believe this is consistent with all the examples mentioned here and with Ron's comments.

tim

Question 23
ManhattanGMAT Staff

Re: Striking differences

Sat Oct 26, 2013 10:14 am

abhinavmishra666 wrote:Hi Ron,

My question is on the use of "infections such as" vs "such infections as", i thought the latter was incorrect as I was
told "such as is split up when we are trying to prove a point or show a degree". Please can you help explain the
usage of such 'something' as vs such as something.

Thanks.

I can't believe the GMAT would ever test this distinction, and furthermore I believe both are correct. Can you provide an example of
any question where you are required to decide between these two options?

1131570003
Students

Re: Striking differences

Wed Apr 22, 2015 9:06 pm

RonPurewal wrote:

Question 23
ajithalexjacob wrote:What are your thoughts on the usage of "in both grammar and vocabulary" vs "which
include grammar and vocabulary" in choices A and C?

Is it alright to use either of them?

the latter ("...include...") creates a sentence with a nonsense meaning.

that construction yields a sentence that says "the striking differences between xxxxx and yyyyy languages, which
include grammar and vocabulary..."
the problem is that grammar and vocabulary themselves are not actually "included" in this group.

I am little bit confused about this explanation.

What is not included in this group ? is it which refer to languages? Can you further elaborate on this issue? Thank you, Ron!

RonPurewal
ManhattanGMAT Staff

Re: Striking differences

Sun Apr 26, 2015 12:31 pm

grammar and vocabulary are not "differences between languages". (the differences between languages are differences in
grammar and/or vocabulary.)

Question 23
grammar and vocabulary are also not languages.

thus, there is nothing that "including grammar and vocabulary" can correctly modify.

RonPurewal
ManhattanGMAT Staff

Re: Striking differences

Sun Apr 26, 2015 12:32 pm

here's a workable version:

The striking differences between certain aspects of Native American and European languages, including grammar and
vocabulary, ...

if you understand why this version DOES work, then you should also understand why the other one doesn't.

RichaChampion

Students

Posts: 138

Question 23
Joined: Thu Jul 10, 2014 1:58 pm

Re: Striking differences

Fri Jul 22, 2016 4:59 am

RonPurewal wrote:

mcmebk wrote:One thing I am a little uncertain about is, when we use including, should it always
immediately follow the noun?

I enjoy all different kinds of movies including actions and horrors.

can we say, various types of movie are produced nowadays, including actions and horrors?

Thanks.

See, this is the kind of question you don't have to think about. Don't forget that the problems are multiple-
choice!

If you face this issue, just take the choice that places "including" as close as possible to the thing it's describing.

I.e., if you faced a choice between your version (given here) and "various types of movies, including xxxxx, are
produced nowadays", then you'd take the latter.

Question 23
Mr. Purewal,

I have some notes that I have collected from some analysis done on some other post by You.

"INCLUDING" is an EXCEPTION to the otherwise robust rules for comma+ing modifiers.

When you see "comma + including", you should think of "including" as a preposition, not as an -ing modifier. Therefore, "including
X" will become a prepositional phrase that describes the stuff preceding the comma.
Thanks for pointing this out / calling it to our attention -- we'll be sure to include it in our revised unit on modifiers in the course.

_______________________________________________________________

Can you please help me to analyse the usage of "including" in the option choice B -

(B) that of European languages, including grammar and vocabulary, has

You have explained in this in the above post, yet I am confused - https://www.manhattanprep.com/gmat/foru ... ml#p113657

What exactly is including modifying in the above Option?

I have one more question - I searched a lot of forums regrading the usages of prepositional phrase. I got confused seeing so much
information after doing google research. I thing this is a very common concept, but important too and at least 90% GMAT SC
questions will have this concept tested.

I have some information about the preposition "with" -

[Clause] + [comma]+ [with] WITH modifies the action (verb). It answers the questions how that action was performed.
[Noun] + [Comma] + [With] WITH Modifies the noun.

Question 23
I scored a century, with a special bat
Clause + comma + with modifies the verb and answers the question how the action was performed.

Starfish, with an innate ability to reproduce, reproduces very fast.


Noun + Comma + With - modifies the noun.

But preposition is was too broad concept.

Can you guide me how comma+prepositions (prepositional phrases) behave in the GMAT question. As far as I know they tend to
modify action before the comma? Am I right? Please guide me if I am correct or there are any exceptions.

RonPurewal

ManhattanGMAT Staff

Re: Striking differences

Sat Jul 23, 2016 7:56 am

RichaChampion wrote:(B) that of European languages, including grammar and vocabulary, has

What exactly is including modifying in the above Option?

that's a wrong answer choice -- which is wrong for a MUCH more obvious reason ("the differences has...") -- so there's no point in
struggling to pin this down.

Question 23
I have one more question - I searched a lot of forums regrading the usages of prepositional phrase. I got confused seeing so much
information after doing google research. I thing this is a very common concept, but important too and at least 90% GMAT SC
questions will have this concept tested.

at least 90%? so, nine out of every ten SC problems?


LOL

i mean... obviously you're exaggerating on purpose, but... the real figure is definitely no greater than 10 or 15 percent, and is
probably even smaller.

in general, if one of these phrases is separated by commas, then it should have some kind of relationship to the entire main
sentence (not just to a noun). this will normally be the case no matter where the phrase is located.

using the example you supplied (modified to make more logical sense) --
Starfish, with an innate ability to reproduce quickly, can produce over a thousand offspring in one week.
here, "with an innate ability..." doesn't ONLY modify "starfish". it also relates to the idea of the whole sentence -- it's the reason
WHY starfish can produce over 1000 offspring in a week.

Question 23
RonPurewal
ManhattanGMAT Staff

Re: Striking differences

Sat Jul 23, 2016 7:58 am

__

stillyou should make sure that you aren't over-emphasizing "rules" like this, and under-emphasizing the fact that this is a
multiple-choice test.
if you see one of these phrases in an answer choice, you'll ALWAYS be able to LOOK AT ITS COUNTERPARTS IN OTHER CHOICES, and
see what THOSE constructions are doing DIFFERENTLY. that will give you a much better -- and much more specific -- idea of what
actually matters about the specific phrase you're looking at.

in other words, any further questions about these kinds of phrases should be asked in terms of specific GMAT problems, rather than
as generalities.

Question 23
Question 32

Question 32
https://gmatclub.com/forum/according-to-a-survey-of-graduating-medical-students-80793.html

Difficulty: 55% (hard)


Question Stats: 54% (01:40) correct 46% (00:56) wrong based on 2759 sessions
A:20% B:14% C:54% D:8% E:4%

sudeep
Director
This is a GMATPrep question and using "to plan on" (no question on the validity, GMATPrep is the boss)

Here is something interesting from Manhattan Staff:

As per Ron, Manhattan Staff:

"plan on VERBing", by the way, is spoken language. it's substandard written language, and should be considered incorrect; the
correct form is "plan to VERB".
http://www.manhattangmat.com/forums/pos ... tml#p25515

TommyWallach
Manhattan GMAT Instructor

Hey All,

I was asked by private message to take this one on, even though there has been plenty of great stuff written already. Here we go:

This is a comparison question, so the whole point is to make sure we're comparing the right two things, and using the correct
terminology.

Question 32
42. According to a survey of graduating medical students conducted by the Association of American Medical Colleges, minority
graduates are nearly four times more likely than are other graduates in planning to practice in socioeconomically deprived areas.
(A) minority graduates are nearly four times more likely than are other graduates in planning to practice
PROBLEM: Should be "four times AS likely AS other graduates TO plan". That's a three-fer!

(B) minority graduates are nearly four times more likely than other graduates who plan on practicing
PROBLEM: Again "four times AS likely AS other graduates TO plan". This one doesn't ever complete, because the "who" opens up a
new modifier, and we never return to the main clause.

(C) minority graduates are nearly four times as likely as other graduates to plan on practicing
CORRECT.

(D) it is nearly four times more likely that minority graduates rather than other graduates will plan to practice
PROBLEM: "Four times AS likely" and RATHER THAN implies preference, which makes no sense here. Also, it's totally unclear what's
being compared.

(E) it is nearly four times as likely for minority graduates than other graduates to plan to practice
PROBLEM: "for minority graduates than other graduates" makes absolutely no sense at all. Where's the comparison?

Hope that helps!

-t

nusmavrik
Director
Tommy

Can I can infer that "X as likely as Y" ---> is comparing two nouns?

Question 32
thanks
TommyWallach wrote:
(C) minority graduates are nearly four times as likely as other graduates to plan on practicing
CORRECT.
-t

Hey Nusma,

Well, technically, it would be "X is Y times as likely as Z to do something". That's the full on comparison construction. But yes, the
two things have to be nouns, because the verb comes after the Z (to do something).

-t

Werewolf
Manager

TommyWallach wrote:
Hey Nusma,

Well, technically, it would be "X is Y times as likely as Z to do something". That's the full on comparison construction. But yes, the two things have to
be nouns, because the verb comes after the Z (to do something).

-t

Tommy,

Question 32
Can u plz explain how (or if) 'more likely' can be used? Not just in this example, but in general.
e.g., is it right to say, "X is more likely to succeed than Y"?

TommyWallach
Manhattan GMAT Instructor

Okay. Lots of questions.

Werewolf: I think I overemphasized this issue. A and B have tons of other problems, and it may be legit to say "four times more
likely." However, D is definitely wrong, because it can't be four times more likely with nothing to compare to grammatically.

I also want to address Ramana's issue, which was also sent to be privately by another student. The preferred form is "plan to VERB."
I don't know for certain is the correct answer choice here would be correct on the GMAT. Just know the preferred form, and that this
form is also possible. I doubt you'll ever be asked to make a direct choice between the two, with no other issues to look at.

-t

TommyWallach
Manhattan GMAT Instructor

I've done some searching, and I do think "as likely as" is definitely preferred, to the extent that it's legit to remove A and B for that
reason. Phew!

-t

Question 32
egmat
e-GMAT Representative

kinjiGC wrote:
According to a survey of graduating medical students conducted by the Association of American Medical Colleges, minority graduates are nearly four
times more likely than are other graduates in planning to practice in socioeconomically deprived areas.

(A) minority graduates are nearly four times more likely than are other graduates in planning to practice
(B) minority graduates are nearly four times more likely than other graduates who plan on practicing
(C) minority graduates are nearly four times as likely as other graduates to plan on practicing
(D) it is nearly four times more likely that minority graduates rather than other graduates will plan to practice
(E) it is nearly four times as likely for minority graduates than other graduates to plan to practice

According to a survey of graduating medical students conducted by the Association of American Medical Colleges,

minority graduates are nearly four times more likely than are other graduates in planning to practice in socioeconomically deprived areas.

Meaning : According to a survey, MG are nearly four times more likely than OG to plan on practicing Z

Option D) rather doesnt make sense. - Eliminated

Option E) to plan to practice there are two intents in the same sentence, making the sentence awkward Eliminated.

I am confused for Option A/B/C.

One of the rule I follow is more should have than. Both A and B satisfy that rule. e-gmat, can you please point out the mistake?

Question 32
Hi Kinjal,

Thanks for posting your doubt here.

Option A is incorrect because "likely" is not followed by "to verb". This word is always followed by a "to verb". For example: Kinjal is likely to
understand this explanation. However, in this choice what we have is "likely... in planning to practice". This is the incorrect idiom here. Now, the other
idiom "more... than..." is fine. But it has been out so cleverly between this "likely" idiom that we only focus on that. There is no problem with "four
times more likely" here.

Option B is also incorrect for the same reason. In fact, the "who clause" just provided additional information. The whole planning part now belongs to
the "other graduates" and do not even relate to "minority graduates" in the main clause.

Option C is the correct answer as it rectifies the idiom error in Choice A. The choice says "likely... to plan on practicing".

Hope this helps.


Thanks.
Shraddha

egmat
e-GMAT Representative
sk5002 wrote:
Hi Shraddha,

Can you please explain the role of underlined portion below in answer choice A? Also, this underlined portion isn't present in B, does that set the
comparison correctly(provided the error with who is fixed in 2nd choice)?

minority graduates are nearly four times more likely than are other graduates in planning to practice

Question 32
Thanks in advance

Hi sk5002,

In both Choice A and B, the comparison is logical and absolutely unambiguous. Presence or absence of "are" does not affect the comparison because
there is nothing else in the sentences with which "minority graduates" can be compared. the only logical entity in the sentence that can be compared to
"minority graduates" is "other graduates". Hence, presence or absence of "are" does not lead to any ambiguity and hence, its presence or absence is
OPTIONAL but NOT INCORRECT.

For example:

Ronny is a better bowler than Roy (is). --> In this sentence, we may or may not choose to repeat the helping Verb "is" because the comparison is
absolutely clear. he two compared entities in this sentence are "Ronny" and "Roy". When a sentence presents CLEAR COMPARISON, repeating the
helping Verb becomes OPTIONAL as we see in this official sentence.

Repeating the helping Verb becomes mandatory when the sentence conveys AMBIGUOUS COMPARISON. For example:

Ronny is familiar with Roy longer than Ria.

This sentence presents AMBIGUOUS COMPARISON because we can interpret two comparisons here:

1. Ronny is familiar with Roy longer than is Ria. --> Entities Compared - Ronny and Ria

2. Ronny is familiar with Roy longer than with Ria. --> Entities Compared - Roy and Ria

So, if the author intends to communicate the first meaning, then he MUST repeat the helping Verb "is". In absence of this helping verb, the sentence

Question 32
leads to ambiguous comparison. You can also read this article on Ellipses in Comparison for more clarity on this topic: how-far-ellipsis-is-permissible-
in-comparison-148973.html

Hope this helps.


Thanks.
SJ

egmat
e-GMAT Representative

sk5002 wrote:
Hi Shraddha,

Thanks for the wonderful explanation. I also read the article by you on ellipsis. But I'm kinda stuck on this comparsion issue and I think this example
should help me clarify-:

This one is from OG.

Original : Heating-oil prices are expected to be higher this year than last because refiners are paying about $5 a barrel more for crude oil than they
were last year.

Modified: Heating-oil prices are expected to be higher this year than those last yearbecause refiners are paying about $5 a barrel more for crude oil
than they were last year.

My Doubt: a) Is the bold and italic in the modified sentence a case of ellipsis when compared to original?
b) How do I know in this sentence or any for that matter what is being compared? The reason I ask this is I got confused whether original
sentence(also the correct one) compares the two time periods or the prices in two years?

Question 32
c) If it's time period than original makes sense but how do I clear this ambiguity while solving comparison questions

Thanks as always for you help. You are great!!

Hi sk5002,

The original sentence is:

Heating-oil prices are expected to be higher this year than last because refiners are paying about $5 a barrel more for crude oil than they were last year.

In this sentence, ellipsis is definitely at play. Let's rewrite the sentence with all the words that have been kept understood in the original:

Heating-oil prices are expected to be higher this year than heating oil prices last year because refiners are paying about $5 a barrel more for crude oil
than they were last year.

So now we know that the two compared entities are "heating oil prices this year" and "heating oil prices last year". So the two "heating oil prices" are
being compared.

Also, you can always ask, what is "higher" because "higher than" is the expression that tells you that there is a contrast in this sentence. So what's
higher? It is the heating oil prices and hence, that entity is being compared in this sentence.

The best way to know what's being compared is to understand the meaning of the sentence. So do pay attention to the original sentence to get the
intended logical comparison in the sentence.

Hope this helps.


Thanks.
SJ

Question 32
egmat
e-GMAT Representative

sk5002 wrote:
Hi Shraddha,

You are the BEST!!! Thanks so much for the prompt reply and I'm sorry for bothering you again with this.
I just have couple of additional questions.

1) In the complete statement as pointed out by you, will there be no "are/were"? As in --

Heating-oil prices are expected to be higher this year than Heating oil prices were last year because refiners are paying about $5 a barrel more for
crude oil than they were last year.

If there will be verb form, then is it safe to assume that we can remove the verb form in ellipsis? Can you please explain this particular case with more
examples.

2) Since prices are being compared, will the following sentence be correct?

Heating-oil prices are expected to be higher this year than those last year because refiners are paying about $5 a barrel more for crude oil than they
were last year.

And, if yes, then why is the below statement wrong? Is it because that its not clear what last year's refer to??

Heating-oil prices are expected to be higher this year than last year's because refiners are paying about $5 a barrel more for crude oil than they were
last year.

Thanks!!!

Question 32
Hi sk5002,

1) I don't think it will be correct to put "were" in the statement before "last year" because the meaning conveyed might be "was expected to be". This is
so because the helping verb "are" is associated with the expectation of the rise in the price. But we want to compare this year's heating price with last
year's price.

2) Heating-oil prices are expected to be higher this year than those last year because refiners are paying about $5 a barrel more for crude oil than they
were last year. --> This statement is correct as "those" stand for "heating oi prices".

The second statement with "last year's" does not work because the two compared entities can be written in identical parallel forms as written in my
earlier post. Hence, "last year's" does not work with "this year".

Hope this helps.


Thanks.
SJ

sayantanc2k
Verbal Expert

ArunpriyanJ wrote:

As far as i have understood 'More likely than' is not a phrase......But 'as likely as' is a correct phrase.

Question 32
experts kindly clarfiy

Thanks,
A

ArunpriyanJ

Gramatically, Both X times more ... than and X times as...as are correct, but they mean different.

Case 1: more...than

I have 5 chocolates more than you have.


Implication: You have x chocolates, Difference is 5. So I have x+5 chocolates

I have 5 times more chocolates than you have.


Implication: You have x chocolates. Difference is 5 times your chocolate, i.e. 5x. So I have x + 5x = 6x chocolates

Case 2: as...as

I have 5 times as many chocolates as you have.


Implication: You have x chocolates. I have 5x chocolates.

In summary: the phrase more.. than involves the mathematical operation addition, whereas the phrase as...as involves the
mathematical operation multiplication.

The error happens frequently when we want to mean that I have 5x chocolates but say I have 5 times more chocolates. When I say I
have 5 times more chocolates, the sentence means I have 6 times as many chocolates as explained above !!

Question 32
Does this explanation make sense?

sayantanc2k
Verbal Expert

sauravpaul wrote:
sayantanc2k wrote:
Gramatically, Both X times more ... than and X times as...as are correct, but they mean different.

In that sense, isn't the correct answer (option C) changing the meaning of the original sentence?

sauravpaul

Yes, the choices A and C have different meanings. Nonetheless, there are major grammatical problems in the other options:

A. The idiom likely in planning is wrong; the correct usage is likely to plan.
B. The relative clause modifier who plan on practicing.....in socioeconomically deprived areas refers to other graduates, but after the clause ends the
main clause should continue to express what the minority graduates are more likely to do. The main clause in the sentence takes the form: minority
graduates are nearly four times more likely than other graduates - incomplete construction.
D and E are meaningless altogether.

Therefore although Option C changes the meaning of the original sentence, we have to accept it since it is the only grammatically correct option; all
other options have severe errors. [....and in the process we assume that the author originally meant 4x and not +4x. ]

Question 32
In general, please note the following:
When multiplication is meant : use X times as
When addition is meant: use more than.

sayantanc2k
Verbal Expert

sauravpaul wrote:
ok thanks. This is another issue that is quite perplexing....that I keep running into some inconsistent information.

I remember reading a post from another expert that we should preserve the meaning of the original sentence. In fact, in that article, the expert cited
few questions where an option was incorrect because it seemed to change the meaning of the original sentence.

However, here we have a question where the correct answer seems to quite clearly change the meaning of the original sentence. Quite perplexing.

sauravpaul

Don't get perplexed. Keep your thoughts straight: If you have 2 grammatically correct options, choose the one that does not change the meaning of the
original sentence. However if you have only one grammatically correct option, you must choose that option.

sayantanc2k
Verbal Expert

Question 32
sgrover18 wrote:
TommyWallach wrote:
Hey All,

I was asked by private message to take this one on, even though there has been plenty of great stuff written already. Here we go:

This is a comparison question, so the whole point is to make sure we're comparing the right two things, and using the correct terminology.

42. According to a survey of graduating medical students conducted by the Association of American Medical Colleges, minority graduates are
nearly four times more likely than are other graduates in planning to practice in socioeconomically deprived areas.
(A) minority graduates are nearly four times more likely than are other graduates in planning to practice
PROBLEM: Should be "four times AS likely AS other graduates TO plan". That's a three-fer!

(B) minority graduates are nearly four times more likely than other graduates who plan on practicing
PROBLEM: Again "four times AS likely AS other graduates TO plan". This one doesn't ever complete, because the "who" opens up a new
modifier, and we never return to the main clause.

(C) minority graduates are nearly four times as likely as other graduates to plan on practicing
CORRECT.

(D) it is nearly four times more likely that minority graduates rather than other graduates will plan to practice
PROBLEM: "Four times AS likely" and RATHER THAN implies preference, which makes no sense here. Also, it's totally unclear what's being
compared.

(E) it is nearly four times as likely for minority graduates than other graduates to plan to practice
PROBLEM: "for minority graduates than other graduates" makes absolutely no sense at all. Where's the comparison?

Hope that helps!

-t

Question 32
Hi,

I understand the "AS likely AS X TO" construction and why C is the best option. However is "more LIKELY than X TO" a correct construction as
well ? ( Which is not a part of the options right now)

For multiplication, "as .... as" is used.


For addition, "more.... than" is used.

The reason is as follows:

Consider the following statement:


I have 5 more books than you have.
This implies that the difference between my books and your books is 5. If you have x books, I have x+5 books.

Now take this statement:


I have 5 times more books than you have.
This implies that the difference between my books and your books is 5x. If you have x books, I have x + 5x books, i.e. 6x books. Thus if the meaning
intended is that I have 5x books, then the sentence is wrong. The correct sentence then would be:

I have 5 times as many books as you do.

sayantanc2k
Verbal Expert

Question 32
nishant12600 wrote:

Hello TommyWallach

Can u please elaborate on the usage of "as likely as" and " more likely than". Is there any circumstance in which both can be used
interchangeably?

No, they are not interchangeable.

X is as likely as Y... implies the likelihood of X = the likelihood of Y


X is more likely than Y... implies likelihood of X > the likelihood of Y

http://www.beatthegmat.com/sc-more-likely-than-as-likely-as-t25931.html

David@VeritasPrep GMAT Instructor

I just wanted to respond to something that Logitech wrote clear back in 2008!

That is eliminating answer choices based on "the original meaning" and a "change in meaning."

There is no "original meaning." Answer choice A is not special. It is just another option. The reason to eliminate choices is because
they are ILLOGICAL or UNGRAMMATICAL.

Question 32
Here is an article I wrote about this some time ago. http://www.beatthegmat.com/the-truth-about-changing-the-meaning-in-
sentence-correction-t76648.html

https://www.manhattanprep.com/gmat/forums/according-to-a-survey-of-graduating-medical-students-conduct-t5501.html

RonPurewal
ManhattanGMAT Staff

Re: According to a survey of graduating medical students con

Fri Nov 28, 2008 8:19 am

Jamie wrote:1. Could you please clarify the difference bt "four times more likely than" and "four times as likely as"?

replicated from this post. this post addresses more than just your question, but your answer is in there:

heh.

mathematically speaking, there's a difference between '4 times more likely' and '4 times as likely'. Specifically, '4 times more likely'
is actually the same as '5 times as likely', although even experts accidentally conflate the two constructions on occasion.

but this is a verbal question, so let's set the mathematical nitpicking aside; there is no language-based reason to prefer one or the
other of these constructions.

Question 32
the biggest problem with D is its poor idiomatic construction. you don't say 'it is X times MORE likely that A will happen, RATHER
THAN B'; 'more' is supposed to go with 'THAN', and is incompatible with 'rather than'. the proper construction would be 'it is X times
more likely that A will happen than that B will happen.' better than either of these, though, is the more compact form: 'A is X times
more likely to happen than is B.'

hth.

2. Is "plan to" a wrong idiom? is "plan on" the correct idiom?

no, "plan to" is fine.


but you missed the idiom that actually _is_ incorrect in this sentence: "likely ... in planning" (incorrect, in (a)) vs. "likely ... to plan"
(correct, in (b)).
"likely" must be used with an infinitive. in their usual dastardly way, the gmat writers have camouflaged this poor idiomatic usage
behind not only lots of noise (the words between "likely" and "in"), but also a second, correct idiom (the one you singled out).
that's tough.

JonathanSchneider
ManhattanGMAT Staff

Re: According to a survey of graduating medical students conduct

Fri May 22, 2009 1:34 am

I'm not entirely clear what you're asking. Here's what I can tell you about B, though, and I hope it answers your question:

By converting to the word "who," we place all of the following information as a modifier of the second word "graduates," as you say.
This causes a major problem, though, as we never finish the preceding clause: "more likely ..." to what? You cannot just say: "These
people are more likely." You can call an outcome more likely. But to call people more likely does not work; we need to know what
they are more likely to do.

Question 32
Does that help?

RonPurewal
ManhattanGMAT Staff

Re: According to a survey of graduating medical students conduct

Thu Aug 27, 2009 1:56 am

NIKESH_PAHUJA wrote:Thanks for nice explanation.

i had one query.

Would option A be right, if it were written as follows :

minority graduates are nearly four times more likely than are other graduates to plan on practicing....

Or following will be correct ?

minority graduates are nearly four times more likely than other graduates to plan on practicing....

Question 32
I have just removed .....are ......from second choice.......Just wanted to know......which comparison is correct ?

they're both correct, but the one without the helping verb is a little better (because it's more concise).

in general, you shouldn't include such helping verbs, in parallel constructions unless they're NECESSARY.
pretty much the only way in which one of these helping verbs can be necessary is if it gets rid of ambiguity.

for instance:
i know more about shakespeare than my brother
is incorrect, because we don't know whether this means (1) my knowledge of shakespeare is superior to my brother's, or (2) i know
more about shakespeare than i know about my own brother.

if you change this to


i know more about shakespeare than does my brother
then the ambiguity is removed.

if you have
i am taller than my brother
then you don't need "...is my brother", since there's no ambiguity to start with.

in your example, there's no such ambiguity in the first place, so you don't need the helping verb.
although it's certainly not wrong to include that verb.

Question 32
RonPurewal
ManhattanGMAT Staff

Re: According to a survey of graduating medical students con

Tue Mar 30, 2010 7:45 am

victorgsiu wrote:Ron,

Excellent post.

thanks.

Regarding your point, "A is X times more likely to happen than is B.", do we need the second "is"?

I ask because the correct answer choice C omits the second "are", i.e., correct choice C: "minority graduates are nearly four times
as likely as [are] other graduates to plan on practicing"

Should the take-aways then be:


1. If I see "more X than ", then I should have verbs on both sides.
2. If I see "as X as", then I should not repeat the verb.

nah. wrong issue.

here's the takeaway:


you only need the 2nd helping verb if the sentence is ambiguous without it.

here's an example:

Question 32
james is more likely to meet with thomas than lydia
--> unacceptable, because there are two meanings.
in this case, you have to add a helping word to dispel the ambiguity:
james is more likely to meet with thomas than is lydia (meaning #1)
james is more likely to meet with thomas than with lydia (meaning #2)

but
james is more likely to drive to work than lydia
you don't need "is" here, since this sentence can only mean one thing. (it's not wrong to include "is" -- just not necessary.)

RonPurewal
ManhattanGMAT Staff

Re: According to a survey of graduating medical students conduct

Mon Feb 28, 2011 6:19 am

ankitp wrote:Ron,

Why is E wrong, it has the correct idiom, and "plan to " is correct, its because it sound awkward?

Thanks

the big, huge, colossal error in choice (c) is its pairing of "four times as likely" with "than".

Question 32
that's bad -- it's "four times AS likely AS"; you can't say "as likely than".

(note: in the choices that say "four times more likely", that should be followed by "than".)

--

also, apparently we shouldn't be concerned with mathematical issues -- technically, "four times as likely as X" is 4X, while "four
times MORE likely than X" is actually 5X. apparently, this is not a big deal on the verbal section.

shengfangqiji33
Forum Guests

Re: According to a survey of graduating medical students conduct

Mon Oct 10, 2011 11:19 am

i had two questions here.

i have known that the sentence below is correct

minority graduates are nearly four times more likely than are other graduates to plan on practicing....

but if i complete the sentence above.is it right?

Question 32
minority graduates are to plan on practicing....
nearly four times more likely than are other graduates (to plan on practicing....)

I have just added "to plan on practicing "at the first part of the sentence and " likely " at the second of the sentence.Just wanted to
know
is the sentence i complete right???

in addition, i have a question about the sentence below.

minority graduates are to plan on practicing ....


nearly four times more likely than are other graduates....

is the helpng verb "are" necessary in the second part of the sentence? is the phrase "to plan " be the object of the first part of the
sentence.

i'll be appreciate if someone can help me.

RonPurewal

ManhattanGMAT Staff

Posts: 19507

Question 32
Joined: Tue Aug 14, 2007 8:23 am

Re: According to a survey of graduating medical students conduct


RonPurewal
ManhattanGMAT Staff

Fri Oct 28, 2011 7:15 am

shengfangqiji33 wrote:but if i complete the sentence above.is it right?

i think you're confusing "above" and "below".

minority graduates are to plan on practicing....


nearly four times more likely than are other graduates (to plan on practicing....)

i'm not sure why you are trying to break up the sentence like this, but, no, this is not a legitimate way to break up the sentence.

if you have "N times as ADJ/ADV...", then the adjective/adverb is a fundamental part of the sentence; you can't take it out. for
instance, the sentence I am twice as tall as my cousin is based on the simpler sentence I am tall; you can't take the "tall" out of this
sentence.

on another note, your attempt to break up the sentence in this way suggests that you aren't thinking about what it means; it's
almost as though you're treating it as some sort of mathematical equation with meaningless parts. not good! you always have to
think about what the sentence means. here, if you do, you'll realize that this is not a legitimate transformation.

Question 32
minority graduates are to plan on practicing ....
nearly four times more likely than are other graduates....

is the helpng verb "are" necessary in the second part of the sentence? is the phrase "to plan " be the object of the first part of the
sentence.

i'm sorry, but i am lost here. what are the "first part" and "second part" that you're referring to here?
are you referring to the first and second parts of the actual, correct sentence?
or are you referring to the two parts you wrote above (which aren't legitimate constructions)?

i'll be appreciate if someone can help me.[/quote]

ghong14
Course Students

Question 32
Re: According to a survey of graduating medical students conduct

Thu Jul 04, 2013 8:57 pm

If we ignore the idiom issue "to Plan" in answer choice A isn't the construction four times as likely as preferred over four times more
likely? I thought that choice A was wrong because of the later issue as well.

tim
ManhattanGMAT Staff

Re: According to a survey of graduating medical students conduct

Sat Jul 06, 2013 1:49 pm

The issue is not what is "preferred", but whether either of those constructions is wrong. And there is no reason to reject either of
those constructions on its own. Remember, sentence correction is not about figuring out what is preferred or correct, but is instead
about finding four wrong answers.

aliag916
Course Students

Re: According to a survey of graduating medical students conduct

Question 32
Sun Oct 20, 2013 3:12 am

Hello, In terms of Sentence Structure please tell me if my analysis is okay ?


The subject, verb and object are as follows:
a. graduates, are and "to practice"; so this is a sentence.
c. graduates, are and "to plan" ; so this is a sentence.
d. It, is and "to practice" ; so this is a sentence.
e. It, is and "to practice" ; so this is a sentence.

Also, can I use this to narrow down the choices and then focus on whats wrong coz i may not recall all the idioms etc....?
Thank you!

RonPurewal
ManhattanGMAT Staff

Re: According to a survey of graduating medical students conduct

Mon Oct 21, 2013 8:33 am

There aren't any issues of overall sentence structure here.

aliag916 wrote:Also, can I use this to narrow down the choices and then focus on whats wrong coz i may not recall all the idioms
etc....?
Thank you!

On this problem... well, no.


As you wrote yourself, all of the choices are complete sentences, so you can't eliminate anything in this problem for that reason.

On other problems, sure -- if there are actually sentence structure issues, then, if you're comfortable doing so, you can use that as a

Question 32
first elimination.

Haibara
Forum Guests

Re: According to a survey of graduating medical students conduct

Wed Jan 29, 2014 4:39 am

RonPurewal wrote:the biggest problem with D is its poor idiomatic construction. you don't say 'it is X times MORE
likely that A will happen, RATHER THAN B'; 'more' is supposed to go with 'THAN', and is incompatible with 'rather
than'. the proper construction would be 'it is X times more likely that A will happen than that B will happen.'

Ron or other experts:


let me rephrase it:

It is nearly four times more likely that minority graduates will plan to practice... than that other graduates will.

The above sentence is correct, right? I want to ask whether "that" and "will" could be left out? I mean the following:

It is nearly four times more likely that minority graduates will plan to practice... than other graduates.

If the above sentence is correct, then I see the only problem that makes D incorrect is the use of "rather" before than.
If the above sentence is incorrect, then I guess the problem lies in parallelism. Because a sentence can not be parallel to a noun
phrase. However as Ron said:

RonPurewal wrote:in general, you shouldn't include such helping verbs, in parallel constructions unless they're NECESSARY.

Question 32
pretty much the only way in which one of these helping verbs can be necessary is if it gets rid of ambiguity.

Since there isn't any ambiguity without "that" and "will", I think the second sentence could also be correct.

What's the problem with my thinking ? Thanks~

RonPurewal
ManhattanGMAT Staff

Re: According to a survey of graduating medical students conduct

Wed Jan 29, 2014 11:51 am

Haibara wrote:It is nearly four times more likely that minority graduates will plan to practice... than that other
graduates will.

The above sentence is correct, right?

Yes.

I want to ask whether "that" and "will" could be left out? I mean the following:

It is nearly four times more likely that minority graduates will plan to practice... than other graduates.

Question 32
This sentence doesn't work.

If the second part of the comparison is just "other graduates", then the first part clearly must be a noun; that noun can work in one
of two ways.

1/
It can appear directly next to "than".
Harvard graduates who start companies are more likely to hire strangers than other graduates.

2/
It can be the subject of "is/are more likely".
Engineering graduates are more likely to earn high starting salaries than other graduates.

In case #2, the sentence would be better written with the comparison in one continuous piece (Engineering graduates are more
likely than other graduates to earn high starting salaries), although the current #2 is not strictly wrong.

--

In the sentence here, "4 times more likely" is buried in the middle of the clause It is ... that xxxx will happen, so the other half needs
to be an entire statement that will parallel the statement that xxx will happen.

RonPurewal
ManhattanGMAT Staff

Question 32
Re: According to a survey of graduating medical students conduct

Fri Jan 31, 2014 6:57 am

Haibara wrote:Ron, you hit the nail on the head. What makes the sentence of comparison more complicated is
exactly the "It is ... that xxxx will happen" structure.

Yeah, but that inversion is what makes the sentence actually readable.

If you're the one who created the sentence " or if you're specifically prepared to see "That xxxxxx happens" as a noun " then,
sure, it's not very hard to interpret those sentences correctly. But, if you're just reading a random block of text one day and
something like that appears ... it's just too hard to read.

From your explanation, can I consider it in the following way?

It is nearly four times more likely that minority graduates will plan to practice... than that other graduates will.

Since "It" refers to "that minority graduates will plan to practice... ", I reorganise it to :

(That minority graduates will plan to practice... ) is nearly four times more likely than (that other graduates will).

This sentence is technically correct, of course. However, a reader who is not specifically expecting this kind of construction would
have to read it multiple times to interpret it correctly.

Question 32
The only reason it's even easy for me to read this sentence is that you've conveniently highlighted the two parallel parts. If you take
the highlighting away, I'll have to read it three or four times (or more), even though we're discussing that construction!

Fortunately, you don't have to construct sentences on this exam; you just have to recognize when things are ok and when they
aren't ok.
(There's a hint here: "Reorganizing" the sentences like this is, for the purpose of GMAT preparation, a waste of your time. It may
make you a more conscientious writer " a noble goal indeed " but that's not the point of this forum.)

RonPurewal
ManhattanGMAT Staff

Re: According to a survey of graduating medical students conduct

Fri Jan 31, 2014 6:57 am

I know the above sentence is especially weird, but in this pattern, I could then clearly tell that the second part of
comparison should be in the form of a sentence, rather than a noun phrase or a prepositional phrase.

You lost me at "noun phrase"; I don't know (or remember) what that is.
This may be part of the problem, though " by being concerned with classifying things, you may be diverting brainpower away from
looking at the two things and seeing whether they form a valid comparison.

Question 32
RonPurewal
ManhattanGMAT Staff

Re: According to a survey of graduating medical students conduct

Fri Jan 31, 2014 6:59 am

Thus, the following sentence(I just made it up randomly. It has nothing to do with the original prompt.) is incorrect :

It is nearly four times more likely that minority graduates will plan to practice in socioeconomically deprived
areas than to work for multinational pharmaceutical corporations.

The above sentence is incorrect, right? Although it looks very parallel.

No. This sentence is fine, for exactly the same reason as "reason #1" listed in my post above (link: post95689.html#p95689 ).
You've got the two parallel constructions flanking "than", so you're good.

In many cases, that's the best you can possibly do.


E.g., In New York City, more people walk than drive to work. --> "Walk" and "drive" are flanking "than", so you're good. (There's
obviously no way to get "more" directly in front of "walk".)

RonPurewal
ManhattanGMAT Staff

Question 32
Re: According to a survey of graduating medical students conduct

Fri Jan 31, 2014 6:59 am

However, if I remove the "It is ... that xxxx will happen" structure from it:

Minority graduates are nearly four times more likely to plan to practice in socioeconomically deprived areas
than to work for multinational pharmaceutical corporations.

This is still exactly the same situation as the previous example.

If you thought the previous example was wrong, then you should think this one is wrong too.
I.e., it appears you thought the previous example was wrong because, as you saw it, "that minority graduates will plan" interfered
with the construction.
The problem there is that you'd have exactly the same kind of interference with "to plan" here.

Both constructions are fine. Both are also optimal " i.e., there's really no feasible way to get the requisite stuff any closer together.

Even you have tacitly acknowledged this point, because you actually changed the wording " and, as a result, the meaning " of the
sentence to get this second version.
The change in meaning isn't so great, but it's there: the first sentence talks about the current likelihood of certain people forming
certain future plans ("will plan"), whereas the second talks about present plans. Not nearly a big enough difference to be tested on
the exam, but a difference nonetheless.

Again, both sentences are fine " for exactly the same reason, and for the same reason as noun form #1 above.

Question 32
Haibara

Forum Guests

Re: According to a survey of graduating medical students conduct

Fri Jan 31, 2014 11:25 am

RonPurewal wrote:

Thus, the following sentence(I just made it up randomly. It has nothing to do with the original prompt.) is
incorrect :

It is nearly four times more likely that minority graduates will plan to practice in socioeconomically
deprived areas than to work for multinational pharmaceutical corporations.

The above sentence is incorrect, right? Although it looks very parallel.

No. This sentence is fine, for exactly the same reason as "reason #1" listed in my post above (link:
post95689.html#p95689 ).
You've got the two parallel constructions flanking "than", so you're good.

In many cases, that's the best you can possibly do.


E.g., In New York City, more people walk than drive to work. --> "Walk" and "drive" are flanking "than", so you're
good. (There's obviously no way to get "more" directly in front of "walk".)

Question 32
RonI'm so grateful for your kind and patient reply to my long post.
Many thanks.

After reading all your elaboration three times, I seem to grasp the main thrust here, which is "flanking".

In New York City, more people walk than drive to work.


--> "Walk" and "drive" are flanking "than" ---->correct

Harvard graduates who start companies are more likely to hire strangers than other graduates.
"strangers" and "other graduates" are flanking "than"--->correct

Engineering graduates are more likely to earn high starting salaries than other graduates.

As you said, it can be reorganised to a better form with the comparison in one continuous piece:

Engineering graduates are more likely than other graduates to earn high starting salaries.
--->So "Engineering graduates" and "other graduates" are flanking "more likely than"---->correct

However, if I eliminate "rather" from choice D:


It is nearly four times more likely that minority graduates than other graduates will plan to practice...

Even though "minority graduates" and "other graduates" are flanking "than", the above sentence is still incorrect, because the
flanking part ---"minority graduates than other graduates"--- is somehow stuck in the "middle " of the sentence?

In contrast, the flanking part "Engineering graduates are more likely than other graduates" is put at the beginning of the
corresponding sentence. The flanking part "strangers and other graduates" is put at the end of the corresponding sentence.

Therefore, I guess, for the position of comparison of two nouns, the 'beginning' and the 'end' are fine, but the 'middle' is not allowed,

Question 32
right?

Also, if I change choice E into:


It is nearly four times as likely for minority graduates as for other graduates to plan to practice...

The above sentence is correct,right? "for minority graduates" and "for other graduates" are flanking "as", and the flanking part
is stuck in the 'middle' of the sentence, but the sentence is correct because the comparison target ('for + NOUN') in the flanking
part is a prepositional phrase that could be put in the 'middle' of a sentence?

Sorry for bothering you with this comparison issue for so long time.
I'm really trying to find a rule that could be engraved in my mind, so I'm well prepared for other similar questions of comparison
when I meet them later in the real test.

jlucero
ManhattanGMAT Staff


In contrast, the flanking part "Engineering graduates are more likely than other graduates" is put at the beginning of the
corresponding sentence. The flanking part "strangers and other graduates" is put at the end of the corresponding sentence.

Therefore, I guess, for the position of comparison of two nouns, the 'beginning' and the 'end' are fine, but the 'middle' is not
allowed, right?

Not exactly. Most often, nouns come at the beginning or end of a sentence, so in most cases, you would see the comparison happen

Question 32
there. But you could also see an example like:

For breakfast, Joe likes to eat omelets rather than oatmeal, and drink milk.

As Ron said in his first post, the biggest issue in D is the unidiomatic construction of D. We all understand what D is trying to say,
but it's just not the way that the

English language says it.

Haibara wrote:Also, if I change choice E into:


It is nearly four times as likely for minority graduates as for other graduates to plan to practice...

The above sentence is correct,right? "for minority graduates" and "for other graduates" are flanking "as", and the flanking
part is stuck in the 'middle' of the sentence, but the sentence is correct because the comparison target ('for + NOUN') in the
flanking part is a prepositional phrase that could be put in the 'middle' of a sentence?

Sorry for bothering you with this comparison issue for so long time.
I'm really trying to find a rule that could be engraved in my mind, so I'm well prepared for other similar questions of comparison
when I meet them later in the real test.

This would be incorrect, again, because of the unidiomatic style of setting this up. In my opinion, there are two correct ways to write
this sentence (comparisons in quotes):

"X" is 4 times as likely as "Y" to do something.


or
It is 4 times as likely that "X does something", as it is that "Y does that same thing".

Answer choice C goes with the first (and simpler) construction- compares nouns to nouns. Answer choices D & E both attempt the
second construction, but fail to compare what X DOES with what Y DOES. If you wanted to use that construction properly, you would
need a sentence like:

Question 32
it is nearly four times more likely that minority graduates will plan to practice in socioeconomically deprived areas, than other
graduates will plan to.

RonPurewal
ManhattanGMAT Staff

Re: According to a survey of graduating medical students conduct

Tue Feb 04, 2014 6:55 am

Haibara wrote:"It is nearly four times as likely for minority graduates as for other graduates to plan to practice..

"Likely for (someone) to (verb)" isn't a thing. Once you've seen that, you've seen that; the issue has nothing to do with the
comparison.

" and "It is nearly four times more likely that minority graduates than other graduates will plan to practice...", except
memorise them as incorrect expressions one by one.

If the sentence starts out with "It is more likely that...", then you can't break up the clause starting with "that", just as you wouldn't
want to break up any other construction that is one of the two things being compared.
Unlike the idiom above, this one requires no knowledge beyond "don't break up stuff that shouldn't be broken up".

Haibara

Question 32
Forum Guests

Thank you Ron very much, I now get it.

RonPurewal

ManhattanGMAT Staff

Re: According to a survey of graduating medical students conduct

Sat Aug 23, 2014 8:41 am

RomanN658 wrote:Hi Ron,

I eliminated (C) and choose (A) because I thought (C) changes the original meaning of the sentence. In my
perspective, "four times more likely" and "four times as likely as" convey entirely different meaning. However, since
you mentioned that the difference is only a mathematical problem, Could you please explain from a native speaker
perspective why this difference is not a meaning issue?

"Meaning issues" only exist when something is actually nonsense.

If you are choosing between two or more reasonable meanings, then either one is fine. (In particular, the meaning of choice A is
not sacrosanct or "preferred" in any way.)

Question 32
RonPurewal
ManhattanGMAT Staff

Re: According to a survey of graduating medical students con

Sat Jun 06, 2015 4:36 am

RajatG730 wrote:Is PLAN ON a correct idiom ?

it's in the correct choice, so, you already know the answer to this question.

RonPurewal
ManhattanGMAT Staff

Re: According to a survey of graduating medical students conduct

Wed Jul 01, 2015 6:12 am

if you come upon this sort of impasse, try to make shorter sentences, with easier words, that contain the same difference.

e.g.,
My team is more likely ____ than your team. (or, My team is more likely than your team _____.)
to win?
in winning?
hopefully this one isn't too hard. if it's not, then you know everything you need to know.

Question 32
RonPurewal
ManhattanGMAT Staff

Re: According to a survey of graduating medical students conduct

Wed Jul 01, 2015 6:12 am

incidentally, a correct sentence can contain both 'likely' and 'in X'... but only as separate constructions.

e.g.,
Car accidents are especially likely in bad weather.

note that this sentence could just as well be written as In bad weather, car accidents are especially likely.
we're NOT saying that accidents are likely to do anything; we're just saying that they are likely. (it would clearly be nonsense to
say that a PERSON 'is likely'. therefore, choice A is nonsense.)

(and of course you can have all three: Drivers are especially likely to have accidents in bad weather.)

RonPurewal
ManhattanGMAT Staff

Re: According to a survey of graduating medical students conduct

Sun Jan 24, 2016 5:42 pm

in general, please try to avoid "To recap..." posts. by definition, such posts add zero value, and just clutter the discussion threads.

Question 32
the point is to read through the thread, in its entirety, and resolve as many issues as you can in so doing. then, ask any outstanding
questions.

RonPurewal
ManhattanGMAT Staff

Re: According to a survey of graduating medical students conduct

Sun Jan 24, 2016 5:42 pm

JacobW468 wrote:that "likely" takes the infinitive

^^ don't know this terminology, so, i don't know.


but, the correct/incorrect use of "likely" is discussed in the thread.

"as X as" is the correct form for comparison (eliminating E)

yes.

RonPurewal
ManhattanGMAT Staff

Question 32
Re: According to a survey of graduating medical students conduct

Sun Jan 24, 2016 5:42 pm

The beginning part of the underline is a false split

i don't know what this is supposed to mean.

if you're saying that you shouldn't compare "minority graduates" and "it"... well, of course you shouldn't! you should compare the
things that actually correspond to each other.

i suspect you're saying something else, since "you shouldn't compare minority graduates and it" is obvious. so, please clarify.

and this isn't a "like" vs "as" issue.

well... no, it isn't. why would you even be thinking about this?
considering that the word "like" doesn't appear anywhere in the choices?

evelynho
Students

Re: According to a survey of graduating medical students conduct

Question 32
Sat Jan 30, 2016 8:03 am

Hi Instructors,
Just wonder if I can make the go-through process simpler.

"Four times" already means the extent of comparison by multiplication, while if it is followed by "more than", it could be redundant
or worse than "four time as likely as". So eliminate AB.
D, "rather than" means the alternative, a kind of replacement, then it distorts the original meaning.Not D.
E, as Ron mentioned, "as likely" and "than" after "four times" is redundant. Not E.
Then it should be C. Please comment.
Best Regards,

RonPurewal
ManhattanGMAT Staff

Re: According to a survey of graduating medical students conduct

Tue Feb 09, 2016 9:23 am

evelynho wrote:Hi Instructors,


Just wonder if I can make the go-through process simpler.

first, please do not do this ^^. we do not want discussion threads to be followed by X number of "okay let me try to summarize this
whole thing" posts.
please post only things that add value to the discussion already in the thread. thank you.

RonPurewal

Question 32
ManhattanGMAT Staff

Re: According to a survey of graduating medical students conduct

Tue Feb 09, 2016 9:26 am

"Four times" already means the extent of comparison by multiplication, while if it is followed by "more than", it could
be redundant or worse than "four time as likely as". So eliminate AB.

^^ no.

"4 times as [adjective] as..." and "4 times more [adjective] than..." are both valid constructions.
they're mathematically distinct, but that's not relevant in SC. neither one is "better" than the other.

this is already covered on page 1 of the discussion thread, where i wrote "there is no reason to prefer one of these constructions
over the other". 3rd post on page 1.
please read the ENTIRE discussion thread before posting. thank you.

DungN738
Forum Guests

Re: According to a survey of graduating medical students conduct

Question 32
Mon May 02, 2016 11:17 pm

RonPurewal wrote:

Haibara wrote:"It is nearly four times as likely for minority graduates as for other graduates to plan to
practice...

"Likely for (someone) to (verb)" isn't a thing. Once you've seen that, you've seen that; the issue has nothing to do
with the comparison.

" and "It is nearly four times more likely that minority graduates than other graduates will plan to
practice...", except memorise them as incorrect expressions one by one.

If the sentence starts out with "It is more likely that...", then you can't break up the clause starting with "that", just
as you wouldn't want to break up any other construction that is one of the two things being compared.
Unlike the idiom above, this one requires no knowledge beyond "don't break up stuff that shouldn't be broken up".

This is my first post here.

Ron, could you kindly elaborate on the part that I colored in red. I don't seem to understand what you were trying to say, although I
have read through all the posts in this thread.

Thanks tons.

RonPurewal
ManhattanGMAT Staff

Question 32
Re: According to a survey of graduating medical students conduct

Fri May 06, 2016 7:55 am

that just isn't idiomatic.

chetan86

Students

Re: According to a survey of graduating medical students conduct

Sun May 22, 2016 7:20 am

calm.jing wrote:

RonPurewal wrote:If the second part of the comparison is just "other graduates", then the first part clearly
must be a noun; that noun can work in one of two ways.

1/
It can appear directly next to "than".
Harvard graduates who start companies are more likely to hire strangers than other graduates.

2/
It can be the subject of "is/are more likely".

Question 32
Engineering graduates are more likely to earn high starting salaries than other graduates.

Hi Ron,

I am confused. I thought sentence #1 is ambiguous, because "other graduates" can be compared to either
"strangers" or "Harvard graduates". Is my thinking correct? Please clarify.

Thanks in advance!

Hi Ron,

It seems you missed this post. I also have the same question. Could you please reply?

Thanks!

RonPurewal
ManhattanGMAT Staff

Re: According to a survey of graduating medical students conduct

Wed May 25, 2016 3:31 am

no, because "other graduates" = other harvard graduates. if you interpret the sentence in your alternative way, you get "harvard
graduates are more likely to hire strangers than other harvard graduates are", which makes no sense.

if you mean "graduates of other schools", then you'd have to write "graduates of other schools", or "other schools' graduates".
...and even then you'd still have a problem, because the first half of the comparison is harvard graduates who start companies --

Question 32
which can't meaningfully be compared to graduates (in general) of other schools.

Question 32
Re: According to a survey of graduating medical students conduct

Tue Jun 28, 2016 3:21 am

jlucero wrote:

Haibara wrote:RonI'm so grateful for your kind and patient reply to my long post.
Many thanks.

After reading all your elaboration three times, I seem to grasp the main thrust here, which is "flanking".

In New York City, more people walk than drive to work.


--> "Walk" and "drive" are flanking "than" ---->correct

Harvard graduates who start companies are more likely to hire strangers than other graduates.
"strangers" and "other graduates" are flanking "than"--->correct

Engineering graduates are more likely to earn high starting salaries than other graduates.

As you said, it can be reorganised to a better form with the comparison in one continuous piece:

Engineering graduates are more likely than other graduates to earn high starting salaries.
--->So "Engineering graduates" and "other graduates" are flanking "more likely than"---->correct

However, if I eliminate "rather" from choice D:


It is nearly four times more likely that minority graduates than other graduates will plan to practice...

Even though "minority graduates" and "other graduates" are flanking "than", the above sentence is still
incorrect, because the flanking part ---"minority graduates than other graduates"--- is somehow stuck in

Question 32
the "middle " of the sentence?

In contrast, the flanking part "Engineering graduates are more likely than other graduates" is put at the
beginning of the corresponding sentence. The flanking part "strangers and other graduates" is put at the
end of the corresponding sentence.

Therefore, I guess, for the position of comparison of two nouns, the 'beginning' and the 'end' are fine, but
the 'middle' is not allowed, right?

Not exactly. Most often, nouns come at the beginning or end of a sentence, so in most cases, you would see the
comparison happen there. But you could also see an example like:

For breakfast, Joe likes to eat omelets rather than oatmeal, and drink milk.

As Ron said in his first post, the biggest issue in D is the unidiomatic construction of D. We all understand what D is
trying to say, but it's just not the way that the English language says it.

Haibara wrote:Also, if I change choice E into:


It is nearly four times as likely for minority graduates as for other graduates to plan to practice...

The above sentence is correct,right? "for minority graduates" and "for other graduates" are flanking
"as", and the flanking part is stuck in the 'middle' of the sentence, but the sentence is correct because the
comparison target ('for + NOUN') in the flanking part is a prepositional phrase that could be put in the
'middle' of a sentence?

Sorry for bothering you with this comparison issue for so long time.
I'm really trying to find a rule that could be engraved in my mind, so I'm well prepared for other similar
questions of comparison when I meet them later in the real test.

Question 32
Question 32
Question 32
Question 32
Question 32
Question 32
Question 32
A The ability of .have become -out for wrong subject verb agreement
E- Scientists ability have become wrong subject verb agreement.
In D, like should be replaced by such as and ever accurate should be used for models
In B it states that The ability of scientists has become ever more accurate wrong it is models, which are more accurate instead of ability
So C wins.

egmat
e-GMAT Representative

jrashish wrote:
bigoyal wrote:
The ability of scientists to provide models of the atmosphere's complex responses to changing conditions, like seasonal and daily cycles or different
planetary conjunctions, have become ever more accurate.

(A) The ability of scientists to provide models of the atmosphere's complex responses to changing conditions, like seasonal and daily cycles or
different planetary conjunctions, have become ever more accurate.
(B) The ability of scientists has become ever more accurate in providing models of the atmosphere's complex responses to changing conditions,
such as seasonal and daily cycles or different planetary conjunctions.
(C) Scientists have become able to provide ever more accurate models of the atmosphere's complex responses to such changing conditions as
seasonal and daily cycles or different planetary conjunctions.
(D) Scientists have become ever more accurate in their ability for providing models of the atmosphere's complex responses to changing conditions,
like seasonal and daily cycles or different planetary conjunctions.
(E) Scientists' ability to provide models of the atmosphere's complex responses to such changing conditions as seasonal and daily cycles or different
planetary conjunctions have become ever more accurate.

Please accompany your answers with your reasoning.

Source:GMATPrep
A The ability of .have become -out for wrong subject verb agreement
E- Scientists ability have become wrong subject verb agreement.
In D, like should be replaced by such as and ever accurate should be used for models
In B it states that The ability of scientists has become ever more accurate wrong it is models, which are more accurate instead of ability
So C wins.

egmat
e-GMAT Representative

jrashish wrote:
bigoyal wrote:
The ability of scientists to provide models of the atmosphere's complex responses to changing conditions, like seasonal and daily cycles or different
planetary conjunctions, have become ever more accurate.

(A) The ability of scientists to provide models of the atmosphere's complex responses to changing conditions, like seasonal and daily cycles or
different planetary conjunctions, have become ever more accurate.
(B) The ability of scientists has become ever more accurate in providing models of the atmosphere's complex responses to changing conditions,
such as seasonal and daily cycles or different planetary conjunctions.
(C) Scientists have become able to provide ever more accurate models of the atmosphere's complex responses to such changing conditions as
seasonal and daily cycles or different planetary conjunctions.
(D) Scientists have become ever more accurate in their ability for providing models of the atmosphere's complex responses to changing conditions,
like seasonal and daily cycles or different planetary conjunctions.
(E) Scientists' ability to provide models of the atmosphere's complex responses to such changing conditions as seasonal and daily cycles or different
planetary conjunctions have become ever more accurate.

Please accompany your answers with your reasoning.

Source:GMATPrep
2. Idiom Error: such as vs like: As in choice A.

3. Idiom Error: ability for providing is not idiomatically correct expression. The correct expression is ability to provide.

Choice E Meaning error as in Choice A.

Thus, choice C is the correct answer.

What are the key take-away messages?


1. If the original sentence does not communicate logical meaning, then the correct answer should be the one that is grammatically correct and that
communicates a logical meaning.
2. Such as should be used to present examples. Like should not be used to present examples.
3. Ability for verb-ing is idiomatically incorrect expression. The correct expression is ability to verb.

https://www.manhattanprep.com/gmat/forums/the-ability-of-scientists-to-provide-models-of-the-t3725.html

RonPurewal

ManhattanGMAT Staff

Re: The ability of scientists to provide models of the


Thu Jul 10, 2008 4:14 am

Sputnik wrote:I felt C is awkward.... Scientists have become able to provide ever

you're not parsing it correctly: 'ever more accurate' is an indivisible phrase here. 'ever more accurate' is roughly equivalent to
'getting more and more accurate all the time'.
once you make that realization, the right answer should make more sense

RonPurewal
ManhattanGMAT Staff

Tue Aug 19, 2008 3:52 am

Guest660 wrote:Hi Ron,

Doesn't C change the meaning ...

A - says ability has become more accurate ??


B - more accurate models ??

well, sure, but remember that you're allowed to change the meaning of a sentence if the original "meaning" doesn't make any sense, as is the case here.

an ability can't be "accurate", because an ability is not something that can be compared quantitatively to a "true" or "target" mark of some sort. by
contrast, models (which can approximate true quantitative phenomena), shots at a target (which can come close to the center of the target), and so on can
be "accurate".

so yes, (c) changes the meaning, i guess, but it's a desirable change of meaning - because it takes the sentence from a nonsense phrasing to a phrasing
that actually makes sense.

RonPurewal
ManhattanGMAT Staff

Tue Aug 19, 2008 3:55 am

H wrote:
I believe that "able" is an adjective, and so it has to modify a noun.
Usually an adjective is placed right before or after the noun that it tries to modify.
"able" in C seems weird to me because it is trying to modify (I believe) "the scientists".
Is "able" a special adjective that is allowed to have such flexibility?
If not, could you share some other examples?
Thanks in advance.

nothing special here; all adjectives can do this.


specifically, any adjective can be separated from the noun it's trying to describe by verbs of equivalence ("copulative verbs"), such as be, become, seem,
look, and so on.

for instance:
that food looks hot.
this bar seems crowded.
here, "hot" and "crowded" are adjectives, describing, respectively, "food" and "bar", but these two sentences are clearly ok.
same sort of deal with choice (c).
RonPurewal
ManhattanGMAT Staff

Re: The ability of scientists to provide models of the

Thu Aug 27, 2009 5:38 am

cesar.rodriguez.blanco wrote:I do not know what "has become more accurate"? The ability or the scientist?

actually, neither.

it would be nonsense to say that an ability is "accurate".


it would be similarly nonsense to say that a person (here, a scientist) is "accurate".

the model should be "accurate".


choice (c) does this.
the other two don't. in both (b) and (e), the ability is described as "accurate". that doesn't make sense.

Is there any difference between "changing conditions, such as seasonal and daily cycles" and "such changing conditions as
seasonal and daily cycles or different planetary conjunctions"?

those are basically the same.

in general, there is no consistent way to distinguish between "such X as Y" and "X such as Y", nor is one of those generally preferred
to the other.

Moreover, I thought "Scientists have become able to provide ever" is wordy because you can say "Scientist can provide"? What is
wrong with my reasoning

you're dividing this in the wrong place.

it's actually
scientists can provide ever more accurate models

where "ever more ADJ" is an idiomatic expression that means "increasingly ADJ".

RonPurewal
ManhattanGMAT Staff

Re: The ability of scientists to provide models of the

Sat Oct 23, 2010 7:35 am

ravi.bamalwa wrote:Ablilty to provide VS Ability for providing VS Ability in providing.

Please correct me if im mistaken but the first phrase above is the only one which is idiomatically correct. The
remaining two are unidiomatic.Hence, by this rule itself, choices B and D can be eliminated, leaving choices A C and
E.
that's correct.

Aksy
Course Students

Re: The ability of scientists to provide models of the

Thu Feb 16, 2012 12:22 am

Before I encountered this problem on GMATPrep, I was of the belief that words such as "more" and "less" should always be followed
by "than" because words such as "more" are used in comparison to something else. In this case, when we say that something has
become more accurate, should't it indicate more accurate than what? - More accurate than it was previously or more accurate than
something else?

tim
ManhattanGMAT Staff

Re: The ability of scientists to provide models of the

Fri Feb 17, 2012 8:26 pm

first, be careful bumping questions. every time you do that it puts the question at the absolute end of the queue for us to answer. bumping a question is
the most effective way to cause a delay in getting your question answered..
to answer your question, "ever more X" is different from "more Y than Z", and both are valid. "ever more X" is in fact a comparison;
it just inherently compares the level of X at any given point in time to the level at any point in the past..

divineacclivity
Forum Guests

Re: The ability of scientists to provide models of the

Tue Oct 02, 2012 10:23 am

RonPurewal wrote:

Sputnik wrote:I felt C is awkward.... Scientists have become able to provide ever

you're not parsing it correctly: 'ever more accurate' is an indivisible phrase here. 'ever more accurate' is roughly
equivalent to 'getting more and more accurate all the time'.

once you make that realization, the right answer should make more sense.

Ron,

I chose C over other choices because ability was being said to be accurate in all other choices.
But I have a confusion, which I want to clarify so that I can pick a correct choice if the other choices are not so clearly wrong :).

Scientists have become able to... - is it a good usage?


shouldn't it be more like this one --> Scientist have become more capable of ... or Scientists are able to do something more
accurately

thanks in advance

jlucero

ManhattanGMAT Staff

Re: The ability of scientists to provide models of the

Fri Oct 12, 2012 3:22 pm

It depends on what you are trying to express:

Scientist have become more capable of doing something.


Scientists are able to do something more accurately.

These say that scientists were able to do something, but are now better at it.

Scientists are now able to do something.

Sounds better because we are using a simple present tense. But if I want to say scientists "have become able", I am using the
present perfect form to say that since a past tense event, something happened.

Since I began studying for the GMAT I have become boring.


Scientists have become able to clone wooly mammoths since they discovered a new method for working with DNA.

jnelson0612
ManhattanGMAT Staff

Re: The ability of scientists to provide models of the

Fri Mar 01, 2013 11:01 pm

harithachillarige wrote:Hi,

Apart from the errors already doscussed ..

Can we eliminate Options A and E because of "Have become" ?


Since the subject of the sentence is singular "Ability", it should be "has become".

Please confirm

I absolutely agree! Good catch! :-)


aflaamM589
Students

Re: The ability of scientists to provide models of the

Fri Jul 15, 2016 9:00 pm

Hello Ron,
Should to provide/ in providing/ for providing be placed as close as possible to ability/ able.
Can A and B be crossed out for this reason.
Whereas in CDE at least the placement is correct.

The ability of scientists to provide models of the atmospheres complex responses to changing conditions, like seasonal and daily
cycles or different planetary conjunctions, have become ever more accurate.
A. The ability of scientists to provide models of the atmospheres complex responses to changing conditions, like seasonal and
daily cycles or different planetary conjunctions, have become ever more accurate.
B. The ability of scientists has become ever more accurate in providing models of the atmospheres complex responses to
changing conditions, such as seasonal and daily cycles or different planetary conjunctions.
C. Scientists have become able to provide ever more accurate models of the atmospheres complex responses to such changing
conditions as seasonal and daily cycles or different planetary conjunctions.
D. Scientists have become ever more accurate in their ability for providing models of the atmospheres complex responses to
changing conditions, like seasonal and daily cycles or different planetary conjunctions.
E. Scientists ability to provide models of the atmospheres complex responses to such changing conditions as seasonal and
daily cycles or different planetary conjunctions have become ever more accurate.

Moreover,
is Scientists ability also problematic in E, stylistically if not grammatically?
( apostrophe with plural ). Can you let us known your thoughts on it.
Thanks in anticipation.
Highly indebted for your guidance all throughout my journey.

RonPurewal
ManhattanGMAT Staff

Re: The ability of scientists to provide models of the

Sun Jul 17, 2016 2:23 am

aflaamM589 wrote:Should to provide/ in providing/ for providing be placed as close as possible to ability/ able.
Can A and B be crossed out for this reason.

yes. well done.

RonPurewal
ManhattanGMAT Staff

Re: The ability of scientists to provide models of the

Sun Jul 17, 2016 2:23 am

Moreover,
B. that beached on an African shore more than a million years ago and then was subsequently butchered by hominids has

Fossils has- subject verb agreement error


then subsequently- redundancy

C. that beached on an African shore more than a million years ago, which was subsequently butchered by hominids, has-
Fossils has- subject verb agreement error

D. having been beached on an African shore more than a million years ago and subsequently butchered by hominids, have
participle(having...) is not parallel to verb (butchered)

E. having beached on an African shore more than a million years ago and then subsequently were butchered by hominids have
then subsequently- redundancy
participle(having...) is not parallel to verb (butchered)

OA- A

Hope this helps!


Dolly Sharma
Verbal Trainer
CrackVerbal

egmat
e-GMAT Representative
https://gmatclub.com/forum/fossils-of-a-whale-that-beached-on-an-african-shore-more-81590-20.html#p1381086
Hi jrashish,

Thank you for posting your query here.

Lets take a look at the sentence structure of this sentence:

Fossils of a whale (Clause-I)


o that (Clause-II)
beached on an African shore more than a million years ago (Clause-II)
and was subsequently butchered by hominids (Clause-II)
have been recovered by paleontologists. (Clause-I)

So, this sentence has two clauses as shown. The subject verb pairs in both the clauses are highlighted.

Now, the dependent clause starting with that tells us two things about a whale:
1. It beached on an African shore at a certain time in the past
2. It was butchered by hominids.
So, the verbs "beached" and "was butchered" are parallel to each other.

Note that, active and passive verbs can be parallel to each other if their subject is the same. Lets take another official example
(correct version) with similar structure:

Dressed as a man and using the name Robert Shurtleff, Deborah Sampson, the first woman to draw a soldier's pension,
o joined the Continental Army in 1782 at the age of 22,
o was injured three times,
o and was discharged in 1783 because she had become too ill to serve.

Here, the list of the verbs for the subject Deborah Sampson is:
1. joined (Active Voice)
2. was injured (Passive Voice)
3. was discharged (Passive Voice)

As we can see, first verb is written in active voice while second and third verbs are in passive voice. However, these verbs are
parallel. Similarly in the given sentence, "beached" and "was butchered" are parallel.

http://www.beatthegmat.com/fossils-of-whale-t105642.html

GMATGuruNY GMAT Instructor


Crystal W wrote:
I have a small question about choice C. I eliminate C according to the has, But do someone know if the use of which is correct? If it is correct, does
which refer to whale?
Thanks in advance!
C: more than a million years ago, which
Here, COMMA + which is incorrectly preceded by an adverb (more than a million years ago).
COMMA + which must be preceded by NOUN.
Eliminate C.

https://www.manhattanprep.com/gmat/forums/sc-fossils-and-whales-sv-agreement-and-prep-phrase-mod-t7965.html

nitin_prakash_khanna
Students

Re: SC: Fossils and Whales, SV Agreement and prep phrase mod
Tue Sep 01, 2009 1:42 am

This is how i understood it....

First of all its one of those passive sentences that are correct on GMAT :>

it is testing three things S-V, Active / Passive voice and modifier.

for me the original sentence is


paleontologists have recovered Fossils of a whale. (which tells that action just finished in recent past and hence present perfect)

Now if you write above in passive

Fossils of a whale have been recovered by paleontologists . (which is what the sentence is saying becuase the emphasis is on the
Fossils rather than Scientists)

Now all of the other text after whale is an essential modifier which defines the whale and off course you need to check parallelism
and grammar in that. All options other than A have some issue.

thanks
nk.

Ben Ku

ManhattanGMAT Staff

Re: SC: Fossils and Whales, SV Agreement and prep phrase mod

Sat Sep 26, 2009 2:03 pm

If we break down this sentence, the CORE of the sentence (after we've stripped away the modifiers/middlemen) is:
"Fossils ... have been recovered." The subject of the sentence is fossils, NOT whale.

The relative pronoun "that" serves to introduce a (very long) noun modifier.
" ... that beached on an African shore more than a million years ago and was subsequently butchered by hominids"
This noun modifier correctly modifies whale.

Let me know if you have other questions. Thanks.

RonPurewal
ManhattanGMAT Staff

Re: SC: Fossils and Whales, SV Agreement and prep phrase mod

Mon Apr 26, 2010 8:40 am

christina.susie.wong wrote:Could you please help explain why in answer choice A "Fossils" does not have to agree in
number with "was" in the part of the sentence that says "and was subsequently."

you don't want "fossils" to agree in number with that verb, because the context makes it quite clear that "fossils" is NOT meant to
be the subject of that verb. (it doesn't make any sense to say that fossils were butchered; its the whale that was butchered.)

you should process the sentence like this:


Fossils of
a whale that beached on an African shore more than a million years ago and was subsequently butchered by hominids have
been recovered by paleontologists

actually, this is the only parallelism that's possible in the sentence in the first place, since "beached" is the only verb that can
possibly be parallel to "and was subsequently butchered".

RonPurewal

ManhattanGMAT Staff

Re: SC: Fossils and Whales, SV Agreement and prep phrase mod

Sat Aug 14, 2010 6:31 am

vinversa wrote:adding to the discussion

Please note the THAT effect here

Fossils of a whale that beached on an African shore more than a million years ago and was subsequently butchered
by hominids have been recovered by paleontologists.

Re-writing the same sentence withOUT "THAT"

Fossils of a whale beached on an African shore more than a million years ago and were subsequently butchered by
hominids.

OTHER similar examples


see the "hand signs" example -- same thing, pretty much exactly.

i am sorry for asking several seemly unrelated questions; acctually, i just want to go one step from the test problem and thus to
learn more actively

nah, these are good questions.

RonPurewal

ManhattanGMAT Staff

Re: SC: Fossils and Whales, SV Agreement and prep phrase mod

Fri Nov 12, 2010 9:18 pm

Mymisc wrote:Would you please explain why (D) is wrong?

"having been beached" is a passive voice construction, which in this case is inappropriate. (this does NOT mean that the passive
voice is incorrect in general -- there are plenty of other situations in which the passive voice is preferred or even required -- but it
just doesn't work here.)
in particular, if i say "an X ... having been stolen", the implication of the passive voice is that someone else stole X.
similarly, "having been beached" seems to indicate that someone else beached the whale. that's not true.

I have two other questions on this SC:


1. The modifier inside the bracket {} is a parallel structure using "AND". There is "was" with butchered, but no "was" with
RonPurewal

ManhattanGMAT Staff

Re: SC: Fossils and Whales, SV Agreement and prep phrase mod

Thu Jun 28, 2012 5:23 am

shah.abhilash wrote:We can delete options C and D saying the subject is still the "FOSSIL" and fossil cant be
butuchered millions of years ago by hominids. Wrong minor modifier. :-)[/color]

no.
in (d), "beached" and "butchered" are pretty clearly parallel (although there are other errors), so you can attribute both of them to
the animal.

in (c), there's an issue with "which", but the issue is that it basically doesn't refer to anything legitimate.

in neither of those choices can you attribute those things to the fossils

RonPurewal

ManhattanGMAT Staff

Re: SC: Fossils and Whales, SV Agreement and prep phrase mod
Mon Apr 21, 2014 7:22 pm

yangfan0307 wrote:Sorry for bring up this thread, I have one question regarding (A),
why it omits the "that" in front of "was subsequently butchered".

thanks.

There's no need to repeat "that", unless the sentence is ambiguous without it.

RonPurewal
ManhattanGMAT Staff

Re: SC: Fossils and Whales, SV Agreement and prep phrase mod

Wed Aug 26, 2015 4:55 am

DiJ92 wrote:Dear ron,


in choice E, there is a construction X having done. Is it correct to use this kind of constructions. thank you

short answer:
i'd bet against any sentence containing this.
--

long answer:

read this.
https://www.manhattanprep.com/gmat/foru ... ml#p112670

now think about the difference between 'people who have __ed' and 'people having __ed'.
see, the idea that "someone has __ed" is normally permanent. (People who have seen a certain movie will always be people who
have seen that movie.)
as per the link, 'who have __ed' suggests that something is permanent, while 'having __ed' suggests that it is transient/temporary.
so, the former is generally going to make more sense here.

https://www.manhattanprep.com/gmat/foru ... ml#p112670

RonPurewal

ManhattanGMAT Staff

Re: On account of a law passed in 1993, making it a crime punish

Wed Mar 25, 2015 3:05 am

ok, i see. (in the version you posted above, there's a ton of junk between "evidence" and "suggesting". that version implies that the
scientists, not the evidence, "suggested" things.)

the difference between those two modifiers is subtle. because it is subtle, it will NEVER be the only decision point in a
problem (--> go look for something more fundamental!).
the difference is basically this:
"...that [verb]s" implies that [verb]ing is a permanent or fundamental aspect of the noun that's described.
"...[verb]ing", on the other hand, implies that it's temporary and can/will change.

e.g., if i tell you that i have friends who work in finance, i'm implying that finance is their long-term career.
on the other hand, if i tell you that i have friends working in finance, then there is no such implication (e.g., maybe they'll quit
tomorrow and become consultants, or form a startup, or retire and move to a tropical island).

RichaChampion
Students

Re: SC: Fossils and Whales, SV Agreement and prep phrase mod

Fri Jul 22, 2016 2:16 am

RonPurewal wrote:oh, and, there's an easier way to detect that (d) is wrong: it tries to block off a modifier with a comma on one side,
but not on the other side.

this isn't acceptable -- a modifier that occurs in the middle of the sentence (i.e., not right at the beginning, and not right at the end)
should be blocked off by either 0 or 2 commas.**

in choices (c) and (d), this doesn't happen -- in each of those choices, the big modifier that ends with "hominids" has a comma
afterward, but NOT a comma before.

--
Question 37

Question 37
https://gmatclub.com/forum/the-first-detailed-study-of-magpie-attacks-in-australia-77464.html

Difficulty: 75% (hard)


Question Stats: 47% (01:56) correct 53% (01:07) wrong based on 1752 sessions
A:6% B:13% C:46% D:13% E:22%

deeptikachalia
Intern
https://gmatclub.com/forum/the-first-detailed-study-of-magpie-attacks-in-australia-77464.html?sid=a336380def35c639da724f8a583571bf#p880155

11 This post received KUDOS


5 This post was BOOKMARKED

noboru wrote:
I am for C.

Here's why:
a) by the time they (who? birds? attacks? people?) had reached adulthood, 98 percent of men and 75 percent of women born in
country have been attacked by the birds
b) by the time they (who? birds? attacks? people?) reach adulthood, 98 percent of men and 75 percent of women, who were born in
the country, had been attacked (incorrect use of perfect tense) by birds.
c) by the time they correctly modifies the noun that immediately follows comma reach adulthood, 98 percent of men and 75 percent
of the women born in the country had been attacked by the birds.
d) 98 percent of the men and 75 percent of the women that were born in the country were attacked by the birds by the time they
(incorrectly modifies birds) reach adulthood.
e) 98 percent of men and 75 percent of women who were born in the country, by (illogical. try to replace the long noun with a
simple word such as "people" and try to read the sentence from beginning till the end, and you will notice "study suggests that
people, by the time they reached adulthood...."makes no sense) the time they reached adulthood had been attacked by birds

Handy Trick: Replace a long wordy noun with a simpler one and re-read the whole sentence to avoid such mistakes.
egmat
e-GMAT Representative
EXPERT'S POST
Hi All,

The first detailed study of magpie attacks in Australia indicates that by the time they had reached adulthood, 98 percent of men and
75 percent of women born in the country have been attacked by the birds.

It is very important to understand the intended meaning of the sentence to ascertain the timeline of the events. The first detailed
study of magpie attacks in Australia suggests that by the time 98% of men and 75% of women reached adulthood, they had already
been attacked by the bird.

Notice that the sentence says that men and women had already been attacked by the bird even before they reached adulthood.
This means that they got attacked first and then they reached adulthood. Now this can be reported in two ways:
a. by the time they reach adulthood, 98% of men and 75% of women have been attacked by the birds. (Reported in Present
context)
b. by the time they reached adulthood, 98% of men and 75% of women had been attacked by the birds. (Reported in Past context)

Error Analysis:
1. This sentence uses past perfect tense for the event that took place later and present perfect tense for the event that took place
earlier. In order to establish the correct sequencing, we must use past perfect for the event that took place earlier, that is men and
women being attacked, and simple past tense for the event that took place later, that is they reaching adulthood.

POE:
Choice A: by the time they had reached adulthood, 98 percent of men and 75 percent of women born in the country have been
attacked by the birds. Incorrect for the verb tense error discussed above.

Choice B: by the time they reach adulthood, 98 percent of men and 75 percent of women, who were born in the country, had been
attacked by the birds. Incorrect. This choice repeats the verb tense error by using simple present tense for the event that took place
later when past perfect has been correctly used for the earlier event.

Choice C: by the time they reached adulthood, 98 percent of men and 75 percent of women born in the country had been attacked
by the birds. Correct. The verb tense error has been corrected here.

Choice D: 98 percent of men and 75 percent of women that were born in the country were attacked by the birds by the time they
reach adulthood. Incorrect.
a. that cannot be used to refer to men and women.
b. reach is not written in the correct verb tense.

Choice E: 98 percent of men and 75 percent of women who were born in the country, by the time they reached adulthood had been
attacked by the birds. Incorrect. We Have modifier ambiguity in this choice. Notice that by the time they reached adulthood is
placed little awkwardly that makes it ambiguous that which entity its modifying. It seems as if it is modifying who were born in the
country. This modifier should have a comma after adulthood to do away with this ambiguity.

PS Pronoun they in Choices A, B, C and E refers to men and women because only these men and women reached adulthood.
There is no other antecedent they can logically refer to.

1. Use past perfect for the event that took place earlier and simple past for the event that took place later.
2. Use the correct combination of verb tenses depending upon the context of the sentence.
3. There should not be any ambiguity in modifier reference.

Hope this helps.


Thanks.
Shraddha
sayantanc2k
Verbal Expert

DensetsuNo wrote:
pmal04 wrote:
The first detailed study of magpie attacks in Australia indicates that by the time they had reached adulthood, 98 percent of men and 75 percent of
women born in country have been attacked by the birds.

a) by the time they had reached adulthood, 98 percent of men and 75 percent of women born in country have HAD been attacked by the birds.
b) by the time they reach adulthood, 98 percent of men and 75 percent of women, who were born in the country, had been attacked by birds. Super
wordy, ambiguous and... Yoda-ish
c) by the time they reached adulthood, 98 percent of men and 75 percent of the women born in the country had been attacked by the birds. Might
be it.
d) 98 percent of the men and 75 percent of the women that were born in the country were attacked by the birds by the time they reachED
adulthood.
e) 98 percent of men and 75 percent of women who were born in the country, by the time they reached adulthood had been attacked by birds. For
this one just try to find the read the two separate pieces trying to figure which one is the main clause.

Since I didn't see anyone solving E) in this way, I wanted to make sure that the following approach is indeed correct.
When comparing sentence C) and E) we can eliminate phrase E) as it is made of two subordinate clauses, ie. none of the two can work alone.
Do you agree?

Densetsu.

Option E does not have 2 independent clauses. It has


1. one main clause whose subject is "98 percent of men and 75 percent of women who were born in the country" and verb is "had been attacked"
and
2. a dependent clause " by the time they reached adulthood".

The positioning of the dependent clause is wrong in option E; it props up within the main clause, making the construction unclear ( although such
embedding of dependent clause within a main clause is alright in some cases if the meaning is not blurred).

sayantanc2k
Verbal Expert

DensetsuNo wrote:
Thanks sayan,
I always saparate clauses with the comma, good to know that it doesn't always mean that they're two separate clauses!

Btw, do you have any example of such "nested" principal clauses?

Regards,
Densetsu

Sorry for the typo..... should be "such embedding of independent clause....". Following is a simple example:

John, who is known for his whimsical decisions, has become the operation manager.
The dependent clause "who is known for his whimsical decisions" is embedded within the main clause.

Nonetheless it is possible to embed even main clause within another using "-.......-". Following is an example from OG13:

Although heirloom tomatoes, grown from seeds saved during the previous year, appear less appetizing than most of their round and red supermarket
cousins - they are often green and striped, or have plenty of bumps and bruisesheirlooms are more flavorful and thus in increasing demand.

sayantanc2k
Verbal Expert

DensetsuNo wrote:
sayantanc2k wrote:
DensetsuNo wrote:
Thanks sayan,
I always saparate clauses with the comma, good to know that it doesn't always mean that they're two separate clauses!

Btw, do you have any example of such "nested" principal clauses?

Regards,
Densetsu

Sorry for the typo..... should be "such embedding of independent clause....". Following is a simple example:

John, who is known for his whimsical decisions, has become the operation manager.
Thu Jul 30, 2009 6:24 am

kimberlylin wrote:The first detailed study of magpie attacks in Australia indicates that by the time they had reached
adulthood, 98 percent of men and 75 percent of women born in the country have been attacked by the birds.

a.) by the time they had reached adulthood, 98 percent of men and 75 percent of women born in the country have
been attacked by the birds.
b.) by the time they reach adulthood, 98 percent of men and 75 percent of women, who were born in the country,
had been attacked by the birds
c.) by the time they reached adulthood, 98 percent of men and 75 percent of women born in the country had been
attacked by the birds
d.) 98 percent of men and 75 percent of women that were born in the country were attacked by the birds by the time
they reach adulthood
e.) 98 percent of men and 75 percent of women who were born in the country, by the time they reached adulthood
had been attacked by the birds

OA: C

I answered E for this question, but now that I look back and reread the problem with Answer E, I understand why this
answer choice is a bit awkward. However I can't find anything grammatically incorrect with the sentence.

Looking at the answer choices, my first instinct is to eliminate choices A, B, and C because the fourth word, "they",
seems problematic to me. I know that sometimes sentences can have a construction of this type. I don't recall the
technical names for a sentence like this, but an example would be: "After he went to the park, Johnny decided to go
home." But despite knowing this, I still think the "they" in A,B,C is ambiguous since it can refer to either the "magpie
attacks". Of course, logically, magpie attacks cannot reach adulthood, so I further assume that the question was
trying to imply that -- by the time magpies reach adulthood, they attack 98% of men and 75% of women. Since the
intent of the sentence is unclear, "they" is incorrect.
our understanding of the gmat's pronoun rules has evolved by leaps and bounds recently.

in particular, we have discovered that the gmat actually has much, much more tolerance for ambiguous pronouns than we had
previously thought.

so, a takeaway for you:


the rules on ambiguous pronouns are NOT absolute.

there are only two ABSOLUTE RULES for pronouns:


(1) the pronoun must stand for a noun that is actually PRESENT in the sentence;
(2) the pronoun and the noun must MATCH IN TERMS OF SINGULAR/PLURAL.

the other "rules", such as those that govern ambiguity of pronouns, are more like "guidelines" or "suggestions".
therefore, you should leave those criteria for last - i.e., until after you've narrowed down the choices based on all other criteria that
you can find.

Can you perhaps provide some examples?

examples of what, exactly?


do you want more sentences in which there are pronouns that are technically "ambiguous" but are actually correct? there are plenty
of those on this forum.

try the following on for size:


pronoun-doubt-t7303.html
maya-and-aztec-t2690-15.html

Another problem I have is - in choices A,B,C (and in fact D and E), do the "they"'s refer to the men/women, or the percents?
if something appears in ALL FIVE CHOICES, you can ignore it.

and, on top of that, if something appears in all five choices, then you KNOW that it is OK - because one of those choices
must, after all, be the correct answer, and everything in the correct answer is correct.

Lastly, in sentences like D and E, is "that" or "who" the proper word to describe "98 percent of men and 75 percent of women"? I
know that "who" should only be used for people but am not sure if we should be describing the percentages or the men/women in
this case.

Thanks!

you want "who".

it's not the percentage that was born in the country.


use context:
you are taking a percentage of ALL people who were born in the country. therefore, the "who" refers loudly and clearly to "men" /
"women", not to the percentages.

RonPurewal
ManhattanGMAT Staff
Re: The first detailed study of magpie attacks in Australia

Sun Sep 13, 2009 12:41 pm

DennaMueller wrote:I'm still confused as to why 'B' could not be the correct answer. It used the past perfect 'had
been'.

in (b), the past perfect is inappropriately matched with the OTHER verb, which is in the present tense.

either of the following two constructions would be appropriate:


...by the time they reach adulthood, they have been attacked...
...by the time they reached adulthood, they had been attacked...

you can't "mix and match" between these - i.e., reach + had been is wrong, as is reached + have been.

RonPurewal
ManhattanGMAT Staff

Re: The first detailed study of magpie attacks in Australia

Sun Sep 13, 2009 12:45 pm


cesar.rodriguez.blanco wrote:What are the differences between options C and E? Why E is wrong?
I do not understand what are the differences in the meaning, although I know that there are severals because the
modifiers are placed in different places.

hmm. well, the major problem is that choice (e) attempts to set off a modifier with a single comma (on the left, but NOT on the
right).

you can't do this. if a modifier is in the middle of a sentence, then you must either
* set it off with TWO commas, one on each side; or
* not set it off with commas at all.

the man who showed up yesterday is back again --> ok (meaning = there is more than one man, and i have to clarify that it's the
one who showed up yesterday)
the man, who showed up yesterday, is back again --> ok (meaning = we already know which man; i'm merely providing more detail)
the man, who showed up yesterday is back again --> not ok
the man who showed up yesterday, is back again --> not ok

--

choice (e) is also fatally awkward, although it would probably take a native speaker (if not a seasoned writer of formal english) to
ascertain that.

RonPurewal
ManhattanGMAT Staff
Re: The first detailed study of magpie attacks in Australia

Thu Nov 25, 2010 4:38 am

maribelsalazar02 wrote:I still can't tell why E is not correct (other than it's awkward).. what are the rules that E is
breaking? Thanks!

that choice is incorrect because it contains a modifier ("by the time they reached adulthood") that is blocked off by a comma on one
side, but not on the other side.
a modifier must be either blocked off by commas or not blocked off by commas. you can't block off a modifier with a
comma on one side but not on the other.

the only exception to this rule occurs in the case of modifiers that are not actually blocked off by commas, but which happen to be
placed next to other construction that contain commas as a matter of necessity.
for instance:
A man from Tornio, Finland, who has developed several innovative cell-phone applications will be speaking at today's meeting.
in this sentence, it appears that "who has developed severas innovative cell-phone applications" is blocked off by only one comma,
but it really isn't -- the comma belongs to the foregoing construction ("from Tornio, Finland"), since city/state and city/country are
always followed by commas.
i.e., the real structure of this sentence is
A man from Tornio, Finland, who has developed several innovative cell-phone applications will be speaking at today's meeting.

this sentence is also a little bit awkward in the sense that the referent of "by the time they reached adulthood" is not entirely clear --
the sentence could also be interpreted as referring to people who "had been born in the country by the time they reached
adulthood".
i mean, heh... but still, technically that's an ambiguity.

RonPurewal
ManhattanGMAT Staff

Re: The first detailed study of magpie attacks in Australia

Sat Jul 30, 2011 4:11 am

kvitkod wrote:Hi, Ron,

Basing on your explanation, I believe that pronoun in D is more or less OK. What other flaws violate Option D?

* the modifier "that..."is used to refer to people. ("that" can only be used to refer to non-persons; it must be changed to "who" for
people)

* "reach" is in the present tense; these people reached adulthood in the past, so that tense makes no sense.

RonPurewal
ManhattanGMAT Staff
Re: The first detailed study of magpie attacks in Australia

Mon Aug 15, 2011 3:17 am

PT.SHAH wrote:Isn't
"by the time they reach adulthood, 98 percent of men and 75 percent of women born in the country have been
attacked by the birds"
a better answer than all the 5 options as the the sentence starts in present tense with "indicates that"?

a statistical study must examine things that have already happened, so, no.

RonPurewal
ManhattanGMAT Staff

Re: The first detailed study of magpie attacks in Australia

Sat Oct 15, 2011 4:00 am

robosc9 wrote:Hi,

How is 'they' ambiguous?

it's a study of 'magpie attacks'


Mon Feb 06, 2012 5:06 pm

Dave, the "had been attacked" is the correct construction as long as it occurs before something else in the sentence; in this case it
occurs before "they reached adulthood". When the people were born is irrelevant, because the attacks are linked to reaching
adulthood. On the other hand, in your second post, by adding the comma youve linked the "had been attacked" to "were born",
which actually does cause a problem as youve identified. BTW, "born" is definitely not a verb in this sentence; it is a modifier..

As for the other questions in your second post, they are a little abstract and perhaps over-generalizations. It would definitely be
more helpful if you could provide some concrete examples so we can help you with those..

Thanghnvn, it sounds like you have the right idea. You definitely need to make sure you understand GMAT grammar rules and apply
them even if they conflict with what youve seen elsewhere..

davetzulin
Forum Guests

Re: The first detailed study of magpie attacks in Australia

Mon Feb 06, 2012 6:34 pm

tim wrote:Dave, the "had been attacked" is the correct construction as long as it occurs before something else in the
sentence; in this case it occurs before "they reached adulthood". When the people were born is irrelevant, because
the attacks are linked to reaching adulthood. On the other hand, in your second post, by adding the comma youve
linked the "had been attacked" to "were born", which actually does cause a problem as youve identified. BTW,
"born" is definitely not a verb in this sentence; it is a modifier..
refering to persons is not allway wrong. This is not hard and fast rule as Ron used to said so. I said this is not absolute error.

is my thinking correct? Ron,Manhantan experts. Pls, confirm

RonPurewal
ManhattanGMAT Staff

Re: The first detailed study of magpie attacks in Australia

Wed Mar 07, 2012 8:46 pm

kuldeep,

kuldeep_rojans wrote:In the choice C Is contradicting your rule.

interesting. noted.
still,
* i've never seen that kind of construction in the part of a problem that's actually tested;
* in fact, i've never seen punctuation explicitly tested at all.

still, that's a good observation; i'll bookmark it.


RonPurewal
ManhattanGMAT Staff

Re: The first detailed study of magpie attacks in Australia

Wed Mar 07, 2012 8:47 pm

thanghnvn wrote:"who relative clause" is prefered than "that relative clause" when the modifier refers to persons.
However, "that relative clause" refering to persons is not allway wrong. This is not hard and fast rule as Ron used to
said so. I said this is not absolute error.

is my thinking correct? Ron,Manhantan experts. Pls, confirm

if the pronoun "that" is used to stand for a person, it is flat-out incorrect.

aps_asks
Forum Guests

Re: The first detailed study of magpie attacks in Australia

Thu Apr 05, 2012 12:08 am

Hi Ron ,

On what basis have you stated the below rule ?


either of the following two constructions would be appropriate:
...by the time they reach adulthood, they have been attacked...
...by the time they reached adulthood, they had been attacked...

So that we can apply the same to other cases

Is it because the tense should be consistent in a sentence ?

tim
ManhattanGMAT Staff

Re: The first detailed study of magpie attacks in Australia

Sun Apr 08, 2012 4:25 pm

it's because the tenses have to be used correctly. please refer to the verb tenses section of our SC strategy guide for information
about how to use these tenses..

thulsy
Forum Guests
Re: The first detailed study of magpie attacks in Australia

Tue Jul 10, 2012 7:07 pm

Ron has given a detailed explanation for this problem on the Nov 18, 2010 Study Hall (that was really helpful!), focusing on verb
tense. I just have one question that is uncovered.

The first detailed study of magpie attacks in Australia indicates ...

I think "indicated" would make more sense than "indicates", since "indicates" means the action is not fixed in any particular
timeframe but here we have "the first detailed study", which occurred in the past.
I'm not questioning the correct answer... just want to know how "indicates" work here. Thanks in advance.

RonPurewal
ManhattanGMAT Staff

Re: The first detailed study of magpie attacks in Australia

Wed Jul 18, 2012 1:18 am

thulsy wrote:I think "indicated" would make more sense than "indicates", since "indicates" means the action is not
fixed in any particular timeframe.

the significance of the study's findings is not limited to the timeframe in which the study itself was conducted, so this is exactly the
meaning that the sentence should have.

gaurav1a2b
Forum Guests

Re: The first detailed study of magpie attacks in Australia

Tue Jan 15, 2013 2:10 am

To Ron,
I personally feel that "who were born in the country" in E is wordy and awkward. (It does not include people "who are born in the
country")
The participle phrase "born in the country" in C is more concise and apt.
Besides Manhattan SC Guide explicitly writes "prefer an adjective to an adjective clause with be"
[page 214, 5th edition]

tim

ManhattanGMAT Staff

Re: The first detailed study of magpie attacks in Australia

Thu Jan 17, 2013 8:54 am

here's a good tip to keep in mind: if you EVER use the phrase "wordy and awkward" (or anything else that sounds similar) to explain
your reasoning on a SC question, you have done something wrong. there is ALWAYS a real reason why SC answer choices are wrong,
and "wordy and awkward" is NEVER that reason..

jlucero

ManhattanGMAT Staff

Posts: 1102

Joined: Wed May 12, 2010 1:33 am

Re: The first detailed study of magpie attacks in Australia

Thu Aug 01, 2013 12:08 pm

bish wrote:

RonPurewal wrote:

maribelsalazar02 wrote:I still can't tell why E is not correct (other than it's awkward).. what are the
rules that E is breaking? Thanks!

that choice is incorrect because it contains a modifier ("by the time they reached adulthood") that is blocked
off by a comma on one side, but not on the other side.
The main problem of choice e is that because "they" appears after the clause who, so they may refer to all people who were born in
the country but not those people who are attacked. Am I right?

RonPurewal
ManhattanGMAT Staff

Re: The first detailed study of magpie attacks in Australia

Mon May 26, 2014 11:56 am

"They" has the same function in that choice as it does in the correct choice, so no objection is possible there.

Whether "they" comes before or after its referent is immaterial, as long as the reference is clear.

eggpain24
Forum Guests

Re: The first detailed study of magpie attacks in Australia

Fri Aug 22, 2014 12:25 pm

Hi, Ron
in choice B

Is the use of non-essential modifier comma+who .....+comma"


change the intended meaning?

98 percent of men and 75 percent of women born in the country

98 percent of men and 75 percent of women, who were born in the country,

we aren't essentially talking about the same group of people, are we?

please clarify, thanks~

RonPurewal
ManhattanGMAT Staff

Re: The first detailed study of magpie attacks in Australia

Mon Oct 27, 2014 1:43 am

rustom.hakimiyan wrote:1) I originally picked B because I thought that the modifier "who were born in the country"
had to be separated by commas in order for it to refer to BOTH men and women. In option C -- "born in the country"
is touching "women" so I thought that the statement incorrectly implied that the attacks were related to men(not
necessarily born in the country) and women who were born in the country.

* punctuation isn't tested on this exam. end of story.


* if a modifier follows "x and y", common sense prevails in assigning the modifier.
e.g., Sheryl likes rock music and men with long hair --> it's clear that "with long hair" only applies to "men".
in this sentence, it's equally clear that "born in the country" applies to both men and women.

RonPurewal
ManhattanGMAT Staff

Re: The first detailed study of magpie attacks in Australia

Mon Oct 27, 2014 1:43 am

2) I realize that in choice B "had been attacked" is an event that happens in the (double past -- previous to another
past) and I thought that the preface of ("by the time" they reach adulthood), the "by the time" part implied that it
was a past event. I believe that's wrong because whenever we use "had been or had xxx" -- we need the other VERB
to be in the past, and not necessarily the context surrounding the other verb?

i got lost toward the end there, but, "by the time..." does not imply the past. that's a function of what comes afterward.

e.g.,
by the time i saw dr. smith, i had been sick for 5 days.
(i saw dr. smith in the past.)

by the time i see dr. smith, i will have been sick for 5 days.
(i will see dr. smith in the future.)
^^ in these instances, english omits "will" from the future. this has never been tested on the gmat.
the point, though, is that "by the time i see... definitely doesn't describe the past. it would be nonsense to use such a construction
to describe the present (we'd just use "now"!), so, it represents the future.

LaraZ595
Forum Guests

Re: The first detailed study of magpie attacks in Australia

Tue Jan 06, 2015 9:11 pm

Hi,

In choice E, can "who" modify both the men and the women? I think "who" can only modify one word in the sentence so it only
modifies women not men. Am I correct? Thanks!

RonPurewal
ManhattanGMAT Staff
anything that can describe a single noun can also describe "(noun) AND (other noun)".

Surya TejA527
Students
Re: The first detailed study of magpie attacks in Australia

Tue Nov 10, 2015 9:59 am

Hey Ron,
One last doubt.
E) 98 percent of men and 75 percent of women who were born in the country, by the time they reached adulthood had been
attacked by the birds.
DOUBT: Should I consider the modifier after "who" to modify only women or the whole sentence ( men & women)
Can I consider this as a decision point to eliminate E?

tim
ManhattanGMAT Staff

Re: The first detailed study of magpie attacks in Australia

Tue Nov 17, 2015 5:50 am

No, you cannot eliminate E due to this construction, mainly because the correct answer uses the same construction!

Crisc419
Students
Re: The first detailed study of magpie attacks in Australia

Fri Aug 05, 2016 7:04 am

RonPurewal wrote:

DennaMueller wrote:I'm still confused as to why 'B' could not be the correct answer. It used the past perfect
'had been'.

in (b), the past perfect is inappropriately matched with the OTHER verb, which is in the present tense.

either of the following two constructions would be appropriate:


...by the time they reach adulthood, they have been attacked...
...by the time they reached adulthood, they had been attacked...

you can't "mix and match" between these - i.e., reach + had been is wrong, as is reached + have been.

do we need "will" here?

by the time they reach adulthood, they will have been attacked...

thanks in advance.

Cris

RonPurewal
ManhattanGMAT Staff
Re: The first detailed study of magpie attacks in Australia

Tue Aug 09, 2016 9:05 am

that version would be grammatically correct, although of course it's nonsense (it's impossible for a study to make such an exact
prediction).

Crisc419
Students

Re: The first detailed study of magpie attacks in Australia

Tue Aug 16, 2016 1:34 am

RonPurewal wrote:that version would be grammatically correct, although of course it's nonsense (it's impossible for a
study to make such an exact prediction).

you didn't use "will", you wrote:


"...by the time they reach adulthood, they have been attacked..."

i ask whether we should add "will", because as you said before, here "by" indicates what happen in the future.
RonPurewal
ManhattanGMAT Staff

Re: The first detailed study of magpie attacks in Australia

Sat Aug 20, 2016 12:49 pm

"by the time they reach adulthood, they have been attacked..."
> GENERAL STATEMENT (no specific timeframe)

"by the time they reach adulthood, they will have been attacked..."
> statement about FUTURE events

in the second case, you might expect "by the time they will reach...", but no such thing exists in english. (if something after "if"
represents the future, then it's written in the same way as the present.)
Question 38

Question 38
https://gmatclub.com/forum/unlike-most-severance-packages-which-require-workers-to-128166.html

Difficulty: 15% (low)


Question Stats: 66% (01:31) correct 34% (00:35) wrong based on 1068 sessions
A:2% B:2% C:9% D:67% E:20%

MGMAT 1 --> 530


MGMAT 2--> 640
MGMAT 3 ---> 610
GMAT ==> 730

mourinhogmat1
Senior Manager

2 This post received KUDOS


1 This post was BOOKMARKED

Straightforward D, I thought.

D. their last scheduled day in order to collect, the automobile companys severance package is available to workers
E. the last day that they are scheduled to collect, the automobile companys severance package is available to workers

We can quickly narrow it down to D and E based on looking out for what we are comparing.

The problem with E is: which require workers to stay until the last day that they are scheduled to collect. The last part of this sentence to collect, seems
to leave the meaning lingering in mid air. We are forced to ask the question to collect WHAT? That's what is awkward with answer choice E. So, choose
D and move on.

OptimusPrepJanielle
Optimus Prep Instructor
Unlike most severance packages, which require workers to stay until the last day scheduled to collect, workers at the
automobile company are eligible for its severance package even if they find a new job before they are terminated.

In the original sentence, the severance packages are being compared to workers. Eliminate A, B, and C for making unequal
comparisons. We are looking for something following the comma that mentions the severance package of the automobile company.

D. their last scheduled day in order to collect, the automobile companys severance package is available to workers
E. the last day that they are scheduled to collect, the automobile companys severance package is available to workers
"the last day that they are scheduled to collect" changes the meaning of the original sentence. The original meaning is that in order
for workers to collect, they must stay until the last day scheduled. Choice E says that the workers must stay until the last day
they're scheduled to collect.

OptimusPrepJanielle
Optimus Prep Instructor

Hello Jesse.

Thanks for answer.


But I want to clarify: does we have pronoun mistake with word they in the answer E and if answer is yes then why in answer D
word their is not a problem

Yes, you are correct. In addition to the meaning of the sentence being distorted, "they" is somewhat ambiguous. "Their" is close
enough to "workers" to be understood as belonging to "workers" and not "severance packages."

https://www.veritasprep.com/blog/2015/09/master-the-gmat-by-applying-jedi-like-skills
Master the GMAT by Applying Jedi-like Skills
September 7, 2015

Once you begin studying for the GMAT, youll realize quickly that there are different levels of mastery. Theres that initial level of competence in which
you learn, or relearn, many of the foundational concepts that you learned in middle school and have since forgotten. Theres a more intermediate level of
mastery in which youre able to blend strategic thinking with foundational concepts.

Then theres the highest level in which you achieve a kind of trance-like, fugue state that allows you to incorporate multiple strategies to break down a
single complex problem and then seamlessly shift to a fresh set of strategies on the next problem, which, of course, will be testing slightly different
concepts from the previous one.

Its the GMAT equivalent of becoming a Jedi who can anticipate his opponents next light saber strike several moves in advance or becoming Neo in the
Matrix, finally deciphering the structure of the streaming code that animates his synthetic world. Pick whatever sci-fi analogy you like its this kind of
expertise that were shooting for when we prepare for the test. The pertinent questions are then the following: how do we accomplish this level of
expertise, and what does it look like once were finally there?

Fortunately for you, dear student, our books are organized with this philosophy in mind. Once youve worked through the skill-builders and the lessons,
youll likely be at the intermediate level of competence. Then it will be through drilling with homework problems and taking practice tests that youll
achieve the level of mastery we seek. But lets take a look at a Sentence Correction question to get a sense of how our thought processes might unfold,
once were functioning in full Jedi-mode.

Unlike most severance packages, which require workers to stay until the last day scheduled to collect, workers at the automobile company are
eligible for its severance package even if they find a new job before they are terminated.

(A) the last day scheduled to collect, workers at the automobile company are eligible for its severance package

(B) the last day they are scheduled to collect, workers are eligible for the automobile companys severance package

(C) their last scheduled day to collect, the automobile company offers its severance package to workers

(D) their last scheduled day in order to collect, the automobile companys severance package is available to workers
(E) the last day that they are scheduled to collect, the automobile companys severance package is available to workers

Having done hundreds of questions, youll notice one structural clue leap immediately: unlike. When you see words such as like or unlike you
know that youre dealing with a comparison, so your first task is to make sure youre comparing appropriate items. Youll also note that the clause
beginning with which require modifies severance packages, so whatever is compared to these severance packages will come after the modifier.

In A, youre comparing severance packages to workers. Wed rather compare severance packages to severance packages or workers to workers. No
good.

In B, again, youre comparing severance packages to workers.

In C, youre comparing severance packages to the automobile company. Nope.

That leaves us with D and E, both of which compare severance packages to automobiles company severance package. Here, youre comparing one
group of severance packages to another, so this is logical. But now you have to switch gears the comparison issue allowed you to eliminate some
incorrect answer choices, but youll have to use another issue to differentiate between your remaining options.

Once were down to two options, you can simply read the two sentences and look for differences. One difference is that E contains the word that in
the phrase the last day that they are scheduled to collect. Perhaps it sounds okay to your ear, but youll recall that when that is used as a relative
pronoun, it should touch the noun it modifies. In this case, it touches, last day. Read literally, the phrase, the last day that they are scheduled to
collect, makes it sound as though they are collecting the last day. Surely this isnt what the sentence intends to convey, so were then left with D,
which is the correct answer.

Takeaway:

Notice how many disparate concepts you had to juggle here: You had to recognize the structural clue indicating that unlike signifies a comparison;
recognize that temporarily skipping over a longer modifying phrase is an effective way to get a sense of the core clause youre evaluating; recall that
once youre down to two answer choices, you can simply zero in on differences between your options; remember the rule stipulating that relative
pronouns must touch what they modify; and last, you had to recognize that Sentence Correction is not only about grammar but also about logic and
meaning, and all in under a minute and a half. Id say thats pretty Jedi-like.
http://www.beatthegmat.com/unlike-most-severance-packages-t59165.html

Patrick_GMATFix GMAT Instructor

This is a comparison that begins with "most severance packages". The other side of the comparison, which comes after the "which"
clause, must be parallel to "most severance packages". The right answer must compare two types of severance packages. A, B and
C can be eliminated on this basis.

The difference between D and E one of meaning. Think about what the author is trying to say. the 2nd package is different from the
1st because the 2nd "is available to workers even if they find a new job before they are terminated". Since this is a contrast, the
author must be trying to say that the 1st package will not be available to workers who leave before they are terminated. In other
words, in the 1st package, if workers leave early they will not get their money. This is what D says: the package "requires workers to
stay until their last day in order to collect [money from the package]"

OA is D. A detailed solution can be found at GMATPrep question 2310

Hope that helps,


-Patrick

https://www.manhattanprep.com/gmat/forums/unlike-most-severance-packages-whiich-require-workers-to-t2001.html

StaceyKoprince
ManhattanGMAT Staff
Thu Jan 03, 2008 3:28 pm

I am glad to hear you don't study from 1000SC - it's pulled together from all kinds of sources and some are downright bad. And, hmmm, now that we
have proof they've lifted from OG / GMATPrep, we might just have to ban that source too... geez, don't any of these guys write their own questions
anymore? If you're really into standardized tests, it's a disgrace not to make your own - especially b/c it's fun to write questions! (If you're us, that is :)

Anyway, to your original question. Since you narrowed to D and E, I'm assuming you don't need any explanation on the apples-to-apples comparison
issue. The difference between D and E is one of meaning.

D says that some severance packages require workers to stay at the company until the last scheduled day they're supposed to work (say, next Friday) or
they don't get their severance (which may get paid that day or at some point in the future - we don't know).

E says that some severance packages require workers to stay until the day that they are scheduled to get their money - in other words, they stay until,
say, next Friday which is the day that they are actually paid.

Either could be the way things work (though D is probably more realistic, but we don't pay attention to that on SC). So we check the meaning of the
original sentence. The original says that these packages "require workers to stay until the last day scheduled to collect..." Could be confusing, because
that's grammatically incorrect in general, which muddles the meaning, but concentrate on "require workers to stay until the last day scheduled" - that
goes along with option D. The "to collect" here is missing "in order" which makes it a little harder to see what's going on.

But notice in E how it separates out the later clause - "the last day THAT they are scheduled to collect" - making this into a separate statement, so now
"scheduled" is paired with "to collect" instead of "last day scheduled" or (as in correct answer D) "last scheduled day."

RonPurewal
ManhattanGMAT Staff
Re: Unlike most severance packages, whiich require workers to

Mon May 18, 2009 7:16 am

sonu_gmat wrote:

because that's grammatically incorrect in general, which muddles the meaning

What's wrong with the grammar here.

if you say "the last day that they are scheduled to collect", this means that "day" must be a direct object.

this makes no sense, unless the employees are collecting days (which they clearly aren't).

--

another example:

"the time that we were scheduled to meet" is incorrect. the literal reading of this (which is the only reading you care about, by the
way) is that you were scheduled to meet the time itself. (ron: "hi, time!" time: "hi, ron!")

(in case you're wondering, the correct way to say this would be "the time at which we were scheduled to meet", since the
preposition "at" must be conserved.)
mcdionysus417
Forum Guests

Hi Ron
I am just wondering what "they" in E is referred to. Can "workers"(or people in general) be "scheduled"?
Thanks!! <3
Monica

jlucero
ManhattanGMAT Staff

Yes, it refers to workers, and the idiom is "scheduled to do X", which is entirely acceptable.

I was scheduled to teach a class this morning.

thanghnvn
Forum Guests

Re: Unlike most severance packages, whiich require workers to

Fri Jan 18, 2013 7:30 am

very hard.

in

the last day they are scheduled to collect


"they" refers to "packages"

and so

packages are scheduled to collect

is wrong because

"packages can not "collect"

is my thinking correct?

jlucero
ManhattanGMAT Staff

Correct.

pjreddy
Course Students

Re: Unlike most severance packages, whiich require workers to

Thu Mar 21, 2013 7:17 pm

I have a quick question: Is there anything incorrect with using "the last day scheduled..." versus "their last day scheduled.." This was
a split I noticed and used to eliminate answer choices. Was this a wrong move to make?
tim
ManhattanGMAT Staff

Re: Unlike most severance packages, whiich require workers to

Fri Mar 22, 2013 12:09 am

This was a wrong move if and only if you were unable to identify a specific reason why one of these options was incorrect. If you
cannot definitively rule out an answer choice as incorrect, you should leave it in play. Keep in mind that not all splits presented in
sentence correction have one right version and one wrong version. Sometimes there is nothing wrong with either option..

pjreddy
Course Students

Thanks Tim. That's a good point to keep in mind. I definitely always thought that splits were always put in for a reason. I still feel like
there has to be a logical reason behind why it makes sense to say "their last scheduled day" over "the last day scheduled to collect"
since we are referring specifically to the workers collecting THEIR severance packages. Can you explain why the latter would be
incorrect (if it is)?

tim
ManhattanGMAT Staff

Re: Unlike most severance packages, whiich require workers to

Sat Mar 23, 2013 1:01 am

the latter would be incorrect only if it failed to convey the meaning the sentence requires. notice that some of the choices that have
this construction provide that clarification later in the sentence, so in those cases the word "the" isn't a problem at all..

jlucero
ManhattanGMAT Staff

Re: Unlike most severance packages, whiich require workers to

Sat Sep 14, 2013 5:40 pm

mcmebk wrote:Hi instructors:

Sorry if my previous post was not clear, the question I have on the usage of "that" is:

Ron said in this structure "the last day that they are scheduled to collect", means they are scheduled to collect "the
last day", which is illogical;

However, we do see sentences like this: "The news that he was killed..." this is a valid construction, why that is ok
here since news can not be killed?

For D, I have a question about the usage of "in order to collect"...

I read many books in order to prepare the upcoming exam - It is my purpose to prepare the exam

I talked to him to understand better the questions - It is my purpose to understand better the questions.

I require a better car to drive faster - I want to drive faster...etc


i suppose so -- but no problem will ever depend on anything nearly as subtle as that!
Question 39

Question 39
https://gmatclub.com/forum/today-s-technology-allows-manufacturers-to-make-small-cars-21772.html

Difficulty: 65% (hard/medium)


Question Stats: 50% (01:42) correct 50% (00:57) wrong based on 1542 sessions
A:10% B:15% C:51% D:11% E:14%

chetan2u
Verbal Forum Moderator

gmatser1 wrote:
Can someone explain why E is wrong?
Hi,
two reasons why E is wrong..

Today's technology allows manufacturers to make small cars more fuel-efficient now than at any time in their production history.
5) more fuel-efficient small cars now than at any time in

1) more fuel efficient small cars means different from 'small cars that are more fuel efficient'...
the comparison shifts from fuel efficiency to more number of fuel efficient small cars, illogically..

2) now is also a time in the production history..


so at any time should change to at any other time..

hope it helped

sayantanc2k
Verbal Expert

qingping wrote:
what does "those" stand for
"Those" stands for "small cars". Small cars manufactured today are more fuel efficient than small cars (those) at any other time.

sayantanc2k
Verbal Expert
manlog wrote:
sayantanc2k wrote:
qingping wrote:
what does "those" stand for ?

"Those" stands for "small cars". Small cars manufactured today are more fuel efficient than small cars (those) at any other time.

sayantanc2k, does answer B has a correct referral "they"? I am stuck with they + verv vs. those

"Those" is preferred in this case, because a "different copy" of "small cars" is required. We are not referring to the same "small cars that are more fuel
efficient", but to a "different copy" of the "small cars", which were produced at any other time in production history. "They" could be used if we were
referring the same cars. Another example may make the mistake more visible:

On the bridge I saw cars that are bigger than those in front of the house.... right
On the bridge I saw cars that are bigger than they are in front of the house....... wrong.

Therefore answer B is wrong.

https://www.manhattanprep.com/gmat/forums/today-s-techonology-allows-manufacturers-to-make-small-cars-t354.html
GMAT 5/18

For this question, I began by eliminating all choices ending in "their" - manufacturers require "it" or something similar, but definitely
not "their". This left answer choices c. and e. Then, I chose c. over e. because of 2 reasons:

1. e. says that manufacturere are making "more" f-e cars now, and I feel this changes the meaning of the original sentence
2. e. has the wording "now" which I think is redundant as "today's" has already been stated

Is my logic and reasons for choosing c. over e. correct, or did I just get lucky (correct answer is c)? Any help would be appreciated.
Thanks!

dbernst
ManhattanGMAT Staff

Good work identifying the pronoun error. To clarify, however, it is not an error in number (it v. their) but an error in ambiguity. In this
case, "their" could refer either to "manufacturers" or "cars"; thus, we can eliminate any answer choices that include "their."

I also agree that the primary error in (e) is redundancy - Because the sentence is discussing "today's" technology it is redundant to
say "now." I also think that the meaning of (e) is ambiguous: are manufacturers making cars that are more fuel efficient, or are
manufacturers simply making more cars (number of cars)?

-dan

RonPurewal
ManhattanGMAT Staff
Re: Today's techonology allows manufacturers to make small cars

Thu Jan 07, 2010 8:21 am

andy_11_30 wrote:Ron once said that the test takes some level of pronoun ambiguity
Here,

I see that we are struggling to set a referrent for THEIR..


Can't their refer to MANUFACTURERS logically ???

Besdies, I very well know that the answer should NOT include THEIR>.

But just a thought crossed, please explain...!!!

yeah. i don't think that pronoun ambiguity is the criterion here.

the sentence is supposed to say that today's small cars are more fuel-efficient than previous small cars -- i.e., OTHER small cars.
this is important, because you can't use a "they"/"their" construction (which would illogically imply that you're talking about the
same small cars mentioned in the first part of the sentence).
the use of "those" in (c), on the other hand, accomplishes this distinction nicely.

analogy:

sprinters in texas can run faster than they can in north dakota.
--> illogical; in this sentence, "they" would be taken to mean "sprinters in texas" (i.e., not just sprinters).

sprinters in texas can run faster than those in north dakota.


--> makes sense; "those" = sprinters, in this case.

same thing with "they/their" vs. "those" in these instances.

RonPurewal
ManhattanGMAT Staff

Re: Today's techonology allows manufacturers to make small cars

Sat Jul 31, 2010 5:14 am

sandeep.19+man wrote:Quick question:

Today's techonology allows manufacturers to make small cars that are more fuel-efficient than they were at any time
in their production history.

Keeping in mind that pronoun ambiguity is not the criterion here. Does they in option (b) refer to

1. small cars
OR

2. small cars that are more fuel efficient

it's not really either of these.


* it's not #1, because we are clearly not talking about all small cars; we're only talking about the small cars that obey the
comparison given.
* #2 is not a proper grammatical analysis -- the comparison itself is contained in the modifier, and you can't split it in half. (i.e., you
can't look at "more fuel efficient" as divorced from "than..."; that just doesn't make any sense.)
the best way to look at it is to say that "they" refers to the same small cars that are referenced by the preceding noun -- i.e., exactly
those small cars that satisfy the given comparison.

if you want to be exact about it:


in the construction "NOUN that are more ADJ than it/they was/were...", the pronoun "it/they" stands for exactly those NOUNs that
satisfy the comparison given.
example:
taxpayers who make more than twice as much income as they did last year are subject to special scrutiny from the IRS.
this is a properly written sentence; in it, "they" stands for the taxpayers who satisfy the condition given here -- i.e., those taxpayers
who make more than twice as much income as they did last year, and ONLY those taxpayers. not all taxpayers in general.

--

the problem with this choice is that it's not true -- we are not talking about the same small cars! in this sentence, we're talking
about small cars that are more fuel-efficient than previously manufactured small cars, so this sort of pronoun doesn't work here.
similarly, in the example sentence above, we could not use "they" to stand for a different group of taxpayers who had paid a lower
amount of tax.

RonPurewal
ManhattanGMAT Staff
Re: Today's techonology allows manufacturers to make small cars

Sat Jul 31, 2010 5:19 am

@ mohitkant

mohitkant wrote:I caught on to the "those" difference and narrowed my options to C and D .

c. small cars that are more fuel-efficient than those at any other time in
d. more fuel-efficient small cars than those at any other time in their

I eventually chose D over C because i thought that saying any other time in production history because of two reasons.

1. "any other time" i felt was irrelevant, history itself indicates that we are talking of a time before the current time. Hence saying
any other time is perhaps not required.

are you sure you're discussing the correct answer choices? you are talking about the words "any other time" as incorrect/irrelevant,
but you seem to be discussing 2 answer choices that both contain those words.

2. I felt "their" was required since we needed to clarify whose production history. I mean are we talking about Production History of
Cars in General or Production history of Cars manufactured by Various Manufacturers.

if a noun is preceded by adjectives, then any pronoun referring to that noun MUST refer to the complete package of adjectives +
noun.
so, in choice (d), "their" must refer to "fuel-efficient small cars"; that doesn't make sense, since fuel-efficient small cars have not
been made throughout the entire production history in question.

also, another problem with choice (d) is the ambiguity in the phrasing "more fuel-efficient small cars". this could mean what the
other choices are saying (i.e., small cars that are more fuel-efficient), but it could also mean "a greater number of fuel-efficient small
cars". the latter, which is probably the way in which most people would initially read this version, is incorrect.

RonPurewal
ManhattanGMAT Staff

@ alvin1839

alvin8139 wrote:Excellent explanation! Only one further question to help me understand better: Why can't I understand choice A as below:

'small cars' is a noun phrase that's not specific meant for today's or previous, but just meant to say small cars in general.
And I compare 'now' with 'at any time'; I chose the wrong choice A, because I ever remembered that there were other prep questions that actually
compares 'time';
'their' just refer to the 'small cars' in general.

unfortunately, that's not the way it works.


ironically, the curse here is that we humans are too smart: we can use context to figure out what the sentence is actually supposed to say, even if that's
not what it actually says.

for instance, look at my (correct) example with taxpayers in the following post:
post42496.html#p42496
in that case, it should be obvious that the pronoun does not stand for taxpayers in general.
the same holds for the choice that you are analyzing.

Can I say if I see answer choices split 'those/that' vs 'they/it' in comparison Questions, 'those/that' are always correct.

nope.
in SC in general, if you try to formulate any rule that says "word X is always correct, and word Y is always incorrect", independently of context, then,
99.99% of the time, your rule will be invalid.

joannat
Forum Guests

Re: Today's techonology allows manufacturers to make small cars

Sun Aug 22, 2010 7:44 pm

Hi Ron,

First I want to thank you for your help. It's just amazing to see that you have been spending so much time and effort replying to our
questions.

I have a question, in this post, since-1990-the-global-economy-has-grown-more-than-it-did-t552.html, you mentioned that by using


"that/those", the sentence structure needs to be exactly parallel.

if the second half says 'that during 10,000 years', then the preceding half must say 'the growth of ___ during something else'

Now in this question:


Todays technology allows manufacturers to make small cars that are more fuel-efficient than those at any other time in
production history.

If we use those here, aren't we missing something like "at XXX time" in he preceding part (small cars)?
Thank you!

tim
ManhattanGMAT Staff

Re: Today's techonology allows manufacturers to make small cars

Mon Sep 20, 2010 2:50 pm

Not really. The context of the sentence ("today's" and "are") is sufficient to inform us that we are dealing with now rather than a
point in the past..

RonPurewal
ManhattanGMAT Staff

Re: Today's techonology allows manufacturers to make small cars

Thu Nov 25, 2010 6:16 am

sheetal_smp wrote:Hello Ron,

I watched the videos on Study Hall and want you to thank you for all the effort you took to explain the SC
fundamentals so well.

thanks
RonPurewal
ManhattanGMAT Staff

Re: Today's techonology allows manufacturers to make small cars

Sun Dec 15, 2013 1:06 pm

Choice A doesn't have that issue. ("Their" in choice A is fine.)

The problem with choice A is that it suggests that manufacturers are retooling existing small cars, rather than creating new ones
that are better.

E.g.,
I'm going to make air conditioners quieter. --> I'm going to modify existing air conditioners so that they make less noise.

I'm going to make quieter air conditioners.


I'm going to make air conditioners that are quieter.
--> I'm going to make a new model that makes less noise.

A is like the first one of these, so it doesn't make sense in context.

Haibara
Forum Guests
Re: Today's techonology allows manufacturers to make small cars

Thu Feb 27, 2014 1:59 am

Ron, I've read all this whole thread. Your elaborations are fabulous.
However, I still have several questions and I also want to range in order all the errors in each choice, which scatter here and there in
different corners of this thread.

I initially choose A.
#1 Choice A is grammatically fine, right? "they" in choice A ,without doubt, refers to "manufacturers" and assumes no ambiguity,
right?
The problem with choice A is more of meaning than of grammar?
Choice A seems to imply that manufacturers have recalled all the smalls cars currently on the road and start installing these recalled
small cars ,perhaps, with some new engines, to make them now running more fuel-effiently than ever before. Choice A mentions
nothing about the new fuel-efficient cars that manufacturers are now producing, right? It diverges from what GMAC want to convey
in this question, so Choice A is wrong.

To imitate choice AI made up two sentences as below

The new computer-based system allows GMAC to make GMAT tests more difficult now than at any other time in its history.

The new computer-based system allows GMAC to make GMAT tests more difficult now than they were at any other time in its
history.

Lees championship in the 500meters Short Track Speed ""Skating Final at the 2014 winter Olympic Games makes her mother
prouder than at any other moment in her mother's life.

Lees championship in the 500meters Short Track Speed ""Skating Final at the 2014 winter Olympic Games makes her mother
prouder than she was at any other moment in her mother's life.
Are above sentences all correct, grammatically and meaningly?

#2 "they" and "their " in choice B grammatically have to refer to the same antecedent? Whether the antecedent is "small cars" or
"manufacturers", they can't make sense either way. So B is wrong.

#3 About the OA, I have the same question as joannat, whose post I now quote below and whose question, from my perspective,
has not been positively addressed by Tom.

joannat wrote:Hi Ron,

First I want to thank you for your help. It's just amazing to see that you have been spending so much time and effort replying to
our questions.

I have a question, in this post, since-1990-the-global-economy-has-grown-more-than-it-did-t552.html, you mentioned that by


using "that/those", the sentence structure needs to be exactly parallel.

if the second half says 'that during 10,000 years', then the preceding half must say 'the growth of ___ during something else'

Now in this question:


Todays technology allows manufacturers to make small cars that are more fuel-efficient than those at any other time in
production history.

If we use those here, aren't we missing something like "small cars now" in the preceding part ?

Thank you!

Also, If I add "were" in choice C as below:


Today's techonology allows manufacturers to make small cars that are more fuel-efficient than were those at any other time in
production history.
Is it still correct?

Ron, sorry for such a long post and appreciate every word of your reply. Thank you very much.

RonPurewal
ManhattanGMAT Staff

Re: Today's techonology allows manufacturers to make small cars

Thu Feb 27, 2014 9:04 pm

That seems ok to me

RonPurewal
ManhattanGMAT Staff

Re: Today's techonology allows manufacturers to make small cars

Sun Mar 02, 2014 5:23 am

thanghnvn wrote:

GMAT 5/18 wrote:Source: Gmat Prep, mba.com, Test II

Today's techonology allows manufacturers to make small cars more fuel efficient now than at any time in their
any other time in production history.

Is the sentence correct?? If yes, how can be omit the verb "were"??

If the sentence already contains explicit time cues, that's an adequate substitute for the tense change.

E.g.,
Today's soldiers are much taller and heavier than the soldiers of World War I.
WWI is a past event, so "were" isn't absolutely necessary.

On the other hand, I'm shorter than my father means I'm shorter than he is now. If I want to compare my current height to my
father's past height, I need to write ...than my father was [in xxxxx timeframe].

RonPurewal
ManhattanGMAT Staff

Re: Today's techonology allows manufacturers to make small cars

Wed Oct 29, 2014 3:43 am

rustom.hakimiyan wrote:Hi Ron,

I understand the previous posts where you mention that they/their refers to the same cars and therefore it's
incorrect. On the flip side, I read choice B as:
"Todays tech allows manufacturers to make cars more fuel efficient than small cars were" -- I read this as the same
types of cars but different engines and therefore, I found this to be correct. Why is my reasoning flawed? Am I
inferring too much here?

the whole point of "they" is to refer to the same cars specified earlier.

analogy:
pets tend to make people happier than they would otherwise be.
--> clearly the same people.

by the same token, "more xxxx than they were..." implies a previous state of the same vehicles.

RonPurewal
ManhattanGMAT Staff

Re: Today's techonology allows manufacturers to make small cars

Wed Oct 29, 2014 3:45 am

in fact...
here, you're trying to isolate the concept of "small cars", WITHOUT the context in which it first appears.

this is the exact reason why the pronouns "that" and "those" exist!
"it", "they", etc. keep all of the original context.
"that", "those" don't; they're used to place the same noun into a DIFFERENT context (as comparison sentences are wont to do).

in fact, if you just take your own explanation up there--still "small cars", but from a different period in history--you have a pretty
good summary of how we use "those" in comparisons.

RonPurewal
ManhattanGMAT Staff

Re: Today's techonology allows manufacturers to make small cars

Wed Oct 29, 2014 3:51 am

rustom.hakimiyan wrote:By that token, I eliminate E because it has "more fuel-efficient small cars now than at ..." --
the "now" throws it off for me because of the above reasoning. Am I way off here?

if you have
... now + than + (other timeframe)
...then the sentence can work, given a sensible context. one timeframe, other timeframe.

e.g., i'm about to give someone a really strong medicine. he is going to feel horribly ill now, but won't ever feel that ill again.
You'll feel sicker now than [b]at any later time.
RonPurewal
ManhattanGMAT Staff

Re: Today's techonology allows manufacturers to make small cars

Wed Oct 29, 2014 3:51 am

it's easier to kill "now" simply because it's redundant, given that "today's..." is already there.
(since "now" is sometimes present and sometimes absent, redundancy should be on your radar; you don't need to see this cold.)

tim
ManhattanGMAT Staff

Re: Today's techonology allows manufacturers to make small cars

Tue Sep 22, 2015 10:41 am

An explanation of why something is correct would not be appropriate. If you find yourself trying to explain why something is correct,
you're totally going about sentence correction the wrong way. SC is all about finding errors and process of elimination. If you have
an iron-clad reason why something is wrong (i.e. you can refer to a specific page number in the SC strategy guide for the rule that
makes it wrong), then eliminate that answer choice. What's left after you've eliminated four answer choices is correct, for no other
reason than because the GMAT says so.
RonPurewal
ManhattanGMAT Staff

Re: Today's techonology allows manufacturers to make small cars

Sun Mar 27, 2016 11:25 am

"stand-alone" meaning, basically, "not part of a larger phrase (in a comparison)".

here's a CORRECT use of "those" as a pronoun in a comparison:


The students at School X are about a year older, on average, than those at School Y.

we CAN'T use "they" here, because "they" would have to stand for "the students at school x" (not just "the students").
this is the reason why we have the pronouns "that" and "those" for comparisons -- so we can pull nouns out of larger phrases,
WITHOUT all the attached qualifiers/modifiers/adjectives/etc.

basically, if you CAN use "they", then you can't use "those". (similarly, with singular nouns, if you can use "it" then you can't
use "that".)

minhux383
Students

Re: Today's techonology allows manufacturers to make small cars

Thu Aug 25, 2016 12:01 pm

KwakuA962 wrote:
Question 40

Question 40
Question 40

Question 40
https://gmatclub.com/forum/in-the-mid-1920s-the-hawthorne-works-of-the-western-electric-87557.html
A:5% B:56% C:4% D:26% E:9%
Difficulty: 55% (hard/medium)
Question Stats: 55% (01:42) correct 45% (00:49) wrong based on 1290 sessions

EducationAisle
Director

Actually 'that' belongs to a category called 'essential' modifiers. Essential modifiers can surprisingly modify either the word or the
phrase before them, whatever makes sense; this is as opposed to non-essential modifiers such as 'which', which (almost always)
modify the word preceding those modifiers.

B expresses the meaning in a most lucid manner and is hence the correct answer. Usage such as "working conditions effects " or
"what the effects changes in working conditions" are clearly awkward.

egmat
e-GMAT Representative

Hi jrashish,

Thanks for posting your doubt here. We appreciate your effort.

Yes, you are correct that in the correct answer choice B, the verb-ing modifier modifies the preceding noun entity "series of
experiments". Yes, it is kind of a bit odd to say that the "series of experiments" did the investigation about something. But it is
certainly not incorrect. This expression is absolutely acceptable on GMAT.
Hope this helps.
Thanks.
Shraddha

egmat
e-GMAT Representative

WoundedTiger wrote:
Hello Experts,

Please chip on this one....This is GMAT prep question and I am bit worried on this

Hi WoundedTiger,

I agree that it sounds a little awkward to say that "a series of experiments" investigated the effects. However, this is the correct answer choice of an
official question. Hence, we must understand that such expressions are acceptable.

The suggestion that you have made - to put a comma before "investigation" - will certainly lead to a correct construction. But the meaning will be
different from the one the correct answer conveys now. And you certainly understand that. And the other choices do contain grammatical errors that
make Choice C correct.

Hope this helps.


Thanks.
SJ
sayantanc2k
Verbal Expert

"Changes" is noun - it is the subject of the "that" clause. Here "that" is the object of the clause and "would have" is the verb:

Changes in working conditions would have effects (that) on workers performance.

http://www.beatthegmat.com/in-the-mid-1920s-the-hawthorne-works-of-the-western-electric-t282520.html

GMATGuruNY GMAT Instructor

In A, it is unclear whether their refers to changes or to conditions.


Eliminate A.

C: The Hawthorne Works WAS the scene...for investigating...what the effects...ARE.


Here, the sequence of events is illogical:
It is not possible that the Hawthorne Works WAS the scene (in the past) for investigating what the effects ARE (in the present).
Eliminate C.

D: changes in working conditions' effects


Here, there are changes in the EFFECTS.
The intended meaning of the original sentence is that there were changes in the WORKING CONDITIONS THEMSELVES.
Since D does not convey the intended meaning, eliminate D.
Construction: what + the + NOUN.
In this construction, what generally serves as the DIRECT OBJECT of a subsequent verb, while the + NOUN serves as the SUBJECT of
this verb.
The director does not understand what the author of the graphic novel intended.
Here, what serves as the direct object of intended, while the author serves as the subject of this verb.
Conveyed meaning:
The author intended WHAT.

E: what the effects changes in working conditions would have on workers' performance
Implication of the portions in red:
The effects would have WHAT.
Not the intended meaning.
Eliminate E.

The correct answer is B.

GMATGuruNY GMAT Instructor

RBBmba@2014 wrote:
A quick clarification -
Is this Construction(what + the + NOUN) strictly followed in GMAT ? I mean, couldn't the DIRECT OBJECT you mentioned be "what the effects"
together ?
If what the effects serves as a direct object, we get:
changes in working conditions would have WHAT THE EFFECTS on workers' performance.
This meaning makes no sense.

Quote:
Also, if we tweak the option E a bit "to investigate what effects changes in working conditions would have on workers performance" , then wouldn't it
Those skeptical of the extent of global warming argue that short-term temperature data are an inadequate means of predicting
long-term trends and point out that the scientific community remains divided over whether significant warming will occur and what
impact it would have.

I think "to investigate what the X is that S V" is different from "to investigate what X S V" . the former is redundant, but the latter
is not.

what is your opinion?

RonPurewal ManhattanGMAT Staff


Re: In the mid-1920s the Hawthorne Works

Sat Dec 19, 2009 8:18 am

I think "to investigate what the X is that S V" is different from "to investigate what X S V" . the former is redundant,
but the latter is not.

you're right, but "the X that S V" (the structure of the correct answer to this problem) is even better.

in the problem you've cited, "what impact..." is the best choice largely because it's parallel to another question-word ("whether...").
i.e.,
you couldn't place "the impact that..." in parallel to "whether...".
"what impact...", on the other hand, is perfectly parallel to "whether...".
mayank
Course Students

Ron, I am a little confused about the explanation why b is correct. According to another post on this forum, Prep. of + Noun +
VerbING is considered incorrect. Doesn't that same principle apply here?

See post:
post26678.html

RonPurewal ManhattanGMAT Staff

post26678.html

Re: As the honeybees stinger is heavily barbed

Wed Jun 24, 2009 7:08 am

bluementor wrote:Question from GMATPrep 2:

As the honeybees stinger is heavily barbed, staying where it is inserted, this results in the act of stinging causing the
bee to sustain a fatal injury.

A. As the honeybees stinger is heavily barbed, staying where it is inserted, this results in the act of stinging causing
B. As the heavily barbed stinger of the honeybee stays where it is inserted, with the result that the act of stinging
causes
C. The honeybees stinger, heavily barbed and staying where it is inserted, results in the fact that the act of stinging
causes
D. The heavily barbed stinger of the honeybee stays where it is inserted, and results in the act of stinging causing
E. The honeybees stinger is heavily barbed and stays where it is inserted, with the result that the act of stinging
causes
OA is E. My queries to the instructors are:

1)Is E correct because of parallelism? i.e. is//stays//causes?


2) In A, B and D, is 'results in..' unidiomatic?
3) What is this sentence trying to convey really? I think I spent about 2.5 mins on this question trying to get to the
gist of it. How should one split the answer choices?

Thanks,
-BM-

first off, the construction (preposition) + NOUN + VERBing is WRONG, unless the preposition refers directly to the
NOUN. (that isn't usually the case, so, if you're in doubt, you should strike choices with this sort of construction.)

for instance:
i've never heard of bees stinging dogs
WRONG. this is not an issue of whether you've heard of bees themselves; it's an issue of whether you've heard of their stinging
dogs.

...results in the act of stinging causing...


WRONG. this doesn't result in the act of stinging itself; it results in what is caused by the act of stinging.

i have a picture of my cousin playing hockey.


CORRECT. this time, the picture is actually of my cousin, so we're good.

therefore, (a) and (d) are wrong because of "...results in the act of stinging causing...".

--
the pronoun "this" in (a) doesn't refer to any particular noun. this consideration also kills (a).

--

you can't say "the stinger results in...".


"results in..." can only be used when it's LITERALLY TRUE. for instance, you could say that the attempt resulted in failure, since the
attempt ITSELF ended in failure.
if you understand this literal meaning, then it goes without saying that you can't use this sort of construction for physical objects.
TAKEAWAY:
you can only say "X results in Y" when X is an ACTION. if X is an OBJECT, you can NEVER say that X "results" in
anything.
this kills choices (c) and (d), in which "stinger" is the subject of the verb "results".

--

choice (b) misuses the connector "as".


the connector "as" connects two complete sentences BY ITSELF. if "as" is used to connect two complete sentences, it
should NOT be used in conjunction with any other connector words.

in choice (b), "as" and "with" are used together. the use of either of these prohibits the use of the other, so this choice is wrong.

--

not sure what you're asking in #3. it looks like at least 2 completely different questions.

the MEANING of the sentence is that the bee's stinger stays where it's placed, and so the bee dies because it can't get away after
the sting.

as far as HOW TO SPLIT, you'll notice that i've given several different angles from which to approach this one.
there aren't really genuine "splits" - i.e., easily identifiable points of divergence between clearly parallel items - so you just have to
try to identify any of the multitude of errors present.

RonPurewal ManhattanGMAT Staff

Re: In the mid-1920s the Hawthorne Works

Wed Jun 29, 2011 6:26 am

mayank wrote:Ron, I am a little confused about the explanation why b is correct. According to another post on this
forum, Prep. of + Noun + VerbING is considered incorrect. Doesn't that same principle apply here?

See post:
post26678.html

you can't just memorize that result; you have to be able to understand how to process that sort of construction, which is correct in
some circumstances and incorrect in others.

in this case, the construction is perfectly fine, because the intended object of "of" is actually "experiments" (not the action of
investigating).
therefore, this construction is analogous to the CORRECT example given in that thread ("i have a picture of my cousin playing
hockey"), not to the incorrect ones.

ShyT Course Students


Re: In the mid-1920s the Hawthorne Works

Wed May 09, 2012 2:19 am

after reviewing this problem I am trying to apply an unorthodox method that seems to work for me sometimes, especially with
meaning/modifer sentences. I was just hoping if one of the instructors were to give me the green light on whether my reasoning is
correct.

For this specific problem

In the mid-1920s the Hawthorne Works of the Western Electric Company was the scene of an intensive series of experiments that
would investigate changes in working conditions as to theireffects on workers' performance.

I try to weave out all of the unimportant modifiers to make sense of the true sentence and before the underlined portion I am left
with this.

The H. Works was the scene ________

A,D crossed out, have "that investigate" seems like would mean that the scene is doing the investigation.

As stated above C is just terrible wordy,

So left with B&D and again "to investigate" (filled in my underline portion) also seems almost like.. who is investigating?

that's why I chose B?

Is this approach logical or did I simply get lucky? can I reuse it?
Thanks

RonPurewal
ManhattanGMAT Staff

Re: In the mid-1920s the Hawthorne Works

Thu May 17, 2012 10:23 am

ShyT wrote:A,D crossed out, have "that investigate" seems like would mean that the scene is doing the investigation.

no. why wouldn't you just apply "that" to the noun phrase right next to it?
a series of experiments that investigated...

this is still somewhat problematic, in the sense that people (not experiments) have to "investigate" things, but the issue is not what
you have written here.

As stated above C is just terrible wordy,

you should NEVER judge the "wordiness" of an INDIVIDUAL answer choice.


you should judge "wordiness" only RELATIVE TO OTHER CHOICES.

in other words, you should never just say "(c) is wordy". instead, it should always be "(c) is MORE wordy than [other choice]"
jlucero
ManhattanGMAT Staff

Re: In the mid-1920s the Hawthorne Works

Fri Aug 24, 2012 11:02 am

vijay19839 wrote:Ron

Can we eliminate Options A & D with the below logic:-

Option A:- "Experiments that would investigate changes.."


Option D:- "Experiments that investigated changes..:

Both means that Experiments will be investigating the changes and looks nonsensical.

(As per your One of the classes in Thursdays with Ron, This rule wasn't applicable for few words such as 'Idea',
'Notion' etc as the entire 'that clause' is the idea or Notion..

"The Idea that X.." or "The Notion that Y..."

Thanks
Vijay

Not in this case, Vijay. Although the people behind the experiments are the ones investigating the changes, you can use words like
experiments, tests, or results to express an active idea.

The experiments found that...


The tests show that...
The results clearly demonstrate that...

Which mean that these are the experiments that found something, tests that show something, and results that clearly demonstrate
something else.

jlucero
ManhattanGMAT Staff

Re: In the mid-1920s the Hawthorne Works

Sat Sep 14, 2013 1:48 pm

mcmebk wrote:Hi Ron

I think there are two more obvious problems with A:

1) Investigate changes in working conditions - Wrong, what the experiment actually investigated was the effects, not
changes;

2) Would investigate... - Wrong; would can be either used as a past tense of will (not applied here) or a sense of likely
in a hypothetical situation (not applied here either); the correct usage of the word "would" here is shown in sentence
2, the conditions would have...since it is a to be discovered results by the time the experiments started investigating.

Thank you Ron for everything


There's definitely lots of ways to eliminate answer choices. I'll definitely agree with you on the second point, but I think your first
point bleeds into what Ron mentioned. The sentence would be fine saying "experiments that would investigate changes AND their
effects on..." As Ron said, the "as to their effects" is an awkward way of trying to talk about the changes and effects together. You
are right that the effects are important, but I don't think you could say that the changes are necessarily unimportant.

RonPurewal
ManhattanGMAT Staff

Re: In the mid-1920s the Hawthorne Works

Sun Sep 22, 2013 4:36 am

imhimanshujaggi wrote:Hello Instructors,


I would like to understand the Subject Verb Agreement in the Main Clause.

Subject: The Hawthrone Works


Verb: Was
Isn't it a forward construction and verb should be "were" instead of "was" ?

"The Hawthorne Works" is a company, so it's singular. It's not more than one company.

Similarly, "the United States" is also singular, because it's a single country.
The United States has approximately 300 million citizens --> Yes.
The United States have approximately 300 million citizens --> No.
You probably know this already. "Hawthorne Works" is similar. One entity, so, singular.
It's probably not a coincidence that the verb isn't underlined. They wouldn't make this an issue, unless there were some other proof
elsewhere (e.g., another singular verb, or the pronoun "it", or something like that) that this noun is singular.

RonPurewal ManhattanGMAT Staff

Re: In the mid-1920s the Hawthorne Works

Mon Apr 14, 2014 9:26 am

m1a2i3l wrote:Hi Ron & other experts,


i have a question here.
Does the phrase 'an experiment investigating XX' technically differ from the phrase 'an experiment to investigate XX'
Can 'to investigate' modify the experiment?

I'd say this is a non-difference.

RonPurewal ManhattanGMAT Staff

Re: In the mid-1920s the Hawthorne Works


Wed Nov 05, 2014 4:43 am

chetan86 wrote:

experiments investigating the effects that changes in working conditions would have on workers'
performance.

Could you please explain the role of "changes in working conditions" in this sentence?

it's a noun (+ modifier). it's the subject of "would have" (whose object is "changes").

if you don't see how this works, just replace the boldface stuff with a shorter noun, like "things".

RonPurewal
ManhattanGMAT Staff

Re: In the mid-1920s the Hawthorne Works

Wed Nov 05, 2014 4:46 am

chetan86 wrote:If I remove "changes in working conditions" then below sentence makes sense?
xxx experiments investigating the effects that would have on workers' performance.
nope. this one is nonsense.
"have" needs an object.
Remy has. > not a sentence. (he has what?)
Remy has stuff. > sentence.

puntkub
Students

Re: In the mid-1920s the Hawthorne Works

Sun Jan 24, 2016 3:57 pm

Hi Instructor,

i have 2 questions would like to ask below.

1.) are these three phase technically different?


(B) investigating
(D) that investigated
(E) to investigate

2. if i changed choice (D) from


(D) that investigated changes in working conditions' effects on workers' performance
to
(D) that investigated the effects that changes in working conditions would have on workers' performance
is this correct?

Thank you
Punt

tim ManhattanGMAT Staff

Re: In the mid-1920s the Hawthorne Works

Sat Apr 09, 2016 9:17 pm

1. If you use different words, then the result is technically different.

2. I would caution you against ever asking "what if" questions about changing parts of verbal questions, because there are often
several interconnected parts that cannot be fully accounted for by a single change. Just focus on why the right answer is right and
why all the wrong answers are wrong.
https://www.manhattanprep.com/gmat/forums/post41381.html#p41381

eyunni

experiments investigate???

Thu Feb 14, 2008 5:25 pm

How can experiments investigate on their own? Hawthorne Works of the Western Electic Company investigated the changes by
experimentation. Right? If so, how is B correct? Please explain.

StaceyKoprince
ManhattanGMAT Staff

Fri Feb 15, 2008 1:49 am

The sentence is not saying that the experiments investigated on their own. "experiments investigating," in answer B, is not a subject-verb setup -
"investigating" is not a verb. It is a modifer explaining what type of experiments were being done. What type of experiments? Ones that investigated the
effects etc.

Words ending in -ing can be verbs, nouns, adjectives, adverbs, participial phrases - they're pretty versatile. Unless you see some form of the verb "to be"
immediately preceding the -ing form, it is not functioning as a verb. It's some other part of speech.

RonPurewal
ManhattanGMAT Staff
Mon Feb 18, 2008 5:20 am

eyunni wrote:Stacey, I understand that 'investigating' here is not a verb. You said: 'What type of experiments? Ones that investigate the
effects etc.'

What does 'Ones' refer to? 'Ones' refer to experiments. Correct? Then do you mean: 'Experiments that investigate the effects'?

i think both of you are right. 'investigating' isn't a verb per se, here - it's a participle that's used as an adjective, like stacey said before - but it's used in a
sense that's fapp* equivalent to 'experiments that investigate...'.

the final word on issues like this is to throw up our hands and say, 'that's the way the gmat uses the word.' remember that you're learning to speak a
second (or third, or ...) language here, called gmat-speak, and that language sometimes uses words in ways that differ from what you're used to.

so: store, in your brain, the fact that the gmat considers this meaning of 'investigate' (i.e., experiments / tests / etc. can 'investigate' topics) to be perfectly
ok. by the way, some brief searches on the internet reveal that lots of reputable sources also use the word in the same way (and nothing strikes me as
unsavory about the usage, if my personal opinion is worth anything)

* = for all practical purposes

RonPurewal
ManhattanGMAT Staff

Re: GMATPREP- Hawthorne Works


Fri Jul 02, 2010 3:56 am

roshan_aslam_engg wrote:the effects that changes in working conditions. is this not a S-V problem? or is 'changes'
used as a noun here.

it's a noun.

if yes, do you have any tip on how to determine wether a word is a noun or a verb. ( i mean for these kind of verbs that are used
both as both nouns and verbs)

the only way to do this is to think about the CONTEXT of the sentence -- just look at the way in which the word is used, and think
about what it means.
there is not going to be any sort of strictly mechanical way to figure this out.

in this sentence, the context is that changes (noun) have some sort of effect. (nothing is actually changing in the current context of
this particular sentence, so it doesn't make sense to consider "changes" as a verb.)

RonPurewal
ManhattanGMAT Staff

Re: GMATPREP- Hawthorne Works

Sat Mar 08, 2014 9:44 am


RonPurewal
ManhattanGMAT Staff

Re: GMATPREP- Hawthorne Works

Sat Nov 01, 2014 7:51 am

if the sentence describes a choice from a set of possibilities, you may see "which + noun";
having reviewed the three proposals for the total cost of the project, we now seek to determine which amount is the most
realistic.

and, of course, sentences with "what" itself acting as a noun/pronoun (i don't know the terms, but they don't matter) are
commonplace:
no one will ever know what happened in those final moments.
we don't know what consumers will think of the new design.
note that the second example is NOT "what consumers". rather, "what" is the object of "consumers will think"

jabgt
Students

Re: GMATPREP- Hawthorne Works

Fri Nov 18, 2016 12:40 am

RonPurewal wrote:

jessietang1987 wrote:Ron: I eliminate the choice E. But I don't know whether my reasoning is right. Plz

Question 40
Question 40
Question 40
Question 40
https://gmatclub.com/forum/as-the-honeybees-stinger-is-heavily-barbed-staying-where-it-78082.html
A:8% B:3% C:15% D:28% E:46%

Difficulty: 75% (hard)


Question Stats: 47% (01:00) correct 53% (01:09) wrong based on 1139 sessions

GMATPill
GMAT Pill Representative
As the honeybees stinger is heavily barbed, staying where it is inserted, this results in the
act of stinging causing the bee to sustain a fatal injury.
A. As the honeybees stinger is heavily barbed, staying where it is inserted, this
results in the act of stinging causing
B. As the heavily barbed stinger of the honeybee stays where it is inserted, with the
result that the act of stinging causes
C. The honeybees stinger, heavily barbed and staying where it is inserted, results in
the fact that the act of stinging causes
D. The heavily barbed stinger of the honeybee stays where it is inserted, and results
in the act of stinging causing
E. The honeybees stinger is heavily barbed and stays where it is inserted, with the
result that the act of stinging causes

Answer is (E) - please reference the video explanation here:


http://www.gmatpill.com/gmat-practice-t ... stion/2407

http://www.beatthegmat.com/as-the-honeybeea-s-stinger-t81041.html

Question 41
GMATGuruNY GMAT Instructor

I received a PM asking me to offer a quick approach.

In A, this has no clear antecedent. Eliminate A.

B is not a complete sentence. Where are the subject and the verb? Eliminate B.

C and D each state that the stinger...results in. A stinger cannot result in something; only an action can result in something.
Eliminate C and D.

The correct answer is E.

GMATGuruNY GMAT Instructor

clock60 wrote:
hi Mitch
great thanks for quick reply. it is very helpful.
if possible one more question
some guys say that :act of stinging causing is wrong
can you elaborate what is wrong in it, is it grammer issue, or meaning, i have no any clues, other that it sounds clumsy?
thanks

D: results in the act of stinging causing


Conveyed meaning:
Something results in THE ACT.
But the intended meaning is NOT that something results in the act but that something results in the CAUSING of a fatal injury.
Since the intended meaning is not conveyed, the construction in D is not viable.

Question 41
https://www.manhattanprep.com/gmat/forums/as-the-honeybeea-s-stinger-is-heavily-barbed-t7194.html

RonPurewal
ManhattanGMAT Staff

Re: As the honeybees stinger is heavily barbed

Wed Jun 24, 2009 7:08 am

bluementor wrote:Question from GMATPrep 2:

As the honeybees stinger is heavily barbed, staying where it is inserted, this results in the act of stinging causing the
bee to sustain a fatal injury.

A. As the honeybees stinger is heavily barbed, staying where it is inserted, this results in the act of stinging causing
B. As the heavily barbed stinger of the honeybee stays where it is inserted, with the result that the act of stinging
causes
C. The honeybees stinger, heavily barbed and staying where it is inserted, results in the fact that the act of stinging
causes
D. The heavily barbed stinger of the honeybee stays where it is inserted, and results in the act of stinging causing
E. The honeybees stinger is heavily barbed and stays where it is inserted, with the result that the act of stinging
causes

OA is E. My queries to the instructors are:

1)Is E correct because of parallelism? i.e. is//stays//causes?


2) In A, B and D, is 'results in..' unidiomatic?

Question 41
Question 41
Question 41
Question 41
therefore, the sentence should still make sense, in context, if you write it as just "preposition + NOUN", without the
"VERBing" modifier.
therefore, the sentence at hand can be reduced to "... results in the act of stinging".
that's incorrect -- the act of stinging is not the result here; the fact that the stinger is heavily barbed does not cause the actual act of
stinging.

go ahead and apply this principle to the examples above:

i've never heard of bees stinging dogs


--> i've never heard of bees
doesn't make sense anymore. wrong.

..results in the act of stinging causing...


--> ..results in the act of stinging
doesn't makes sense anymore. wrong.

i have a picture of my cousin playing hockey.


--> i have a picture of my cousin.
still makes sense, so this one is ok.

jnelson0612
ManhattanGMAT Staff

Re: As the honeybees stinger is heavily barbed

Fri Aug 19, 2011 6:20 pm

Question 41
Question 41
Question 41
Question 41
Re: As the honeybees stinger is heavily barbed

Fri Feb 17, 2012 8:44 pm

Tim,

really really great answers. though I need to clarify on (2)

the problem with "with" was from this post by Ron:

also, the way the gmat uses 'with ...', it must be followed by a noun or noun equivalent. so, for instance, you could say 'with her 15
recordings', but you can't say 'with her 15 recordings disappointing...' (which is no longer a noun phrase).

'with her 15 recordings disappointing' just looks like "with + noun + modifier". It seems incorrect to me only because the
"recordings" are doing the verb-ing "disappointing", but I don't believe that was Ron's intention with that example.

then in our example:

"with the result being"


"with the result that"

both cases look like:

"with + noun + modifier"

how come a present participle modifier is not OK, but a relative pronoun modifier is OK modifying a noun following with?

thanks again Tim

Question 41
RonPurewal
ManhattanGMAT Staff

Re: As the honeybees stinger is heavily barbed

Mon Feb 27, 2012 5:55 am

"with" occupies a rather special place in the hearts of gmac's problem writers. in other words, "with" is NOT used like other
prepositions, and so, accordingly, there are some unique points to absorb about its use.

the clearest example of this special usage is in the non-underlined part of og12 problem 29:
with individual bulls and cows receiving awards, fetching unprecedented prices, and exciting enormous interest

this usage directly violates the principles for the use of other prepositions -- specifically, "with + noun + VERBing" is allowed even
though the VERBing, rather than the noun, is the intended object of "with". (i.e., in the sentence above, prize-stock breeding was
not "with bulls and cows" -- it was specifically with prizes awarded to these animals.)

this is veeeerrry interesting, since identical constructions with other prepositions are definitely incorrect. for instance, i've never
heard of people biting dogs is incorrect, because the intended object of the preposition is the action (the biting) rather than the
noun ("people"); instead, one could write i've never heard that people have bitten dogs.

a similar usage can be found in og12 #23, with a past participle rather than an ING form ("with its weight concentrated...")

so, it is confirmed: "with" has its own special set of rules, independent of (and contradictory to) the rules for other pronouns.
caveat lector!

davetzulin
Forum Guests

Question 41
Re: As the honeybees stinger is heavily barbed

Tue Mar 20, 2012 6:44 pm

thanks again Ron,

So "I've never heard of bees stinging dogs"


should be
"I've never heard that bees sting dogs"

How about this? "I've never heard of bees that sting dogs", i'm guessing this is still wrong.

so in summary is preposition + noun + verbing almost always incorrect? the case you mentioned:

"there is a picture of Tom sleeping"

seems pretty rare on gmat problems anyway

RonPurewal
ManhattanGMAT Staff

Re: As the honeybees stinger is heavily barbed

Fri Mar 23, 2012 11:40 am

agarwalmanoj2000 wrote:Please advise how to identify whether preposition refers directly to the NOUN or not?

Question 41
you have to understand the message that the sentence is trying to convey. there is not going to be a formulaic approach.

1)
He is CEO of an MNC soaring quickly.
He is CEO of an MNC. << Right makes sense

this one would be wrong, unless a multinational corporation can soar through the air.

3)
i've never heard of bees stinging dogs
i've never heard of bees << Right makes sense

but that's not the message that the sentence is trying to convey. you have to use a certain amount of common sense to approach
this stuff.
in this sentence, the speaker is very clearly not implying that he/she has never heard of bees themselves. instead, the speaker is
implying that the fact that bees sting dogs is what's novel or unfamiliar.
so, your analysis here actually shows exactly why this version is wrong.

4)
Retailers reported loss, because of their sales being bad.
Retailers reported loss, because of their sales << Right makes sense

same problem as #3. the issue is not the sales themselves; the issue is the fact that the sales are bad. that message is not properly
conveyed.

5)
Retailers reported loss, because of their sales having been bad.

Question 41
Question 41
Question 41
Question 41
Sat Nov 10, 2012 7:24 pm

thanghnvn wrote:I can not understand how "with" phrase is used in E.

normally, "with" phrase can modify noun or clause.

when "with" phrase modifies a clause, I do not understand the meaning relation between the two. Ron, pls, explain the
meaning relation so that I can realize the correct E next time when I see the "with" phrase used this way.

In this case, "with" does modify the entire clause. It's the reason that "resulting in X" also sounds right (and might be preferred, but
not necessary).

In this case, the "with the result" answers the question, "what happens next?" It's an unusual idiom, but that's probably true of all
idioms.

Also, one of our great instructors just did a blog post on this exact problem:

http://www.manhattangmat.com/blog/index ... rection-2/

zhouyj1089
Forum Guests

Re: As the honeybees stinger is heavily barbed

Question 41
Thu Oct 17, 2013 1:24 am

Hi Instructors,

I have a quick question about the usage of "this".

I read another thread for the question:

Because there are provisions of the new maritime code that provide that even tiny islets can be the basis for claims
to the fisheries and oil fields of large sea areas, they have already stimulatedinternational disputes over uninhabited
islands.

Ron pointed that:

choice (d) uses "this" as a standalone pronoun. that's pretty much never acceptable in a formal written sentence.
if you're going to use "this", you should use it as an adjective: this thing, this finding, this statistic, etc.

Besides, I remember the book, Manhattan 5th SC, mentions that "Do not use 'this' or 'these'/'those' in place of nouns."

According to these rules said either by Ron or Manhattan 5th SC, this sentence "this results in the act of stinging ..." is
grammatically unacceptable, because "this" is used as an noun here. So choice A can be eliminated by this reason.

However, in the article "Comparing Things in GMATPrep Sentence Correction", which is an analytical article of the question
"honeybee's stinger" , Stacey said that

the word 'this' can also refer to an entire idea, including a separate independent clause in the same sentence or a separate
sentence. The power in my house went out. This irritates me. The power in my house went out; this irritates me. Im really saying
that this situation irritates me.

Question 41
Here is link of the article:
https://www.manhattangmat.com/blog/index.php/2012/10/25/comparing-things-in-gmatprep-sentence-correction-2/

According to the rule said by Stacey, this sentence "this results in the act of stinging ..." is grammatically acceptable.

I am so confused. Is "this" a standalone pronoun as "that" and "those", using as adjective as "that" and "those"?

Thanks a lot!

RonPurewal
ManhattanGMAT Staff

Re: As the honeybees stinger is heavily barbed

Thu Oct 17, 2013 4:47 am

My understanding is that "this", in formal English, is not a pronoun at all. It's strictly an adjective.

In almost all less formal English contexts, it is used as a pronoun quite frequently.

I'll see whether Stacey has any examples of "this" as a pronoun in official GMAC materials. But I'd bet not.

bodhisattwabiswas
Forum Guests

Re: As the honeybees stinger is heavily barbed

Question 41
Thu Nov 07, 2013 6:48 am

RonPurewal wrote:

1)
He is CEO of an MNC soaring quickly.
He is CEO of an MNC. << Right makes sense

this one would be wrong, unless a multinational corporation can soar through the air.

I think I understood the concept of the construction <(preposition) + NOUN + VERBing>...thanks to your valuable explanation.
But, I couldn't understand why this particular sentence is wrong.
Here 'He' is the CEO of the actual MNC, nah?
please explain...

RonPurewal
ManhattanGMAT Staff

Re: As the honeybees stinger is heavily barbed

Thu Nov 07, 2013 8:25 am

But, I couldn't understand why this particular sentence is wrong.


Here 'He' is the CEO of the actual MNC, nah?

Question 41
That's the reason why the construction is wrong.

When you have "no comma + __ing", it sticks to the closest noun, whether you like it or not.

RonPurewal
ManhattanGMAT Staff

Re: As the honeybees stinger is heavily barbed

Fri Nov 08, 2013 12:11 pm

bodhisattwabiswas wrote:I thought that the sentence means --- He is the CEO of an MNC, which (the MNC) is soaring
quickly; isn't so?

That's exactly what the sentence means, and exactly why the sentence is nonsense. Companies don't fly through the air.

JIYUS618
Forum Guests
Is "soaring" a adjective?
1.soaring MNC/MNC is soaring
2.Rising rice price/Rice price is rising
these two are the same??
if not,what's the different?
Thank you in advance.

RonPurewal
ManhattanGMAT Staff

Question 41
Re: As the honeybees stinger is heavily barbed

Thu May 01, 2014 8:30 am

I don't think I understand your question.

My answer to your question as written:


#1 is about multinational corporations, while #2 is about prices"so, no, they're not the same.

I don't think that is what you are trying to ask, so please clarify. Thanks.

JIYUS618
Forum Guests

Re: As the honeybees stinger is heavily barbed

Thu May 01, 2014 11:15 am

I mean,
1.soaring MNC = MNC is soaring
2.Rising rice price = Rice price is rising
In 1 and 2, the left phrase both has the same meaning as the right phrase?
if not,what's the different?

what you mean is "soaring MNC" doesn't have the same meaning as "MNC is soaring", while"Rising rice price" have the same
meaning as "Rice price is rising"
as for me, both of them are the same in usage.
It really confuse me.

Question 41
Pls clarify.
Thank you in advance.

RonPurewal
ManhattanGMAT Staff

Re: As the honeybees stinger is heavily barbed

Sun May 04, 2014 12:18 pm

I mean,
1.soaring MNC = MNC is soaring
2.Rising rice price = Rice price is rising

These would both be equivalences, yes.

An exception would only occur if these parts were incorrectly isolated from the context. (E.g., in "the stock price of this MNC is
soaring""a message that makes perfect sense"it's not possible to analyze just the words "this MNC is soaring". In this context, if
you have "this MNC", then you have to keep "the stock price of this MNC"; you can't keep a modifier but get rid of its referent!)

SC312
Forum Guests

Re: As the honeybees stinger is heavily barbed

Question 41
Tue Jun 17, 2014 1:31 am

Ron,

Can you explain the use of "with" in the correct answer choice ?
From earlier posts, I have seen the following 2 usages :-
1. The use of "with" usually signifies a "sense of possession". One of your examples where you explained this is : -
With five all-state players in its backfield, ballard high looks
to shut down opposing offenses completely.

This sentence makes sense because ballard high actually has five all-state players in its backfield, thus justifying the use of "with".

2. Also, in one of your other post you have pointed out that
'with' may be used with a present participle (-ING form) to represent circumstances that are contemporaneous with the action
described in the main clause.

In which category will this option fall or is it a completely different usage ?

RonPurewal
ManhattanGMAT Staff

Re: As the honeybees stinger is heavily barbed

Wed Jun 18, 2014 12:47 pm

Well, if you're talking about a result, you normally use possessive-like constructions.
Something has a result.
You can talk about its results (= an action's results = the results of an action).

Question 41
So, your #1 actually covers this usage.

SC312
Forum Guests

Re: As the honeybees stinger is heavily barbed

Wed Jul 02, 2014 11:37 am

Ron,

Can you confirm whether the following sentence is correct and whether the use of with is correct ?

I have changed the original question since the author did not cite the source of the problem.

In less than a decade, India has become the worlds leading exporter of cotton, selling nearly 20,000 tons a year to Asian markets,
with almost half going to South-East Asian countries.

If the sentence is correct, the use of with refers to a contemporaneous action rather than signifying a sense of possession. Let
me know if my understanding is correct.

Thanks

RonPurewal
ManhattanGMAT Staff

Re: As the honeybees stinger is heavily barbed

Question 41
Thu Jul 03, 2014 8:06 am

Basically. It's a portion/component/aspect of the larger situation described right before it.

Just "contemporaneous" isn't accurate enough, though.


E.g., if two things happened at the same time with equal significance/size/importance/priority in context, then this modifier
construction would be inappropriate. In such cases, a parallel structure would be used instead.

RonPurewal
ManhattanGMAT Staff

Re: As the honeybees stinger is heavily barbed

Thu Jul 03, 2014 8:09 am

More importantly""
Trying to slap explicit labels (e.g., "contemporaneous action") on these things shouldn't be your goal.

In my discussions on here, such labels are a necessary evil, since I have to (attempt to) give somewhat general descriptions.
On the other hand, when it comes to your understanding, these labels are an inappropriate vehicle. They're clumsy, and almost
impossible to apply in real time.

Your goal should just be to understand how the correct examples work.
If you can't easily shove them into a particular category, that's fine""you don't have to! You just have to (a) understand how they
work, and (b) be able to recognize future examples that work like them.

If your thinking is restricted to precise definitions, you won't be able to see beyond the simplest ideas. (Precise definitions are
impossible for anything but the most simple ideas.) If you can think in terms of examples, though, you'll be able to understand much

Question 41
more, on a much deeper level.

E.g., think about how you would define "rude behavior". Would you produce a definition? Of course not; that would basically be
impossible. Instead, you'd just list a whole bunch of examples of rude behavior, and, together, those examples would constitute
your "definition".
That should be your goal here, too. If you collect enough examples of how a particular modifier works, those examples will serve as
a "definition"""which will be much, much better than any actual definition.

divineacclivity
Forum Guests

Re: As the honeybees stinger is heavily barbed

Sat Jul 05, 2014 10:20 am

RonPurewal wrote:if you have "preposition + NOUN + VERBing", then "VERBing" is just a modifier, and can
be dropped without changing the surrounding grammar.
therefore, the sentence should still make sense, in context, if you write it as just "preposition + NOUN",
without the "VERBing" modifier.
therefore, the sentence at hand can be reduced to "... results in the act of stinging".
that's incorrect -- the act of stinging is not the result here; the fact that the stinger is heavily barbed does not cause
the actual act of stinging.

go ahead and apply this principle to the examples above:

i've never heard of bees stinging dogs


--> i've never heard of bees
doesn't make sense anymore. wrong.

Question 41
Question 41
Question 41
Question 41
RonPurewal
ManhattanGMAT Staff

Re: As the honeybees stinger is heavily barbed

Wed Dec 09, 2015 10:16 am

if you're looking at a correctly written sentence, you should ALWAYS be able to figure out what the words are doing. you just
need to use context as your guide.

basically, there's going to be some intended meaning, which should be clear in context.
if you're looking at a CORRECT sentence, then all the words are doing things that convey the intended meaning correctly.

this is a rather humble observation, but it should be enough for you to figure out how everything works (in any correctly written
sentence).

if you're looking at an incorrect sentence and you're not sure how something works, it's not really worth worrying about. remember,
it's an incorrect example! the thing you're looking at might not even be a thing.

RonPurewal
ManhattanGMAT Staff

Re: As the honeybees stinger is heavily barbed

Wed Dec 09, 2015 10:17 am

also, in this case, you have "the act of ___", which by itself confirms that "___" is a noun. (this is just one tiny example of 'you can
figure this stuff out from the context'.)

Question 41
rahulsnh
Students

Re: As the honeybees stinger is heavily barbed

Sat Aug 13, 2016 4:10 am

RonPurewal wrote:the clearest example of this special usage is in the non-underlined part of og12 problem 29:
with individual bulls and cows receiving awards, fetching unprecedented prices, and exciting enormous
interest

this usage directly violates the principles for the use of other prepositions -- specifically, "with + noun + VERBing" is
allowed even though the VERBing, rather than the noun, is the intended object of "with". (i.e., in the sentence above,
prize-stock breeding was not "with bulls and cows" -- it was specifically with prizes awarded to these animals.)

Hi Ron,

Sorry to pull up an old thread but I'm stuck

The way I see the sentence is that prize-stock breeding was with bulls and cows and "receiving ... ", "fetching ... ", and "exciting ... "
modify the bulls and cows. If so, then intended object of "with" is indeed "bulls and cows".

Pls help me see the sentence from your perspective such that I can bridge the gap.

Thanks in advance,
Rahul
PS: Not that one is expected to understand "prize stock breeding" to solve SC but my understanding of context maybe impeding. I

Question 41
see "prize stock breeding" as breeding with chosen stock.

RonPurewal
ManhattanGMAT Staff

Re: As the honeybees stinger is heavily barbed

Sat Aug 20, 2016 5:04 am

think more carefully about the context of that sentence. the "with" is meant to be followed by the whole idea of those actions (the
sentence would be nonsense if it just said "bulls and cows" there).

remember, we're not supposed to discuss OG problems here. we cannot discuss this problem any further. (any further posts
about this OG problem will be deleted.)

RichaChampion
Students

Re: As the honeybees stinger is heavily barbed

Mon Aug 29, 2016 11:45 pm

RonPurewal wrote:

sudaif wrote:can someone please explain the following


"first off, the construction (preposition) + NOUN + VERBing is WRONG, unless the preposition refers directly

Question 41
to the NOUN. (that isn't usually the case, so, if you're in doubt, you should strike choices with this sort of
construction.)

for instance:
i've never heard of bees stinging dogs
WRONG. this is not an issue of whether you've heard of bees themselves; it's an issue of whether you've
heard of their stinging dogs.

...results in the act of stinging causing...


WRONG. this doesn't result in the act of stinging itself; it results in what is caused by the act of stinging.

i have a picture of my cousin playing hockey.


CORRECT. this time, the picture is actually of my cousin, so we're good.

therefore, (a) and (d) are wrong because of "...results in the act of stinging causing..."."

not sure what construction that is....and why it is wrong.

if you have "preposition + NOUN + VERBing", then "VERBing" is just a modifier, and can be dropped
without changing the surrounding grammar.
therefore, the sentence should still make sense, in context, if you write it as just "preposition + NOUN",
without the "VERBing" modifier.
therefore, the sentence at hand can be reduced to "... results in the act of stinging".
that's incorrect -- the act of stinging is not the result here; the fact that the stinger is heavily barbed does not cause
the actual act of stinging.

go ahead and apply this principle to the examples above:

i've never heard of bees stinging dogs

Question 41
--> i've never heard of bees
doesn't make sense anymore. wrong.

..results in the act of stinging causing...


--> ..results in the act of stinging
doesn't makes sense anymore. wrong.

i have a picture of my cousin playing hockey.


--> i have a picture of my cousin.
still makes sense, so this one is ok

Mr. Purewal, when you say this -

"preposition + NOUN + VERBing", then "VERBing" is just a modifier, and can be dropped without changing the surrounding
grammar.

You actually mean to say if we remove VERBing and the sentence make sense then Preposition+Noun+Verbing is correctly used
there else not. Right?

RonPurewal wrote:"

this is veeeerrry interesting, since identical constructions with other prepositions are definitely incorrect. for instance, i've never
heard of people biting dogs is incorrect, because the intended object of the preposition is the action (the biting) rather
than the noun ("people"); instead, one could write i've never heard that people have bitten dogs.

Question 41
Sir, I find the above bold portion very difficult to comprehend. I thinked it for 1 Hour, but failed to completely grasp.

P.S. This is not an OG problem, but a problem from the test pack that is free of cost(2 FREE test provided by GMAT)

RonPurewal
ManhattanGMAT Staff

Re: As the honeybees stinger is heavily barbed

Sat Sep 03, 2016 12:41 am

just forget the "rules", and look at the EXAMPLES. (that's how you should be looking at ALL of this stuff, anyway.) --specifically, the
sentences about "bees stinging dogs" and "my cousin playing hockey".

those sentences aren't very complicated, and their meanings are easy to understand.

"i've never heard of bees"


... this is NOT the same thing.
so, that sentence is wrong.

"i took a picture of my cousin"


... same core message.
so, this sentence works.

RonPurewal

Question 41
ManhattanGMAT Staff

Re: As the honeybees stinger is heavily barbed

Sat Sep 03, 2016 12:41 am

also... two things

1/
when you say "i thought about this for an hour"... i sincerely hope you're exaggerating.

if you're stuck on some idea for more than a couple of minutes, then LEAVE IT, and come back to it LATER (like several days later --
NOT the same day).

if you continue to stare at it, your brain isn't going to suddenly have a "lightbulb moment".

if you actually spent a whole HOUR -- like, 60 literal minutes -- just pondering that one concept, then... you wasted at least 55
minutes of your life.

2/
i saw your signature.
you have a 740, and you seriously want to re-take this test?

that's a mistake.

SooyoungC899
Students

Question 41
Re: As the honeybees stinger is heavily barbed

Tue Sep 27, 2016 8:16 pm

Ron,

I think what you put below is wrong. This structure is grammatically correct.

i've never heard of bees stinging dogs


--> i've never heard of bees
doesn't make sense anymore. wrong.

Another example of this sentence with the same structure:


I have heard of my child punching other kid

This is correct.

RonPurewal
ManhattanGMAT Staff

Re: As the honeybees stinger is heavily barbed

Fri Sep 30, 2016 2:30 pm

no.

Question 41
jabgt Students

Re: As the honeybees stinger is heavily barbed

Thu Nov 17, 2016 9:41 am

RonPurewal wrote:"with" occupies a rather special place in the hearts of gmac's problem writers. in other words,
"with" is NOT used like other prepositions, and so, accordingly, there are some unique points to absorb about
its use.

the clearest example of this special usage is in the non-underlined part of og12 problem 29:
with individual bulls and cows receiving awards, fetching unprecedented prices, and exciting enormous
interest

this usage directly violates the principles for the use of other prepositions -- specifically, "with + noun + VERBing" is
allowed even though the VERBing, rather than the noun, is the intended object of "with". (i.e., in the sentence above,
prize-stock breeding was not "with bulls and cows" -- it was specifically with prizes awarded to these animals.)

this is veeeerrry interesting, since identical constructions with other prepositions are definitely incorrect. for instance,
i've never heard of people biting dogs is incorrect, because the intended object of the preposition is the action (the
biting) rather than the noun ("people"); instead, one could write i've never heard that people have bitten dogs.

a similar usage can be found in og12 #23, with a past participle rather than an ING form ("with its weight
concentrated...")

so, it is confirmed: "with" has its own special set of rules, independent of (and contradictory to) the rules for other
pronouns.
caveat lector!

Question 41
Dear Ron sir,

Is the use of "after" in "Nine months after the county banned jet skis and other water bikes from the tranquil waters of Puget Sound,
a judge overturned the ban on the ground ..." (cited from one Prep problem ) analogous to "with" in "with its weight
concentrated..." (the above example you gave to us)? Or is it a conjunction? I guess it is one preposition because of "nine months"
before it. Or should I take "nine month after" the whole stuff as a conjunction?

(Ron sir, I ask this question is not to ask for the noun of the function of "nine months after" since you have taught us that the term
itself doesn't matter. -- I not only read your lectures from this forum and watch "Thursday" sessions, but also take notes each time.)
After I learn the above use of "with" , I recall I have encountered that "after" sentence. I have checked dictionary as well, but
unfortunately I haven't found such example.)

Thank you!

RonPurewal
ManhattanGMAT Staff

Re: As the honeybees stinger is heavily barbed

Wed Dec 07, 2016 10:17 am

that usage of "after" is followed by a complete sentence, so, there is definitely no analogy to "with" (which cannot be followed by a
complete sentence under any circumstances).

Question 41

Das könnte Ihnen auch gefallen